150
TRƯỜNG ĐÔNG TOÁN HỌC MIỀN NAM 2017 NHỮNG BÀI TOÁN HAY VÀ KHÓ LƯU GIANG NAM – TRẦN BÁ ĐẠT VÕ THÀNH ĐẠT – LƯƠNG VĂN KHẢI – PHẠM THỊ HỒNG NHUNG Đội Huấn luyện viên Trường đông Toán học miền Nam 2017

75,j1*Á 1*72~1+Z& 0,N11$0 1+|1*%},72~1+$

  • Upload
    others

  • View
    3

  • Download
    0

Embed Size (px)

Citation preview

TRƯỜNG ĐÔNG TOÁN HỌCMIỀN NAM 2017

NHỮNG BÀI TOÁN HAY VÀ KHÓ

LƯU GIANG NAM – TRẦN BÁ ĐẠT

VÕ THÀNH ĐẠT – LƯƠNG VĂN KHẢI – PHẠM THỊ HỒNG NHUNG

Đội Huấn luyện viên Trường đông Toán học miền Nam 2017

LƯU GIANG NAM - TRẦN BÁ ĐẠT

VÕ THÀNH ĐẠT - LƯƠNG VĂN KHẢI - PHẠM THỊ HỒNG NHUNG

Đội HLV Trường đông Toán học miền Nam 2017

TRƯỜNG ĐÔNG TOÁN HỌC

MIỀN NAM 2017

Những bài toán hay và khó

Tháng 12/ 2017

Mục lục

1 ĐỀ BÀI 9

1.1 Đại số . . . . . . . . . . . . . . . . . . . . . . . . . . . . . . . . . 9

1.2 Hình học . . . . . . . . . . . . . . . . . . . . . . . . . . . . . . . . 12

1.3 Phương trình hàm - Dãy số . . . . . . . . . . . . . . . . . . . . . . 14

1.4 Số học . . . . . . . . . . . . . . . . . . . . . . . . . . . . . . . . . 17

1.5 Tổ hợp . . . . . . . . . . . . . . . . . . . . . . . . . . . . . . . . . 19

2 HƯỚNG DẪN GIẢI 25

2.1 Đại số . . . . . . . . . . . . . . . . . . . . . . . . . . . . . . . . . 25

2.2 Hình học . . . . . . . . . . . . . . . . . . . . . . . . . . . . . . . . 51

2.3 Phương trình hàm - Dãy số . . . . . . . . . . . . . . . . . . . . . . 76

2.4 Số học . . . . . . . . . . . . . . . . . . . . . . . . . . . . . . . . . 109

2.5 Tổ hợp . . . . . . . . . . . . . . . . . . . . . . . . . . . . . . . . . 121

LỜI NÓI ĐẦU

Nét đăc trưng của các chương trình Gặp gỡ Toán học và Trường đông Toánhọc miền Nam chính là đội Huấn luyện viên. Là các sinh viên trưởng thành từphong trào Toán Olympic, chúng tôi quay lại giúp đỡ các bạn học sinh chuyênToán với tất cả nhiệt huyết, đam mê còn nguyên vẹn từ ngày xưa, ngày màchúng tôi ăn ngủ cùng những bài toán, nói những câu chuyện chỉ có những bàitoán, giải những bài toán mỗi ngày và đọc những bài viết về phương pháp giảitoán mỗi đêm. Hơn ai hết, chúng tôi hiểu rõ những gì mà các học sinh chuyênToán trải qua, và chúng tôi muốn chia sẻ những khó khăn ấy cùng các bạn.

Tập san này ra đời không ngoài mục đích đó. Chúng tôi muốn đem nhữnggì có ở Trường đông Toán học miền Nam tới thật nhiều các bạn học sinh, để cácbạn, đặc biệt là những bạn chưa có điều kiện tham gia chương trình, đúng nhưtinh thần Bring Math to Everyone của thầy Trần Nam Dũng, đúng với nhữnggiá trị cốt lõi mà Gặp gỡ Toán học và Trường đông Toán học miền Nam. Trongtập san này, các bài toán được chia theo lĩnh vực, đề bài được đưa ra trước đểcác bạn học sinh suy nghĩ, sau đó là phần hướng dẫn giải. Có những bài sẽ cólời giải đầy đủ, có những bài chỉ có phần hướng dẫn sơ lược, và có những bàisẽ có thêm phần nhận xét và mở rộng. Xuất hiện ở đây không chỉ có những bàitoán được dùng trong các bài giảng của các thầy mà còn là những bài toán đượcthảo luận sau giờ học, trong giờ giải lao của các bạn học sinh.

Các biên tập viên của tập san này chính là các Huấn luyện viên của Trườngđông Toán học miền Nam năm nay:

• Bạn Lưu Giang Nam (sinh viên khoa Toán - Tin học trường ĐH Khoa họcTự nhiên, ĐHQG Tp. HCM) biên tập phần Phương trình hàm - Dãy số.

• Bạn Trần Bá Đạt (sinh viên khoa Toán - Tin học trường ĐH Sư phạm Tp.HCM) biên tập phần Tổ hợp.

• Bạn Võ Thành Đạt (sinh viên khoa Toán - Tin học trường ĐH Khoa học Tự

8 MỤC LỤC

nhiên, ĐHQG Tp. HCM) biên tập phần Bất đẳng thức - Đa thức.

• Bạn Lương Văn Khải (sinh viên khoa Toán - Tin học trường ĐH Khoa họcTự nhiên, ĐHQG Tp. HCM) biên tập phần Hình học.

• Bạn Phạm Thị Hồng Nhung (cựu học sinh trường THPT chuyên Lê QuýĐôn, tỉnh Bà Rịa - Vũng Tàu) biên tập phần Số học.

Chúng tôi xin gửi lời cảm ơn tới thầy Trần Nam Dũng (ĐH Khoa học Tựnhiên, ĐHQG Tp. HCM), thầy Trần Quang Hùng (THPT chuyên Khoa học Tựnhiên, ĐH Khoa học Tự nhiên, ĐHQG Hà Nội), thầy Nguyễn Song Minh (TitanEducation Hà Nội), thầy Võ Quốc Bá Cẩn (Archimedes Academy, Hà Nội), thầyLê Phúc Lữ (FPT Software Tp. HCM) đã cung cấp các bài giảng. Cảm ơn các bạnNgô Hoàng Anh, Phạm Hoàng Minh (học sinh chuyên Toán trường Phổ thôngNăng khiếu, ĐHQG Tp. HCM), bạn Nguyễn Minh Uyên (học sinh chuyên Toántrường THPT chuyên Thoại Ngọc Hầu, An Giang), bạn Lư Thương Thương (họcsinh chuyên Toán trường THPT chuyên Lê Hồng Phong, TP Hồ Chí Minh) đãnhiệt tình hỗ trợ ban biên tập. Những bài nằng trong đề chính thức, đề đề nghịTrường Đông hay các đề tiêu thụ sẽ được ghi rõ nguồn, những bài còn lại lànhững bài tập hay của các thầy dạy tại Trường Đông miền Nam 2017 (do cácbạn học sinh và huấn luyện viên tham dự buổi học ghi chép lại) và những bàimở rộng, thú vị xung quanh những bài có ghi nguồn. Cảm ơn đơn vị tổ chứcchương trình là Công ty cổ phần Giáo dục Titan - Titan Education đã tạo mọiđiều kiện để chúng tôi hoàn thành được tập san này.

Chúng tôi rất mong nhận được sự đóng góp của bạn đọc để những ấn phẩmsau được hoàn thiện hơn. Mọi đóng góp xin gửi qua hộp thư ở fanpage chườngtrình https://www.facebook.com/gapgotoanhoc/. Những ý kiến của các bạnsẽ là những kinh nghiệm lớn cho chúng tôi trong những lần biên tập sách tiếptheo.

Cảm ơn tất cả các bạn!

Chương 1

ĐỀ BÀI

1.1 Đại số

Bài 1. (Đề tiêu thụ ngày 1 trường đông miền Nam) Cho a, b, c là ba số thực saocho (a− b)(b− c)(c− a) 6= 0. Tìm giá trị nhỏ nhất của biểu thức

P =(a2 + b2 + c2 + ab+ bc+ ca

) [ 1

(a− b)2+

1

(b− c)2+

1

(c− a)2

].

Bài 2. Cho a, b, c là các số dương thoả mãn abc = 1. Chứng minh rằng

a√a+ 1

+b√b+ 1

+c√c+ 1

≥ 3√

2

2.

Bài 3. (Đề đề nghị trường đông miền Nam)

1. Cho a, b, c là ba số thực. Đặt s = a2 + bc − ab, r = b2 + ca − bc, t =c2 + ab− ca. Chứng minh rằng sr + ts+ rt = a3b+ b3c+ c3a. Từ đó suy ra(a2 + b2 + c2)

2 ≥ 3 (a3b+ b3c+ c3a).

2. Cho x, y, z là các số thực dương thoả mãn điều kiện xy + yz + zx = 1.Chứng minh rằng

3−√

3 +x2

y+y2

z+z2

x≥ (x+ y + z)2.

10 ĐỀ BÀI

Bài 4. (Đề đề nghị trường đông miền Nam) Cho a, b, c là ba số thực dương. Xétbất đẳng thức(

a2 + bc

b+ c

)n+

(b2 + ca

c+ a

)n+

(c2 + ab

a+ b

)n≥ an + bn + cn.

1. Chứng minh minh bất đẳng thức trên với n = 1.

2. Với n = 2 thì bất đẳng thức trên còn đúng không? Nếu có, hãy chứngminh. Nếu không, hãy chỉ ra phản ví dụ.

Bài 5. (Đề chính thức trường đông Trung Trung Bộ) Cho a, b, c là các số thựcdương thoả mãn điều kiện a+ b+ c = 3. Chứng minh rằng

9

abc≥ 2

(a

c+c

b+b

a

)+ 3.

Bài 6. Tìm số thực k nhỏ nhất sao cho bất đẳng thức

xyz + 2 + k[(x− 1)2 + (y − 1)2 + (z − 1)2

]≥ x+ y + z

đúng với mọi x, y, z không âm.

Bài 7. (Đề đề nghị trường đông miền Nam) Cho a, b, c là ba cạnh của một tamgiác, chứng minh bất đẳng thức sau:

a2b(a− b) + b2c(b− c) + c2a(c− a) ≥ 0.

Bài 8. (Đề đề nghị trường đông miền Nam) Cho a, b, c là các số thực không âmthoả mãn điều kiện a+ b+ c = 3. Chứng minh rằng

a

b3 + 16+

b

c3 + 16+

c

a3 + 16≥ 1

6.

Bài 9. Chứng minh rằng với mọi số thực a, b, c, ta đều có

(b+ c− a)2(c+ a− b)2(a+ b− c)2 ≥ (b2 + c2 − a2)(c2 + a2 − b2)(a2 + b2 − c2).

Bài 10. Cho a, b, c, d là các số thực dương. Chứng minh rằng√ab+ ac+ ad+ bc+ bd+ cd

6≥ 3

√abc+ bcd+ cda+ dab

4.

1.1 Đại số 11

Bài 11. Tìm tất cả các đa thức P (x) ∈ R[x] thoả mãn

P (x)P (2x2) = P (x3 + x).

Bài 12. Cho đa thức P (x) ∈ R[x] và P (x) ≥ 0 với mọi x ∈ R. Chứng minh rằngđa thức P (x) có thể biểu diễn dưới dạng

P (x) = (A(x))2 + (B(x))2

trong đó A(x), B(x) cũng là các đa thức có hệ số thực.

Bài 13. Cho đa thức

f(x) = xn + an−2xn−2 + an−3x

n−3 + . . .+ a1x+ a0 ∈ R[x].

Chứng minh rằng tồn tại i ∈ {1, 2, . . . , n} sao cho |f(i)| ≥ n!

Cin

.

Bài 14. Cho số nguyên n ≥ 3 và P (x) ∈ R[x] thoả mãn

P (x) = xn + an−3xn−3 + an−4x

n−4 + . . .+ a1x+ a0.

Biết ít nhất một số trong các số a0, a1, . . . , an−3 khác 0. Chứng minh rằng P (x)không thể có toàn nghiệm thực.

Bài 15. Cho n là số nguyên dương chẵn. Một đa thức monic bậc n có n nghiệmthực (không nhất thiết phân biệt). Giả sử y là số thực dương thoả với mọi sốthực t < y, ta có P (t) > 0. Chứng minh rằng n

√P (0)− n

√P (y) ≥ y.

Bài 16. Tìm các đa thức f(x), g(x) hệ số nguyên thoả mãn

f (g(x)) = x2017 + 2018x+ 1.

Bài 17. Cho đa thức hệ số nguyên P (x) = anxn+an−1x

n−1 + . . .+a1x+a0 (an 6=

0). Giả sử tồn tại m sao cho m ≥ maxi∈0,n

{∣∣∣∣ aian∣∣∣∣} + 2 và P (m) là số nguyên tố.

Chứng minh rằng P (x) bất khả quy trên Z.

Bài 18. Cho số nguyên dương n. Tìm tất cả đa thức P thoả mãn đồng thời haiđiều kiện sau:

i. P (x) = anxn + an−1x

n−1 + . . . + a1x + a0 trong đó {a0, a1, . . . , an} ={0, 1, 2, . . . , n}.

ii. Đa thức P có n nghiệm thực phân biệt.

12 ĐỀ BÀI

Bài 19. Tìm tất cả các đa thức hệ số nguyên f sao cho n|m ⇒ f(n)|f(m) vớimọi các số nguyên dương m,n.

Bài 20. (Đề chính thức ngày 1 trường đông miền Nam) Cho đa thức monic P (x)bậc n > 1 (tức P có hệ số của bậc lớn nhất bằng 1) có n nghiệm thực x1, x2, ...xnphân biệt khác 0. Chứng minh rằng:

1

x1P ′(x1)+

1

x2P ′(x2)+ ...+

1

xnP ′(xn)=

(−1)n+1

x1x2...xn.

1.2 Hình học

Bài 1. (Trần Quang Hùng) Cho 4ABC. điểm P di chuyển trên cạnh PC. Q,Rlần lượt là hai điểm đối xứng với P qua CA,AB. Lấy điểm M nằm trên đườngtròn ngoại tiếp 4AQR sao cho AM ‖ BC. Chứng minh đường thẳng PM điqua một điểm cố định khi P di chuyển trên cạnh PC.

Bài 2. (Trần Quang Hùng) Cho 4ABC có đường cao AH. Giả sử đường trònđường kính BC tiếp xúc đường tròn nội tiếp 4ABC. Chứng minh rằng AH +BC = AB + AC.

Bài 3. (Trần Quang Hùng) Cho 4ABC có hai đường cao BE và CF . Đườngtròn bàng tiếp góc A là (Ia). Hai tiếp tuyến chung trong của (AEF ) và (Ia) cắtBC tại P và Q. Chứng minh rằng BP = CQ.

Bài 4. (Trần Quang Hùng) Cho đường tròn (O1) và (O2) ngoài nhau có ABlà một tiếp tuyến cung ngoài và CD là một tiếp tuyến chung trong. Gọi P =AB ∩ CD,Q = AD ∩BC. Chứng minh rằng PQ ⊥ O1O2.

Bài 5. (Trần Quang Hùng) Cho tứ giác ABCD nội tiếp đường tròn tâm O vàmột điểm P thuộc cung CD không chứa A và B. Gọi E = PA ∩ BD,G =BP ∩AC,H = AP ∩CD,F = BP ∩CD,Q = EF ∩HG. Chứng minh rằng PQluôn đi qua một điểm cố định khi P di chuyển.

Bài 6. (Trần Quang Hùng) CCho tam giác ABC nội tiếp đường tròn tâm O cốđịnh có hai điểm BC cố định, A di chuyển trên (O). Gọi AD,BE,CF là bađường cao của tam giác cắt nhau tại H. DE,DF lần lượt cắt HB,HC tại Q, R.Gọi M là trung điểm QR. Chứng minh rằng HM đi qua điểm cố định.

Bài 7. (Trần Quang Hùng) Cho4ABC nội tiếp đường tròn (O) cố định có B, Ccố định và A di động trên (O). Gọi E, F lần lượt là điểm đồi xứng của B, C quaCA, AB. Gọi M = CE ∩ AB,N = BF ∩ AC. Chứng minh rằng đường thẳngqua A vuông góc với MN đi qua một điểm cố định.

1.2 Hình học 13

Bài 8. (Đề chính thức trường đông Trung Trung Bộ) Cho tam giác ABC nhọn,BE,CF là các đường cao. M là trung điểm của BC. N là giao điểm của AMvà EF . Gọi X là hình chiếu của N lên BC. Y, Ztheo thứ tự là hình chiếu của Xtrên AB,AC .Chứng minh rằng N là trực tâm tam giác AY Z.

Bài 9. (Trần Quang Hùng) Cho 4ABC có đường trung tuyến AM . Đường caoBE cắt đường trung tuyến AM tại P . Lấy Q sao cho QE ⊥ AM,CQ ⊥ AB.Chứng minh rằng AQ ⊥ CP .

Bài 10. (Đề chính thức ngày 1 trường đông miền Nam) Cho tam giác ABC nộitiếp đường tròn (O). Tiếp tuyến tại B,C của đường tròn (O) cắt nhau tại T . GọiM,N lần lượt là các điểm thuộc tia BT,CT sao cho BM = BC = CN . Đườngthẳng MN cắt CA,AB theo thứ tự tại E,F ;BE giao CT tại P,CF giao BT tạiQ. Chứng minh rằng AP = AQ.

Bài 11. (Đề chính thức ngày 2 trường đông miền Nam) Cho hai đường tròn (O1)và (O2) tiếp xúc ngoài tại M . Một đường thẳng cắt (O1) tại A,B và tiếp xúc với(O2) tại E (B nằm giữa A và E). Đường thẳng EM cắt (O1) tại điểm J khácM . C là một điểm thuộc cung MJ không chứa A,B của (O1) (C khác M và J).Kẻ tiếp tuyến CF với đường tròn (O2) (F là tiếp điểm) sao cho các đoạn thẳngCF và MJ không cắt nhau. Gọi I là giao điểm của các đường thẳng CJ và EF ,K là giao điểm khác A của đường thẳng AI và đường tròn (O1). Chứng minhrằng:

1. Tứ giác MCFI là tứ giác nội tiếp và JA2 = JI2 = JM.JE.

2. CI là phân giác ngoài tại C của tam giác ABC.

3. K là tâm đường tròn ngoại tiếp của tam giác BCI.

Bài 12. (Đề đề nghị trường đông Trung Trung Bộ) Cho tam giác ABC. Đườngtròn (K) bất kỳ tiếp xúc đoạn thẳng AC,AB lần lượt tại E,F . (K) cắt đoạnthẳng BC tại M,N sao cho N nằm giữa B và M . FM giao EN tại I. Đườngtròn ngoại tiếp các tam giác IFN và IEM cắt nhau tại J khác I. Chứng minhrằng IJ đi qua A và KJ vuông góc IJ .

Bài 13. (Đề đề nghị trường đông miền Nam) Cho (I) là đường tròn nội tiếp tamgiác nhọn ABC, tiếp xúc BC,CA,AB ở D,E, F . (O) là đường tròn ngoại tiếptam giác ABC. Lấy D′ đối xứng với D qua EF .

1. Chứng minh rằng AD′, BC,OI đồng quy.

2. Gọi H, J lần lượt là trực tâm các tam giác ABC và AEF . Gọi R là giaoHJ và ID, T là giao D′J và OI. Chứng minh rằng D′, R, T, I đồng viên.

14 ĐỀ BÀI

Bài 14. (Trần Quang Hùng) Cho tam giác ABC,P là điểm bất kỳ. A1 là hìnhchiếu của P lên BC.A2 là trung điểm AA1.A2P cắt BC tại A3.A4 đối xứng A1

qua A3. Chứng minh rằng PA4 luôn đi qua một điểm cố định.

Bài 15. (Tây Ninh 2017) Cho năm điểm A, B, C, D và E cùng nằm trên mộtđường tròn. Gọi M , N , P và Q lần lượt là hình chiếu vuông góc của E xuốngcác đường thẳng AB, BC, CD và DA. Gọi hình chiếu vuông góc của E xuốngcác đường thẳng MN , NP , PQ, QM lần lượt là R, S, T và U cùng nằm trênmột đường thẳng.

1.3 Phương trình hàm - Dãy số

Bài 1. (Đề chính thức ngày 2 trường đông miền Nam) Tìm các hàm f : R → Rthoả mãn:

f(x2) + f(xy) = f(x)f(y) + yf(x) + xf(x+ y), ∀ xy ∈ R.

Bài 2. (Đề tiêu thụ bài giảng trường đông miền Nam) Cho hàm số :f : N → Nthoả

f(f(n)) + f(n) = 6n+ 4.

1. Tính f(2017)

2. Tìm tất cả các hàm f thoả mãn.

Bài 3. (Đề tiêu thụ bài giảng trường đông miền Nam) Cho dãy số

(xn) :

{x1 = x2 = 97

xn+2 = xn+1xn +√(

x2n+1 − 1)

(x2n − 1) ∀n ≥ 1.

1. Chứng minh 2 + 2xn là số chính phương.

2. Chứng minh 2 +√

2 + 2xn là số chính phương,

Bài 4. (Đề tiêu thụ bài giảng trường đông miền Nam) Xét dãy số nguyên {an}∞n=1

thoả mãn:

a1 = 2; a2 = 7;−1

2< an+2 −

a2n+1

an≤ 1

2.

Chứng minh rằng với mọi n > 1,ta có an là số lẻ.

1.3 Phương trình hàm - Dãy số 15

Bài 5. Tìm f : R→ R thoả mãn:

f(x3 + f(y)) = y + f 3(x) ∀ x, y ∈ R.

Bài 6. Tìm tất cả các hàm f : R→ R thoả mãn:

f(xf(x+ y)) = f(yf(x)) + x2, ∀x, y ∈ R.

Bài 7. (Đề tiêu thụ bài giảng trường đông miền Nam) Tìm tất cả hàm số f : R→R thoả:

f(f(y) + x2 + 1) + 2x = y + (f(x+ 1))2, ∀x, y ∈ R.

Bài 8. Tìm tất cả các hàm f(x) : [1; +∞)→ [1; +∞) thoả{x ≤ f(x) ≤ 2x+ 2

xf(x+ 1) = (f(x))2 − 1

Bài 9. Tìm tất cả các hàm f : R→ R thoả mãn:

f(yf(x)− x) = f(x)f(y) + 2x, ∀x, y ∈ R.

Bài 10. Tìm f: R+ → R+ thoả mãn:

f(x+ y) + f(x).f(y) = f(xy) + f(x) + f(y).

Bài 11. (Đề tiêu thụ bài giảng trường đông miền Nam) Tìm các hàm f : Q2 → Qthoả mãn

f(x, y) + f(y, z) + f(z, x) = f(0, x+ y + z).

với mọi x, y, z ∈ Q.

Bài 12. Tìm f : R⇒ R thoả:

f(x)f(yf(x)− 1) = x2f(y)− f(x), ∀x, y ∈ R

Bài 13. Cho f : Q2 → R+ với Q2 = {(x, y)|x, y ∈ Q}. Giả sử f thoả mãn cácđiều kiện sau:

1. f(a, b, c) = f(a, c)f(b, c), ∀a, b, c ∈ Q.

2. f(c, ab) = f(c, a)f(c, b), ∀a, b, c ∈ Q.

3. f(a, 1− a) = 1, ∀a ∈ Q.

16 ĐỀ BÀI

Chứng minh rằng f(a, a) = f(a,−a) = 1 và f(a, b)f(b, a) = 1.

Bài 14. Cho hàm số f : N? × N? → N? (với N? = N ∪ {0}) thoả mãn các điềukiện sau:

1. f(0, x) = x+ 1

2. f(x+ 1, 0) = f(x, 1)

3. f(x+ 1, y + 1) = f(x, f(x+ 1, y))

Tính f(1, 2017), f(2, 2017), f(3, 2017), f(4, 2017).

Bài 15. Cho dãy số (xn) được xác định bởi:{x1 = 4

xn+1 = x2n − 2,∀ ∈ N∗

Tínhlim

n→+∞

xn+1

x1x2x3...xn

Bài 16. Cho dãy số (un) được xác định bởi công thức truy hồi : u1 = α ∈ R

un+1 =u3n + 9un − 6

3u2n − 6un + 7,∀α ∈ N∗

Tìm α để dãy số (un) có giới hạn hữu hạn khi và tìm giới hạn của dãy số trongcác trường hợp đó.

Bài 17. Cho hai dãy số (an), (bn) được xác định bởi hệ thức truy hồi:

a1 = 3, b1 = 2, an+1 = a2n + 2b2n, bn+1 = 2anbn,∀n ∈ N∗

Tìmlim

n→+∞2n√bn; lim

n→+∞2n√a1a2...an

Bài 18. Cho dãy số (xn) được xác định bởi hệ thức truy hồi: x1 = a ≥ 1

xn+1 =x2n − 2xn

2

[xn]2,∀n ∈ N∗

Chứng minh dãy số (xn) đã cho có giới hạn hữu hạn khi n→ +∞.

1.4 Số học 17

Bài 19. Tìm tất cả các hàm số f : R→ R thoả mãn:

f(yf(x+ y) + f(x)) = 4x+ 2yf(x+ y), ∀x, y ∈ R.

Bài 20. Đặt I = [0, 1] và G = {(x, y)|x, y ∈ I}. Tìm tất cả các hàm số f : G→ Ithoả mãn với mọi x, y, z ∈ I ta có:

1. f(f(x, y), z) = f(x, f(y, z)),

2. f(x, 1) = x, f(1, y) = y,

3. f(zx, zy) = zkf(x, y).

với k là một số thực dương không liên quan đến x, y, z.

1.4 Số học

Bài 1. (Đề chính thức ngày 2 trường đông miền Nam) Biết rằng với dãy nguyêndương 1 < k1 < k2 < ... < kn và dãy nguyên tương ứng s1, s2, ..., sn, với mọi sốnguyên dương N đều tồn tại i ∈ {1, 2, ..., n} sao cho N ≡ si (mod ki).

1. Tìm dãy {kn} và {sn} thoả mãn khi k1 = 2 và khi k1 = 3.

2. Chứng minh rằngn∑i=1

1

ki> 1.

3. Tìm n nhỏ nhất để có các dãy TMĐK đề bài.

Bài 2. (Đề tiêu thụ bài giảng trường đông miền Nam) Tìm n ∈ N và n > 1 sao

cho 3n − 1... n3.

Bài 3. (Đề tiêu thụ bài giảng trường đông miền Nam) Chứng minh rằng tồn tạivô số số nguyên tố dạng 2nk + 1 với k nguyên dương và n ≥ 2017.

Bài 4. (Đề tiêu thụ bài giảng trường đông miền Nam) Tìm số nguyên dương n

nhỏ nhất sao cho 1n + 2n + . . .+ 2016n 6 ... 2017.

Bài 5. (Đề tiêu thụ bài giảng trường đông miền Nam) Cho 9 số nguyên dươngphân biệt d1, d2, . . . , d9 và đa thức P (x) = (x + d1)(x + d2) . . . (x + d9). Chứng

18 ĐỀ BÀI

minh rằng có số N nguyên dương sao cho ∀x ≥ N thì P (x) có ước nguyên tốlớn hơn 20.

Bài 6. (Đề tiêu thụ bài giảng trường đông miền Nam) Cho P (x) ∈ Z[x]. Biếta1, a2, . . . , an là các số nguyên thoả mãn: ∀x ∈ Z,∃i ∈ {1; 2; . . . ;n} sao cho

P (x)... ai. Chứng minh rằng ∃j : P (x)

... aj ∀x ∈ Z.

Bài 7. (Đề tiêu thụ bài giảng trường đông miền Nam) Tìm các số nguyên dươngn sao cho với mọi số nguyên dương k, tồn tại số tự nhiên a sao cho a3 + a − kchia hết cho n.

Bài 8. (Đề tiêu thụ bài giảng trường đông miền Nam) Tìm đa thức P (x) hệ sốnguyên biết với mọi số nguyên tố p, a, b nguyên dương thì ab ≡ 1 (mod p) ⇒P (a).P (b) ≡ 1 (mod p).

Bài 9. (Đề tiêu thụ bài giảng trường đông miền Nam) An và Bảo cùng nhau chơimột trò chơi: họ lần lượt viết các số tuỳ thích lên bảng thành một dòng, mỗingười 3 số, An viết trước. Sau đó Bảo "nhường" An điền dấu + hoặc − tuỳ ý vàogiữa các số đã viết. An thắng nếu kết quả trên bảng không chia hết cho bất cứsố tự nhiên nào từ 11 đến 18. Bảo thắng nếu xảy ra trường hợp ngược lại. Annói rằng mình kiểm soát nhiều hơn, nên chắc chắn chiến thắng. Bạn có đồng ýkhông? Tại sao?

Bài 10. (Đề đề nghị trường đông Trung Trung Bộ) Giả sử N∗ phân hoạch thành 3dãy tăng {an}, {bn}, {cn} thoả mãn:

i. can = bn + 1

ii. an+1 > bn

iii. cn+1cn − (n+ 1)cn+1 − ncn chẵn.

Chứng minh rằng an = n2.

Bài 11. (Nguyễn Song Minh) Tìm các đa thức thoả điều kiện: a2 − b2 ∈ Q thìP (a)− P (b) ∈ Q.

Bài 12. (Nguyễn Song Minh) Tìm tất cả đa thức P (x) hệ số nguyên thoả mãn

2n | P (3n) ∀n ∈ N∗.

Bài 13. (Nguyễn Song Minh) Tìm các đa thức P (x) hệ số nguyên, a, b nguyêndương và a > b sao cho:

P (n) | an − bn ∀n ∈ N∗.

1.5 Tổ hợp 19

Bài 14. (Nguyễn Song Minh) Tìm tất cả các đa thức P (x) hệ số nguyên sao cho

(P (n);P (2017n)) = 1 ∀n ∈ Z+.

Bài 15. Cho số nguyên dương d. Gọi f(d) là số nguyên dương nhỏ nhất có đúng

d ước nguyên dương. Chứng minh rằng f(2k+1)... f(2k) ∀k ∈ N.

1.5 Tổ hợp

Bài 1. (Đề đề nghị trường đông miền Nam) 23 người bạn muốn cùng nhau chơibóng đá. Họ sẽ phải chọn ra một người làm trọng tài và 22 người còn lại chialàm hai đội đá với nhau. Họ muốn chia sao cho tổng cân nặng của mỗi đội làbằng nhau. Giả sử cân nặng của từng người trong số 23 người là các số nguyêndương và với bất kì cách chọn trọng tài nào thì họ cũng có thể chia thành haiđội mà tổng cân nặng của mỗi đội bằng nhau. Chứng minh rằng 23 người nàycó cân nặng bằng nhau.

Bài 2. (Đề đề nghị trường đông miền Nam) Bạn An chơi trò chơi xếp hình vớiluật chơi như sau: Cho một hình vuông 4× 4 chia thành 16 ô, có 15 mảnh ghépvà một ô trống. Trong mỗi bước chơi, An sẽ được phép trượt các mảnh ghép vàoô trống để thu được hình mới. Bạn An sẽ thắng nếu sau hữu hạn bước trượt, Anthu được hình như sau:

Hỏi An có thể chiến thắng nếu hình ban đầu là hình sau hay không?

20 ĐỀ BÀI

Bài 3. (Đề đề nghị trường đông miền Nam) Cho một dãy vô hạn a1, a2, .., an, ...với a1 = 1. Biết rằng với mỗi n > 1 thì:

• Nếu ước lẻ lớn nhất của n đồng dư với 1 module 4 thì an = an−1 + 1.

• Nếu ước lẻ lớn nhất của n đồng dư với 3 module 4 thì an = an−1 − 1.

1. Chứng minh rằng trong dãy số đó mỗi số nguyên dương xuất hiện vô sốlần.

2. Đặt bn = mini∈N(ai = n). Tìm một công thức tính bn theo n.

Bài 4. Cho lục giác đều ABCDEF cạnh 1 được điền các số như hình vẽ.

0

1

0

1

1

2

A

BF

C

D

E

1.5 Tổ hợp 21

Có một con ếch ở vị trí A nhảy xung quanh các đỉnh của đa giác với độ dài cácbước nhảy nguyên. Gọi m là số cách nhảy của ếch sao cho tổng các số nó nhảyqua là 2017. Chứng minh rằng m không là số chính phương.

Bài 5. Cho hai dung dịch A và B thoả mãn:

i. Số đo khối lượng của 1 lít A bằng số đo thể tích của 1kg B.

ii. p lít dung dịch A nặng bằng q lít dung dịch B với p, q là số nguyên tố phânbiệt.

Người ta chia các dung dịch A và B vào các bình giống nhau chứa 1 lít dungdịch và vỏ bình nặng 1kg. Chứng minh rằng có duy nhất 1 cách để ghép cácbình cùng loại A hoặc B với nhau sao cho tổng khối lượng thuộc (2017, 2018).

Bài 6. Có 2020 người đến một buổi tiệc được chia vào các phòng khác nhau saocho:

i. Không người nào trong một phòng quen biết tất cả các người trong phòngđó

ii. Trong nhóm 3 người bất kì thuộc cùng một phòng, luôn tồn tại ít nhất 2người không quen biết nhau.

iii. Với bất kì một nhóm 2 người nào trong một phòng mà không quen biếtlẫn nhau, tồn tại đúng một người trong cùng phòng đó quen biết cả haingười này.

1. Chứng minh rằng trong mỗi phòng, mỗi người có số người quen bằngnhau.

2. Tìm số phòng lớn nhất.

Bài 7. Cho tập hợp A = {1, 2, . . . , 2n} với n nguyên dương. Một hoán vị cácphần tử của A được gọi là đẹp nếu như có ít nhất hai phần tử hơn kém nhau nđơn vị đứng cạnh nhau. Chứng minh rằng số hoán vị đẹp nhiều hơn số hoán vịkhông đẹp

Bài 8. Cho n là một số nguyên dương và S là tập hợp các điểm (x, y) trên mặtphẳng với x, y không âm và x + y < n. Các điểm trong S được tô màu đỏ hoặcxanh theo qui luật, nếu (x, y) là màu đỏ thì (x′, y′) được tô màu đỏ với x ≤ x′ vày ≤ y′. Đặt A là số cách chọn n điểm xanh mà hoành độ x của nó khác nhau và

22 ĐỀ BÀI

đặt B là số cách chọn n điểm xanh mà tung độ y của chúng khác nhau. Chứngminh rằng A = B

Bài 9. Cho một dãy n tấm bìa đặt sấp ở trên bàn được đánh số từ 1 tới n. Mỗilần cho phép thay đổi trạng thái của k tấm bìa liên tiếp từ sấp thành ngửa vàngược lại

1. Chứng minh rằng có thể chuyển hết tấm bìa từ sấp sang ngửa khi và chỉ

khi m... k

2. Nếu n không chia hết cho k, tìm số bìa tối đa có thể chuyển sang ngửa.

Bài 10. Trong một bảng vuông n×n, ta đặt những chiếc đèn lên các ô của bảng,mỗi ô một đèn. Ở mỗi lần thay đổi, ta được phép chọn một đèn làm gốc và thayđổi trạng thái của đèn đó và tất cả các đèn khác cùng hàng cùng cột với nó từtắt sang bật và ngược lại. Với trạng thái ban đầu là bất kì, ta có thể đưa tất cảđèn về trạng thái bật được hay không với

1. n = 6.

2. n = 2017.

Bài 11. Cho một bảng 5× 5 được tô trắng đen xen kẽ, các ô ở góc được tô đen.Trên mỗi ô đen có các đồng xu đen và trên các ô trắng có các đồng xu trắng.Các đồng xu có thể di chuyển đến các ô bên cạnh. A và B cùng chơi một trò chơinhư sau: Đầu tiên, A khởi động trò chơi bằng cách lấy một đồng xu đen ra khỏibảng rồi di chuyển một đồng xu trắng vào ô trống. Sau đó, B di chuyển mộtđồng xu đen vào ô trống. Các lượt sau đó, A sẽ di chuyển một đồng xu trắngvào ô trống và B di chuyển đồng xu đen vào ô trống. Trò chơi kết thúc khi mộttrong hai người không thể di chuyển được theo luật trên, và người còn lại làngười chiến thắng. Hỏi có chiến thuật thắng hay không, nếu có hãy chỉ ra ai làngười thắng?

Bài 12. Cho lưới tam giác đều như hình vẽ, trong đó mỗi cạnh có chứa n điểm(không tính hai đầu mút của cạnh), các đoạn thẳng song song với cạnh tamgiác lớn được nối với nhau. Đếm số tam giác đều có đỉnh là các điểm trong lướiđã cho

1.5 Tổ hợp 23

B C

A

Bài 13. Cho 2018 bóng đèn đang ở trạng thái sáng được xếp liên tiếp nhau trênmột đường thẳng. Hai người cùng chơi trò chơi như sau: Ở mỗi lượt chơi củamình, mỗi người sẽ chọn một bóng đèn sáng, sau đó đổi trạng thái của bóngđèn đó cùng với 4 bóng đèn phía sau nó.

1. Chứng minh rằng trò chơi sẽ dừng lại sau hữu hạn bước.

2. Ai có thể luôn là người chiến thắng? Hãy đưa ra chiến thuật thắng ấy.

Bài 14. Cho 33 điểm khác nhau nằm bên trong một hình vuông có cạnh là 4. Vẽ33 đường tròn nhận các điểm này làm tâm, có cùng bán kính

√2. Chứng minh

rằng tồn tại một đường tròn trong số chúng chứa ít nhất 3 điểm trong số 33điểm nói trên.

Bài 15. Có 2010 que diêm trên bàn. A và B cùng chơi trò chơi theo lượt nhưsau: Đến lượt của mình, họ sẽ lấy đi 1, 3, 4, 5 hoặc 7 que diêm. Người lấy quediêm cuối cùng sẽ chiến thắng. Nếu A chơi trước, hỏi người nào sẽ có chiếnthuật thắng?

24 ĐỀ BÀI

Chương 2

HƯỚNG DẪN GIẢI

2.1 Đại số

Cho a, b, c là ba số thực sao cho (a − b)(b − c)(c − a) 6= 0. Tìm giá trị nhỏnhất của biểu thức

P =(a2 + b2 + c2 + ab+ bc+ ca

) [ 1

(a− b)2+

1

(b− c)2+

1

(c− a)2

].

Bài 1

Lời giải. Ta có

a2 + b2 + c2 + ab+ bc+ ca =2

3(a+ b+ c)2 +

1

6

[(a− b)2 + (b− c)2 + (c− a)2

]≥ 1

6

[(a− b)2 + (b− c)2 + (c− a)2

]⇒ P ≥ 1

6

[(a− b)2 + (b− c)2 + (c− a)2

] [ 1

(a− b)2+

1

(b− c)2+

1

(c− a)2

]Giả sử a > b > c. Đặt x = a− b, y = b− c

P ≥ 1

6

[x2 + y2 + (x+ y)2

] [ 1

x2+

1

y2+

1

(x+ y)2

]

=1

3

(x2 + xy + y2

) [ 1

x2+

1

y2+

1

(x+ y)2

]

26 HƯỚNG DẪN GIẢI

Đặt t =x

y

P ≥ 1

3

(t2 + t+ 1

) [ 1

t2+

1

(t+ 1)2+ 1

]=

1

3

(t2 + t+ 1

) [2(t2 + t) + 1

(t2 + t)2+ 1

]Đặt z = t2 + t

P ≥ 1

3(z + 1)

[(z + 1)2

z2

]=

(z + 1)3

3z2≥

27.z

2.z

23z2

=9

4

Dấu "=" xảy ra⇔{b = 0a = −c .

Vậy giá trị nhỏ nhất của P là9

4.

Nhận xét.

1. Ta có thể tổng quát bài toán như sau: Cho x, y, z là các số thực đôi một phânbiệt. Với những giá trị nào của k thì biểu thức:

P =[x2 + y2 + z2 + k(xy + yz + zx)

] [ 1

(x− y)2+

1

(y − z)2+

1

(z − x)2

]đạt giá trị nhỏ nhất và hãy tìm giá trị nhỏ nhất đó.

Chứng minh tương tự như lời giải.

2. Một số bài toán tương tự:

(a) (VMO 2008) Cho x, y, z là các số thực không âm đôi một phân biệt.Chứng minh rằng:

(xy + yz + zx)

[1

(x− y)2+

1

(y − z)2+

1

(z − x)2

]≥ 4.

(b) (Đào Hải Long) Cho x, y, z là các số thực đôi một phân biệt. Chứngminh rằng:

(x2 + y2 + z2)

[1

(x− y)2+

1

(y − z)2+

1

(z − x)2

]≥ 9

2.

2.1 Đại số 27

Cho a, b, c là các số dương thoả mãn abc = 1. Chứng minh rằng

a√a+ 1

+b√b+ 1

+c√c+ 1

≥ 3√

2

2.

Bài 2

Lời giải. Xét f(x) =x√x+ 1

− 3√

2

8lnx−

√2

2với x > 0.

Ta có: f ′(x) =x+ 2

2(x+ 1)32

− 3√

2

8x. Giải phương trình f ′(x) = 0 trên (0,+∞) ta

được nghiệm duy nhất x = 1.

x

f ′(x)

f(x)

0 1 +∞

− 0 +

+∞+∞

00

+∞+∞

Từ bảng biến thiên ta được f(x) ≥ 0 với x > 0. Suy ra

f(a) + f(b) + f(c) ≥ 0

⇔ a√a+ 1

+b√b+ 1

+c√c+ 1

− 3√

2

8x(ln a+ ln b+ ln c)− 3

√2

2≥ 0.

Do abc = 1 nên ln a+ ln b+ ln c = 0. Từ đó ta được điều phải chứng minh.

Nhận xét.

1. Công thức lnxy = lnx+ ln y giúp ta chuyển điều kiện tích thành tổng để ápdụng phương pháp tiếp tuyến.

2. Một số bài toán tương tự:

(a) Cho a, b, c > 0 thoả mãn abc = 1. Chứng minh rằng√a2 + 1 +

√b2 + 1 +

√c2 + 1 ≤

√2(a+ b+ c).

28 HƯỚNG DẪN GIẢI

(b) (Moldova TST 2011) Cho các số dương x1, x2, . . . , xn thoả mãn x1x2 . . . xn =1. Chứng minh rằng

1

x1 (x1 + 1)+

1

x2 (x2 + 1)+ . . .+

1

xn (xn + 1)≥ n

2.

1. Cho a, b, c là ba số thực. Đặt s = a2 + bc − ab, r = b2 + ca − bc, t =c2 + ab − ca. Chứng minh rằng sr + ts + rt = a3b + b3c + c3a. Từ đósuy ra (a2 + b2 + c2)

2 ≥ 3 (a3b+ b3c+ c3a).

2. Cho x, y, z là các số thực dương thoả mãn điều kiện xy+ yz+ zx = 1.Chứng minh rằng

3−√

3 +x2

y+y2

z+z2

x≥ (x+ y + z)2.

Bài 3

Lời giải.

1. Bằng tính toán trực tiếp ta sẽ kiểm tra được sr+ ts+ rt = a3b+ b3c+ c3a,sau đó áp dụng bất đẳng thức: (r + t+ s)2 ≥ 3(sr + ts+ rt) ∀r, s, t ∈ R

2. Áp dụng bất đẳng thức ở câu 1 kết hợp với bất đẳng thức Cauchy Schwarzta được(x2

y+y2

z+z2

x

)2

≥ 3

(x3√yz

+y3√zx

+z3√xy

)≥ 3

(2x3

y + z+

2y3

z + x+

2z3

x+ y

)

≥ 3(x2 + y2 + z2)2

xy + yz + zx= 3(x2 + y2 + z2)2

⇒ x2

y+y2

z+z2

x≥√

3(x2 + y2 + z2) =√

3[(x+ y + z)2 − 2

].

Ta quy về chứng minh√

3[(x+ y + z)2 − 2

]≥ (x+ y + z)2 − 3 +

√3

⇔(√

3− 1) [

(x+ y + z)2 − 3]≥ 0

(luôn đúng theo bất đẳng thức Cauchy-Schwarz).

Từ đây ta được điều phải chứng minh.

2.1 Đại số 29

Nhận xét.

1. Bất đẳng thức ở câu a còn được gọi là bất đẳng thức Vasc, là một bổ đề mạnhhay được sử dụng trong chứng minh bất đẳng thức.

2. Bất đẳng thức ở câu b chính là đề chọn đội tuyển Iran năm 2010.

3. Ngoài ra ta có thể tổng quát bất đẳng thức ở câu 1 như sau: Xét bất đẳngthức sau với các biến thực a, b, c:

m∑cyc

a4 + n∑cyc

a2b2 + p∑cyc

a3b+ g∑cyc

ab3 − (m+ n+ p+ g)∑cyc

a2bc ≥ 0.

Khi đó bất đẳng thức này đúng nếu{m > 03m(m+ n) ≥ p2 + pg + g2

.

Chứng minh: xem trong [1].

Cho a, b, c là ba số thực dương. Xét bất đẳng thức(a2 + bc

b+ c

)n+

(b2 + ca

c+ a

)n+

(c2 + ab

a+ b

)n≥ an + bn + cn (2.1)

1. Chứng minh minh bất đẳng thức (2.1) với n = 1.

2. Với n = 2 thì bất đẳng thức (2.1) còn đúng không? Nếu có, hãy chứngminh. Nếu không, hãy chỉ ra phản ví dụ.

Bài 4

Lời giải.

1. Với n = 1, bất đẳng thức (2.1) trở thành

a2 + bc

b+ c+b2 + ca

c+ a+c2 + ab

a+ b≥ a+ b+ c

⇔ (a+ b)(a+ c)

b+ c+

(b+ a)(b+ c)

c+ a+

(c+ a)(c+ b)

a+ b≥ 2(a+ b+ c)

30 HƯỚNG DẪN GIẢI

Áp dụng bất đẳng thức AM-GM ta được

(a+ b)(a+ c)

b+ c+

(b+ a)(b+ c)

c+ a≥ 2(a+ b)

(b+ a)(b+ c)

c+ a+

(c+ a)(c+ b)

a+ b≥ 2(b+ c)

(c+ a)(c+ b)

a+ b+

(a+ b)(a+ c)

b+ c≥ 2(c+ a)

Cộng lại theo vế ta được đpcm.

2. Với n = 2, bất đẳng thức (2.1) trở thành∑sym

(a+ b)2(a+ c)2

(b+ c)2≥ 2

∑sym

a(a+ b)(a+ c)

b+ c

⇔∑sym

(a+ b)2(a+ c)2

(b+ c)2+ 2

∑sym

(a+ b)(a+ c)

≥ 2∑sym

(a+ b+ c)(a+ b)(a+ c)

b+ c(2.2)

Đặt x = b+ c, y = c+ a, z = a+ b. Khi đó (2.2) tương đương với

y2z2

x2+z2x2

y2+x2y2

z2+ 2(xy + yz + zx) ≥ (x+ y + z)

(yz

x+zx

y+xy

z

)

⇔ y2z2

x2+z2x2

y2+x2y2

z2+xy+yz+ zx ≥ xy(x+ y)

z+yz(y + z)

x+zx(z + x)

y

⇔ x4y4 + y4z4 + z4x4 + x2y2z2(xy + yz + zx)

≥ x3y3(zx+ yz) + y3z3(xy + zx) + z3x3(yz + xy) (2.3)

Đặt xy = m, yz = n, zx = p. Khi đó (2.3) tương đương với

m4 + n4 + p4 +mnp(m+ n+ p) ≥ m3(n+ p) + n3(m+ p) + p3(m+ n).

Đây là bất đẳng thức Schur bậc 4 nên ta có điều phải chứng minh.

2.1 Đại số 31

Nhận xét.

1. Với n = 2 thì (2.1) còn đúng với a, b, c ∈ R thoả (a + b)(b + c)(c + a) 6= 0.Thật vậy, dựa vô chứng minh ở trên, ta chỉ cần chứng minh bất đẳng thứcSchur bậc 4 đúng với mọi a, b, c ∈ R bằng cách đặt α = m + n − p, β =

m+ p− n, γ = n+ p−m thì m =α + β

2, n =

α + γ

2, p =

β + γ

2. Khi đó

(2.3) tương đương

α2 (β − γ)2 + β2 (γ − α)2 + γ2 (α− β)2 ≥ 0 (?)

2. Ta có thể chứng minh được (2.1) còn đúng với n là số thực dương (xem trong[1]).

Cho a, b, c là các số thực dương thoả mãn điều kiện a + b + c = 3. Chứngminh rằng

9

abc≥ 2

(a

c+c

b+b

a

)+ 3. (2.4)

Bài 5

Lời giải. Không mất tính tổng quát giả sử a = max{a, b, c}. Bất đẳng thức (2.4)tương đương

9 ≥ 2(a2b+ b2c+ c2a

)+ 3abc⇔ 27 ≥ 6

(a2b+ b2c+ c2a

)+ 9abc

⇔ (a+ b+ c)3 ≥ 6(a2b+ b2c+ c2a

)+ 9abc

⇔ a3 + b3 + c3 − 3abc ≥ 3(a2b+ b2c+ c2a− ab2 − bc2 − ca2

)⇔ (a+ b+ c)

[(a− b)2 + (b− c)2 + (a− c)2

]≥ 6(a− b)(b− c)(a− c) (4)

Ta có(a− b)(b− c) ≤ 1

4(a− b+ b− c)2 =

1

4(a− c)2

⇒ (a− c)(a− b)(b− c) ≤ 1

4(a− c)3

⇒ 6(a− c)(a− b)(b− c) ≤ 3

2(a− c)3

và(a− b)2 + (b− c)2 ≥ 1

2(a− b+ b− c)2 =

1

2(a− c)2

32 HƯỚNG DẪN GIẢI

⇒ (a+ b+ c)[(a− b)2 + (b− c)2 + (a− c)2

]≥ 3

2(a+ b+ c)(a− c)2

Ta quy về chứng minh

3

2(a+ b+ c)(a− c)2 ≥ 3

2(a− c)3

⇔ (a− c)2(b+ 2c) ≥ 0

(luôn đúng với mọi a, b, c thoả điều kiện đề bài).

Vậy ta có điều phải chứng minh.

Tìm số thực k nhỏ nhất sao cho bất đẳng thức

xyz + 2 + k[(x− 1)2 + (y − 1)2 + (z − 1)2

]≥ x+ y + z (2.5)

đúng với mọi x, y, z không âm.

Bài 6

Lời giải. Do (2.5) đúng với mọi bộ (x, y, z) không âm nên cũng đúng với (0, t+1, t+ 1), khi đó (2.5) trở thành

k ≥ 2t

1 + 2t2,∀t ≥ 1⇒ k ≥ min

[1;+∞)

2t

1 + 2t2⇒ k ≥ 1√

2.

Tiếp theo ta sẽ chứng minh (5) đúng với k =1√2

. Thật vậy, với k =1√2

thì (2.5)

trở thành

xyz + 2 +1√2

[(x− 1)2 + (y − 1)2 + (z − 1)2

]≥ x+ y + z

Không mất tính tổng quát ta có thể giả sử (y−1)(z−1) ≥ 0, suy ra yz+1 ≥ y+zhay xyz + x ≥ x(y + z). Từ đây ta quy về chứng minh

(x− 1)2 + (y − 1)2 + (z − 1)2 ≥√

2(y + z − 2)(1− x)

⇔ 1

2(x− y)2 +

[x+ y − 2√

2− (1− z)

]2≥ 0.

Nên ta có điều phải chứng minh.

2.1 Đại số 33

Nhận xét.

1. Do [(x− 1)(y − 1)] · [(y − 1)(z − 1)] · [(z − 1)(x− 1)] = (x− 1)2(y− 1)2(z−1)2 ≥ 0 nên trong 3 số (x− 1)(y − 1); (y − 1)(z − 1); (z − 1)(x− 1) phải cóít nhất một số không âm, do vậy ta có thể giả sử (y − 1)(z − 1) ≥ 0.

2. Với k = 2 ta được bài toán của thầy Trần Nam Dũng trong cuộc thi HelloIMO 2007: Cho x, y, z là các số thực dương. Chứng minh rằng ta luôn có bấtđẳng thức

xyz + 2(x2 + y2 + z2) + 8 ≥ 5(x+ y + z).

Cho a, b, c là ba cạnh của một tam giác, chứng minh bất đẳng thức sau:

a2b(a− b) + b2c(b− c) + c2a(c− a) ≥ 0.

Bài 7

Lời giải. Do a, b, c là ba cạnh của một tam giác nên tồn tại các số dương x, y, zsao cho a = y + z, b = z + x, c = x + y. Khi đó bất đẳng thức cần chứng minhtrở trành

xy3 + yz3 + zx3 ≥ xyz(x+ y + z)⇔ x2

y+y2

z+z2

x≥ x+ y + z

⇔ (x− y)2

y+

(y − z)2

z+

(z − x)2

x≥ 0.

Vậy ta có điều phải chứng minh.

Nhận xét.

1. Phép thế trong lời giải được gọi là phép thế Ravi, thường được sử trong cácbài toán chứng minh bất đẳng thức có điều kiện "3 cạnh tam giác". Hình vẽsau chỉ ra sự tồn tại của các số x, y, z.

34 HƯỚNG DẪN GIẢI

y z

z

x

x

y

c

a

b

2. Bài tập tương tự:

(a) (Moldova TST 2006): Cho a, b, c là độ dài ba cạnh của một tam giác.Chứng minh rằng

a2(b

c− 1

)+ b2

( ca− 1)

+ c2(ab− 1)≥ 0.

(b) (IMO 1961): Cho a, b, c là ba cạnh của một tam giác với diện tích S.Chứng minh rằng

a2 + b2 + c2 ≥ 4√

3S.

Cho a, b, c là các số thực không âm thoả mãn điều kiện a+ b+ c = 3. Chứngminh rằng

a

b3 + 16+

b

c3 + 16+

c

a3 + 16≥ 1

6.

Bài 8

Lời giải. Trước hết xin phát biểu không chứng minh một BĐT quen thuộc.

Bổ đề 1. Với x, y, z là các số thực không âm, ta có

xy2 + yz2 + zx2 + xyz ≤ 4

27(x+ y + z)3.

Chứng minh BĐT trên rất dễ dàng xin nhường lại cho bạn đọc.

2.1 Đại số 35

Quay lại bài toán. Ta có

3

16−(∑ a

b3 + 16

)=∑(

a

16− a

b3 + 16

)=∑ ab3

16(b3 + 16)

Do đó bất đẳng thức cần chứng minh tương tương

ab3

b3 + 16+

bc3

c3 + 16+

ca3

a3 + 16≤ 1

3.

Mặt khác áp dụng bất đẳng AM-GM ta được

ab3

b3 + 16=

ab3

b3 + 8 + 8≤ ab3

33√

8.8.b3=ab2

12.

Áp dụng tương tự ta sẽ được

ab3

b3 + 16+

bc3

c3 + 16+

ca3

a3 + 16≤ ab2 + bc2 + ca2

12.

Tới đấy áp dụng Bổ đề 1 ta được

ab2 + bc2 + ca2 ≤ ab2 + bc2 + ca2 + abc ≤ 4

27(a+ b+ c)3 = 4.

Từ đó ta được điều phải chứng minh. Đẳng thức xảy ra khi (a, b, c) là hoán vịcủa (0, 1, 2).

Nhận xét.

1. Chuyển bất đẳng thức cần chứng minh từ dạng "≥" sang "≤" được gọi là kỹthuật AM-GM ngược dấu.

2. Bổ đề 1 là một bổ đề quen thuộc hay được sử dụng.

3. Bài tập tương tự (USA National Mathematical Olympiad 2017): Cho a, b, c, dlà các số thực không âm thoả mãn a + b + c + d = 4. Tìm giá trị nhỏ nhấtcủa

a

b3 + 4+

b

c3 + 4+

c

d3 + 4+

d

a3 + 4.

36 HƯỚNG DẪN GIẢI

Chứng minh rằng với mọi số thực a, b, c, ta đều có

(b+c−a)2(c+a−b)2(a+b−c)2 ≥ (b2+c2−a2)(c2+a2−b2)(a2+b2−c2). (2.6)

Bài 9

Lời giải. Nếu a2 + b2 + c2 = 0 thì ta sẽ được a = b = c = 0. Khi đó bất đẳng thứcluôn đúng nên ta có thể giả sử a2 + b2 + c2 > 0. Khi đó (2.6) tương đương

(a2+b2+c2)(b+c−a)2(c+a−b)2(a+b−c)2 ≥ (a2+b2+c2)(b2+c2−a2)(c2+a2−b2)(a2+b2−c2).

Mà3(a2 + b2 + c2) ≥ (a+ b+ c)2

nên ta quy về chứng minh

(a+ b+ c)2(b+ c− a)2(c+ a− b)2(a+ b− c)2

≥ 3(a2 + b2 + c2)(b2 + c2 − a2)(c2 + a2 − b2)(a2 + b2 − c2) (2.7)

Mặt khác ta có

(a2+b2+c2)(b2+c2−a2)(c2+a2−b2)(a2+b2−c2) = 2(a4b4 + b4c4 + c4a4

)−(a8 + b8 + c8

)(a+ b+ c)(b+ c−a)(c+a− b)(a+ b− c) = 2

(a2b2 + b2c2 + c2a2

)−(a4 + b4 + c4

)Do đó, (2.7) tương đương[2(a2b2 + b2c2 + c2a2

)−(a4 + b4 + c4

)]2 ≥ 3[2(a4b4 + b4c4 + c4a4

)−(a8 + b8 + c8

)]⇔ a8 + b8 + c8 + 2a2b2c2(a2 + b2 + c2) ≥

(a2b2 + b2c2 + c2a2

) (a2 + b2 + c2

)Đây là bất đẳng thức Schur bậc 4 nên từ đây ta có điều phải chứng minh.

Nhận xét.

1. Đẳng thức (x+y+z)(y+z−x)(z+x−y)(x+y−z) = 2∑x2y2−

∑x4 thường

được sử dụng trong các bài toán có biểu thức (y+z−x)(z+x−y)(x+y−z),được gọi là đẳng thức Heron.

2. Bài tập tương tự: Cho a, b, c > 0. Chứng minh rằng

abc(ab+ bc+ ca) ≥ (a2 + b2 + c2)(a+ b− c)(c+ a− b)(b+ c− a).

2.1 Đại số 37

Cho a, b, c, d là các số thực dương. Chứng minh rằng√ab+ ac+ ad+ bc+ bd+ cd

6≥ 3

√abc+ bcd+ cda+ dab

4. (2.8)

Bài 10

Lời giải. Đặt S2 = ab + ac + ad + bc + bd + cd, S3 = abc + bcd + cda + dab. Xétđa thức

P (x) = (x− a)(x− b)(x− c)(x− d) = x4− (a+ b+ c+ d)x3 +S2x2−S3x+ abcd.

Ta có P (a) = P (b) = P (c) = P (d) nên theo định lý Rolle, tồn tại α, β, γ dươngsao cho P ′(α) = P ′(β) = P ′(γ) = 0 nên

P ′(x) = 4 (x− α) (x− β) (x− γ) = 4x3−4 (α + β + γ)x2+4 (αβ + βγ + γα)x−4αβγ

(do P ′(x) có hệ số của bậc cao nhất bằng 4).

Mặt khác ta lại có P ′(x) = 4x3− 3(a+ b+ c+d)x2 + 2S2x−S3 nên từ đó ta được

4x3−4 (α + β + γ)x2+4 (αβ + βγ + γα)x−4αβγ = 4x3−3(a+b+c+d)x2+2S2x−S3.

Đồng nhất hệ số hai vế, suy ra

S2 = 2 (αβ + βγ + γα)

S3 = 4αβγ.

Khi đó (2.8) trở thành √αβ + βγ + γα

3≥ 3√αβγ.

Điều này luôn đúng theo bất đẳng thức AM-GM nên ta được điều phải chứngminh.

Nhận xét.

1. Bài tập tương tự (VMO 1996): Cho các số thực không âm a, b, c, d thoả mãn

2(ab+ ac+ ad+ bc+ bd+ cd) + abc+ bcd+ cda+ dab = 16.

Chứng minh rằng

a+ b+ c+ d ≥ 2

3(ab+ bc+ ca+ ad+ ac+ bd).

38 HƯỚNG DẪN GIẢI

2. Ta có bài toán tổng quát (Bất đẳng thức Maclaurin): Cho số nguyên n ≥ 2

và các số thực không âm x1, x2, x3, . . . , xn. Đặt Sk =∑

1≤i1<i2<...<ik≤n

k∏j=1

xij

và dk =

(SkCkn

) 1k

, k = 1, n. Chứng minh rằng

d1 ≥ d2 ≥ d3 ≥ . . . ≥ dn.

Chứng minh: xem trong [7].

Tìm tất cả các đa thức P (x) ∈ R[x] thoả mãn

P (x)P (2x2) = P (x3 + x). (2.9)

Bài 11

Lời giải. Xét các trường hợp:

TH1: P là đa thức hằng. Trong trường hợp này ta tìm được các đa thức thoảmãn yêu cầu bài toán là

P (x) = 0; P (x) = 1.

TH2: P không là đa thức hằng. Từ (2.9) dễ thấy P (0) ∈ {0, 1}.

• Nếu P (0) = 0, tồn tại đa thức G(x) ∈ R[x] và k ∈ Z+ thoả mãn

P (x) = xkG(x) và G(0) 6= 0.

Khi đó từ (2.9) ta được

xkG(x)(2x2)kG(2x2)

=(x3 + x

)kG(x3 + x).

⇔(2x2)kG(x)G(2x2) = (x2 + 1)kG(x3 + x).

Từ đây suy ra G(0) = 0, mâu thuẫn.

• Nếu P (0) = 1, đặt degP = m với m ∈ Z+. Từ (2.9) ta cũng được

hệ số cao nhất của P là1

2m. Gọi x1, x2, . . . , xm là n nghiệm phức của

2.1 Đại số 39

đa thức P . Theo định lý Viet ta có |x1x2 . . . xm| = 2m. Suy ra tồn tạim0 ∈ {1, 2, . . . ,m} sao cho |xm0| ≥ 2.Ta sẽ chứng minh nếu số phức z có |z| ≥ 2 thì |z2 + 1| > 1. Thật vậy,ta có |z2 + 1| ≥ |z2| − 1 = |z|2 − 1 ≥ 4− 1 > 1.Xét dãy số

(zn) :

{z0 = xm0

zn = z3n−1 + zn−1 ∀n ∈ Z+

Áp dụng nhận xét trên ta sẽ chứng được bằng quy nạp rằng

|zn| > |zn−1| ≥ 2 ∀n ∈ Z+.

Mặt khác ta lại có zn là nghiệm của P (x) với mọi n ∈ N. Suy ra P (x)có vô số nghiệm, mâu thuẫn.

vậy các đa thức cần tìm thoả mãn đề bài là P (x) = 0; P (x) = 1.

Nhận xét.

1. Ta có bất đẳng thức về modun của số phức như sau: Với z1, z2 ∈ C thì|z1|+ |z2| ≥ |z1 + z2| ≥ |z1| − |z2|.

2. Ta có định lý quan trọng sau: Giả sử f, g, h là các đa thức với hệ số thực thoảmãn điều kiện deg(f) + deg(g) = deg(h) và thoả mãn một trong hai điềukiện sau:

(i) deg(f) 6= deg(g)

(ii) deg(f) = deg(g) và f ∗ + g∗ 6= 0, trong đó f ∗, g∗ là hệ số cao nhất củacác đa thức f và g tương ứng.

Khi đó với mọi số nguyên dương n tồn tại nhiều nhất một đa thức P(x) cóbậc n và thoả mãn phương trình P (f(x))P (g(x)) = P (h(x)).

Chứng minh: xem trong [4].

Áp dụng định lý này ta có một số bài toán tương tự:

(a) (VMO 1990) Tìm tất cả các đa thức P ∈ R[x] thoả mãn P (x)P (2x2) =P (2x3 + x).

(b) Tìm tất cả các đa thức P ∈ C[x] thoả mãn P (x)P (−x) = P (x2).

40 HƯỚNG DẪN GIẢI

Cho đa thức P (x) ∈ R[x] và P (x) ≥ 0 với mọi x ∈ R. Chứng minh rằng đathức P (x) có thể biểu diễn dưới dạng

P (x) = (A(x))2 + (B(x))2

trong đó A(x), B(x) cũng là các đa thức có hệ số thực.

Bài 12

Lời giải. Ta bỏ qua trường hợp đơn giản là P (x) ≡ C (với C là hằng số). DoP (x) ≥ 0 với mọi x ∈ R nên đa thức P (x) có bậc bằng 2n và có thể phân tíchđược dưới dạng tích các nhân tử bậc hai không âm, nghĩa là:

P (x) =n∏i=1

[(aix+ bi)

2 + c2i]

trong đó ai, bi, ci ∈ R với mọi i = 1, 2, . . . , n.

Từ hằng đẳng thức(p2 + q2

) (r2 + s2

)= (pr + qs)2 + (ps− qr)2

ta được nhận xét sau: Tích của hai biểu thức có dạng (u(x))2 + (v(x))2 cũng làmột biểu thức có dạng đó. Từ đó bằng quy nạp ta sẽ thu được điều phải chứngminh.

Cho đa thức

f(x) = xn + an−2xn−2 + an−3x

n−3 + . . .+ a1x+ a0 ∈ R[x].

Chứng minh rằng tồn tại i ∈ {1, 2, . . . , n} sao cho |f(i)| ≥ n!

Cin

.

Bài 13

Lời giải. Đặt g(x) = f(x)− xn. Áp dụng công thức nội suy Lagrange ta được

g(x) =n∑i=1

g(i) ·n∏j=1j 6=i

x− ji− j

=n∑i=1

g(i) · 1n∏j=1j 6=i

(i− j)·

n∏j=1j 6=i

(x− j)

.

2.1 Đại số 41

Ta cón∏j=1j 6=i

(i− j) = (i− 1)(i− 2) . . . [i− (i− 1)][i− (i+ 1)] . . . (i− n)

= (−1)n−i(i− 1)!(n− i)! =(−1)n−in!

iCin

.

Nên

g(x) =n∑i=1

(−1)n−i · i · g(i) · Cin

n!·

n∏j=1j 6=i

(x− j)

So sánh hệ số của xn−1 ở 2 vế ta được

n∑i=1

(−1)n−i · i · g(i) · Cin

n!= 0⇔

n∑i=1

(−1)n−i · i · (f(i)− in) · Cin

n!= 0

⇔n∑i=1

Cin(−1)n−iin+1 =

n∑i=1

iCin(−1)n−if(i).

Giả sử |f(i)| < n!

Cin

∀i = 1, 2, . . . , n thì suy ra∣∣∣∣∣n∑i=1

iCin(−1)n−if(i)

∣∣∣∣∣ =n∑i=1

iCin|f(i)| <

(n∑i=1

i

)· n! =

n(n+ 1)!

2

∣∣∣∣∣n∑i=1

Cin(−1)n−iin+1

∣∣∣∣∣ < n(n+ 1)!

2.

Mặt khác ta lại cón∑i=1

Cin(−1)n−iin+1 =

n(n+ 1)!

2.

Thật vậy xét h(x) = xn − (x − 1)(x − 2) . . . (x − n). Áp dụng công thức nội suyLagrange ta được

h(x) =n∑i=1

h(i) ·n∏j=1j 6=i

x− ji− j

=n∑i=1

(−1)n−i · in+1 · Ci

n

n!·

n∏j=1j 6=i

(x− j)

42 HƯỚNG DẪN GIẢI

So sánh hệ số của xn−1 ở 2 vế ta được

1 + 2 + . . .+ n =

n∑i=1

Cin(−1)n−iin+1

n!

⇔ n(n+ 1)

2=

n∑i=1

Cin(−1)n−iin+1

n!⇔ n(n+ 1)!

2=

n∑i=1

Cin(−1)n−iin+1.

Vậy điều giả sử là sai, từ đó ta được điều phải chứng minh.

Nhận xét. Một trong những ứng dụng của công thức nội suy Lagrange là tínhtổng và chứng minh bất đẳng thức liên quan đến đa thức.Một số bài toán tương tự:

1. Cho số nguyên dương n. Tínhn∑k=0

(−1)k · Ckn · kn và

n∑k=0

(−1)k · Ckn · kn+2.

2. (VMO 1977) Cho n + 1 số nguyên đôi một phân biệt x0, x1, . . . , xn. Xét cácđa thức P (x) = xn + an−1x

n−1 + . . . + a1x + a0 ∈ R[x]. Chứng minh rằng

maxi∈0,n|P (xi)| ≥

n!

2n.

Cho số nguyên n ≥ 3 và P (x) ∈ R[x] thoả mãn

P (x) = xn + an−3xn−3 + an−4x

n−4 + . . .+ a1x+ a0.

Biết ít nhất một số trong các số a0, a1, . . . , an−3 khác 0. Chứng minh rằngP (x) không thể có toàn nghiệm thực.

Bài 14

Lời giải. Gọi x1, x2, . . . , xn là các nghiệm của P (x). Giả sử xi ∈ R với mọi i =1, 2, . . . , n, khi đó theo định lý Viet ta có

n∑i=1

xi = 0

∑1≤i<j≤n

xixj = 0⇔n∑i=1

x2i =

(n∑i=1

xi

)2

− 2∑

1≤i<j≤n

xixj = 0

2.1 Đại số 43

nên xi = 0 với mọi i = 1, 2, . . . , n, tức là P (x) chỉ có nghiệm thực duy nhất là0, dẫn tới a0 = a1 = . . . = an−3 (mâu thuẫn với giả thiết). Vậy điều giả sử là sainên ta có điều phải chứng minh.

Cho n là số nguyên dương chẵn. Một đa thức monic bậc n có n nghiệmthực (không nhất thiết phân biệt). Giả sử y là số thực dương thoả với mọisố thực t < y, ta có P (t) > 0. Chứng minh rằng n

√P (0)− n

√P (y) ≥ y.

Bài 15

Lời giải. Gọi x1, x2, . . . , xn là các nghiệm của P (x). Nếu tồn tại i ∈ {1, 2, . . . , n}sao cho xi < y thì P (xi) > 0 (mâu thuẫn vì xi là nghiệm của P (x)). Vậy ta có0 < y ≤ xi

(i = 1, n

).

Mặt khác ta có P (x) = (x− x1) (x− x2) . . . (x− xn) và n chẵn nên

P (0) = (−1)nx1x2 . . . xn = x1x2 . . . xn > 0 (do n là số chẵn)

P (y) = (y − x1) (y − x2) . . . (y − xn) = (−1)n (x1 − y) (x2 − y) . . . (xn − y)

= (x1 − y) (x2 − y) . . . (xn − y) ≥ 0.

Điều cần chứng minh trở thành[y + n

√(x1 − y) (x2 − y) . . . (xn − y)

]n≤ x1x2 . . . xn.

Áp dụng BĐT Holder ta được

x1x2 . . . xn = (y + x1 − y) (y + x2 − y) . . . (y + xn − y)

≥[

n√yn + n

√(x1 − y) (x2 − y) . . . (xn − y)

]n=[y + n

√(x1 − y) (x2 − y) . . . (xn − y)

]n.

Đây chính là điều phải chứng minh.

Tìm các đa thức f(x), g(x) hệ số nguyên thoả mãn

f (g(x)) = x2017 + 2018x+ 1. (2.10)

Bài 16

44 HƯỚNG DẪN GIẢI

Lời giải. Ta bỏ qua trường hợp đơn giản khi f hoặc g là các đa thức hằng. Gọim = deg f và n = deg g. Từ giả thiết ta được mn = 2017.

TH1: m > 1;n > 1. Trong (2.10), đạo hàm hai vế, ta được

g′(x)f ′ (g(x)) = 2017x2016 + 2018 (2.11)

Mặt khác ta có

2017 6 ... 2

2018... 2

2018 6 ... 22

nên theo tiêu chuẩn Eisenstein ta có 2017x2016+

2018 bất khả quy trên Z[x]. Rõ ràng điều này mâu thuẫn với (2.11).

TH2: m = 1;n = 2017. Đặt f(x) = ax+b và g(x) =2017∑i=0

cixi với a, b, c1, c2, . . . , c2017 ∈

Z và a, c2017 6= 0. Khi đó (2.10) trở thành

a

(2017∑i=0

cixi

)+ b = x2017 + 2018x+ 1.

Đồng nhất hệ số hai vế ta được:

ac2017 = 1; aci = 0(i = 2, 2016

); ac1 = 2018; ac0 + b = 1.

Từ đây ta sẽ tìm được các đa thức sau thoả mãn đề bài:

f(x) = x+ b; g(x) = x2017 + 2018x+ 1− b

f(x) = −x+ b; g(x) = −x2017 − 2018x+ b− 1.

TH3: n = 1,m = 2017. Đặt f(x) =2017∑i=0

kixi và g(x) = sx+t với s, t, k1, k2, . . . , k2017 ∈

Z và s, k2017 6= 0. Làm tương tự như TH2 ta sẽ được s = ±1.

• Nếu s = 1, ta cóg(x− t) = x

f(g(x− t)) = (x− t)2017 + 2018(x− t) + 1.

Suy ra

f(x) = (x− t)2017 + 2018(x− t) + 1; g(x) = x+ t.

2.1 Đại số 45

• Nếu s = −1, tương tự như trên, ta cũng tìm được

f(x) = −(x− t)2017 − 2018(x− t) + 1; g(x) = −x+ t.

Vậy tất cả các cặp đa thức f, g cần tìm là

f(x) = x+ b; g(x) = x2017 + 2018x+ 1− b

f(x) = −x+ b; g(x) = −x2017 − 2018x+ b− 1

f(x) = (x− t)2017 + 2018(x− t) + 1; g(x) = x+ t

f(x) = −(x− t)2017 − 2018(x− t) + 1; g(x) = −x+ t.

Cho đa thức hệ số nguyên P (x) = anxn+an−1x

n−1 + . . .+a1x+a0 (an 6= 0).

Giả sử tồn tại m sao cho m ≥ maxi∈0,n

{∣∣∣∣ aian∣∣∣∣} + 2 và P (m) là số nguyên tố.

Chứng minh rằng P (x) bất khả quy trên Z.

Bài 17

Lời giải. Giả sử tồn tại các đa thứcQ,H khác hằng thoả mãn P (x) = Q(x)H(x).Do P (m) là số nguyên tố nên |Q(m)| = 1 hoặc |H(m)| = 1. Giả sử |Q(m)| =1, gọi degQ = k và xi (i ∈ 1, k) là các nghiệm của Q(x). Khi đó Q(x) =

ak∏i=1

(x− xi) với a ∈ Z \ {0}, suy ra 1 = |Q(m)| = |a|k∏i=1

|m− xi|.

Do |a| ≥ 1 nên tồn tại j ∈ {1, 2, . . . , k} sao cho |m − xj| ≤ 1. Mà |m − xj| ≥|m| − |xj| nên suy ra |xj| ≥ m+ 1 > 1.

Mặt khác, do xj cũng là nghiệm của P (x) nênn∑i=0

aixij = 0, suy ra

|xj|n =

∣∣∣∣∣n−1∑i=0

aianxij

∣∣∣∣∣ ≤n−1∑i=0

∣∣∣∣ aian∣∣∣∣ |xj|i ≤M

(n−1∑i=0

|xj|i)

= M · |xj|n − 1

|xj| − 1<M |xj|n

|xj| − 1

⇒ |xj| < 1 +M ≤ 1 +m− 2 = m− 1

Tới đây ta được một điều mâu thuẫn nên điều giả sử ban đầu là sai, tức là P (x)bất khả quy trên Z.

46 HƯỚNG DẪN GIẢI

Nhận xét. Qua bài 17 và bài 18, ta sẽ phát biểu lại một số tiêu chuẩn để chứngminh đa thức bất khả quy

1. Tiêu chuẩn 1 (tiêu chuẩn Eisenstenin): Cho đa thức P (x) =n∑i=0

aixi (i = 0, n)

với ai ∈ Z. Khi đó nếu tồn tại số nguyên tố p thoả mãn đồng thời các điềukiện:

(i) Tất cả các hệ số trừ an chia hết cho p.

(ii) a0 không chia hết cho p2.

Khi đó P (x) bất khả quy trên Z.

2. Tiêu chuẩn 2 (tiêu chuẩn Eisenstenin suy rộng): Cho đa thức P (x) =n∑i=0

aixi (i =

0, n) với ai ∈ Z. Khi đó nếu tồn tại số nguyên tố p thoả mãn đồng thời cácđiều kiện:

(i) an không chia hết cho p.

(ii) a0 không chia hết cho p2.

(iii) a0, a1, . . . , an−k chia hết cho p

Khi đó nếu P (x) = H(x)G(x) thì một trong hai đa thức H(x), G(x) có bậcnhỏ hơn k.

3. Tiêu chuẩn 3 (tiêu chuẩn Perron): Cho đa thức P (x) =n∑i=0

aixi (i = 0, n) với

ai ∈ Z và a0 6= 0. Khi đó, nếu

|an−1| > |a0|+ |a1|+ . . .+ |an−2|+ |an|,

thì P (x) bất khả quy trên Z.

4. Tiêu chuẩn 4 (tiêu chuẩn đánh giá về nghiệm): Cho P ∈ Z[x] và m ∈ Z thoảmãn |P (m)| là 1 hoặc là một số nguyên tố sao cho P (x) không có nghiệmthuộc {z ∈ C : |z −m| ≤ 1}.Khi đó P (x) bất khả quy trên Z.

2.1 Đại số 47

Cho số nguyên dương n. Tìm tất cả đa thức P thoả mãn đồng thời hai điềukiện sau:

i. P (x) = anxn + an−1x

n−1 + . . . + a1x + a0 trong đó {a0, a1, . . . , an} ={0, 1, 2, . . . , n}.

ii. Đa thức P có n nghiệm thực phân biệt.

Bài 18

Lời giải. Từ giả thiết suy ra an 6= 0.

Gọi x1, x2, . . . , xn là n nghiệm thực phân biệt của đa thức P . Theo định lý Viete,ta có

ε1 =n∑i=1

xi = −an−1an

. . .

εn−1 =n∑i=1

n∏j=1j 6=i

xj = (−1)n−1a1an

εn =n∏i=1

xi = (−1)na0an

Do {a0, a1, . . . , an} = {0, 1, 2, . . . , n} nên P (x) > 0 với x > 0. Suy ra xi ≤ 0,∀i =1, n.

Nếu xi < 0,∀i = 1, n thì[εi > 0εi < 0

,∀i = 1, n, nên suy ra ai 6= 0,∀i = 0, n− 1

(mâu thuẫn với giả thiết).

Vậy đa thức P có một nghiệm bằng 0, giả sử xn = 0, từ đó ta cũng có a0 = 0.

• Với n = 1: ta tìm được đa thức duy nhất thoả mãn yêu cầu bài toán làP (x) = x.

• Với n = 2: ta tìm được tất cả đa thức thoả mãn yêu cầu bài toán là P (x) =x2 + 2x;P (x) = 2x2 + x.

• Với n = 3: ta tìm được tất cả đa thức thoả mãn yêu cầu bài toán là P (x) =x3 + 3x2 + 2x;P (x) = 2x3 + 3x2 + x.

• Với n ≥ 4

Đặt Q (x) = anxn−1 + an−1x

n−2 + . . . + a2x + a1 thì ta có Q(x) có n − 1nghiệm âm phân biệt x1, x2, . . . , xn−1 và P (x) = xQ(x).

48 HƯỚNG DẪN GIẢI

Đặt ti = −xi,∀i = 1, n− 1, suy ra

ti > 0, ∀i = 1, n− 1

n−1∑i=1

ti =an−1an

(2.12)

n−1∑i=1

n−1∏j=1j 6=i

tj =a2an

(2.13)

n−1∏i=1

ti =a1an

(2.14)

Từ (2.13) và (2.14) ta được

n−1∑i=1

1

ti=a2an

:a1an

=a2a1

(2.15)

Từ (2.12) và (2.15) ta được(n−1∑i=1

ti

)(n−1∑i=1

1

ti

)=an−1an· a2a1

(2.16)

Theo bất đẳng thức AM-GM ta được(n−1∑i=1

ti

)(n−1∑i=1

1

ti

)≥ (n− 1)2.

Do {a0, a1, . . . , an} = {0, 1, 2, . . . , n} nên

an−1an· a2a1≤ n (n− 1)

2

Từ đây ta suy ra n− 1 ≤ n

2, suy ra n ≤ 2 (mâu thuẫn vì n ≥ 4).

Kết luận:

• Nếu n = 1 thì tất cả đa thức thoả mãn yêu cầu bài toán là P (x) = x.

2.1 Đại số 49

• Nếu n = 2 thì tất cả đa thức thoả mãn yêu cầu bài toán là P (x) = x2 +2x;P (x) = 2x2 + x.

• Nếu n = 3 thì tất cả đa thức thoả mãn yêu cầu bài toán là P (x) = x3 +3x2 + 2x;P (x) = 2x3 + 3x2 + x.

• Nếu n ≥ 4 thì không tồn tại đa thức nào thoả mãn yêu cầu bài toán.

Tìm tất cả các đa thức hệ số nguyên f sao cho n|m ⇒ f(n)|f(m) với mọicác số nguyên dương m,n.

Bài 19

Lời giải. Gọi f ∈ Z[x] là đa thức thoả mãn điều kiện đề bài và a ∈ Z \ {0} là hệsố cao nhất của f . Đặt deg f = k (k ∈ N).

Với mỗi n nguyên, ta thấy f(n)|f(dn),∀d ∈ Z nên

f(n) |k∑i=0

Cik(−1)if((d− i)n) = k!ank.

Suy raf(n)|k!

(f(n)− ank

),∀n ∈ Z. (2.17)

Mà deg f(x) > deg[f(x)− axk

]nên tồn tại n0 ∈ N đủ lớn sao cho

|f(n)| >∣∣f(n)− ank

∣∣ ,∀n > n0. (2.18)

Từ (2.17) và (2.18) suy ra

f(n) = ank,∀n > n0.

Vậy tất cả các đa thức thoả mãn điều kiện đề bài là f(n) = ank (k ∈ N).

Nhận xét.

1. Chứng minh đẳng thứck∑i=0

Cik(−1)if((d− i)n) = ak!nk bằng công thức nội

suy Lagrange.

2. Ta có một bài toán "họ hàng" của bài toán trên như sau: Tìm tất cả các đathức f với hệ số nguyên sao cho f(n)|f(m)⇒ n|m.Chứng minh: xem trong [9].

50 HƯỚNG DẪN GIẢI

Cho đa thức monic P (x) bậc n > 1 (tức P có hệ số của bậc lớn nhất bằng1) có n nghiệm thực x1, x2, ...xn phân biệt khác 0. Chứng minh rằng:

1

x1P ′(x1)+

1

x2P ′(x2)+ ...+

1

xnP ′(xn)=

(−1)n+1

x1x2...xn.

Bài 20

Lời giải. Đặt Pi(x) là đa thức bậc n− 1 có dạng:

Pi(x) =∏j 6=i

(x− xj)

Khi đó ta có:Pi(xj) = 0, ∀j 6= i

P ′(x) =n∑i=1

Pi(x)

Ngoài ra ta có:P ′(xi) = Pi(xi).

Xét đa thức:

Q(x) =n∑i=1

Pi(x)

P ′(xi)− 1.

Vì các đa thức Pi có bậc là n− 1 nên Q cũng có bậc là n− 1. Thay các giá trị xivào ta có:

Q(xj) =n∑i=1

Pi(xj)

P ′(xj)− 1 =

Pj(xj)

P ′(xj)− 1 = 0

Vậy ta có Q có n nghiệm. Điều đó dẫn đến Q ≡ 0. Cuối cũng chỉ cần đống nhấthệ số sẽ có kết quả như mong muốn.

1. Đồng nhất hệ số bậc cao nhất ta có:

1

P ′(x1)+

1

P ′(x2)+ ...+

1

P ′(xn)= 0.

2.2 Hình học 51

2. Đồng nhất hệ số bậc tự do ta có:

(−1)n−1x2x3...xnP ′(x1)

+(−1)n−1x1x3...xn

P ′(x2)+ ...+

(−1)n−1x1...xn−1xnP ′(xn)

= 1.

Hay tương đương:

1

x1P ′(x1)+

1

x2P ′(x2)+ ...+

1

xnP ′(xn)=

(−1)n+1

x1x2...xn.

2.2 Hình học

Cho 4ABC. điểm P di chuyển trên cạnh BC. Q,R lần lượt là hai điểmđối xứng với P qua CA,AB. Lấy điểm M nằm trên đường tròn ngoại tiếp4AQR sao cho AM ‖ BC. Chứng minh đường thẳng PM đi qua một điểmcố định khi P di chuyển trên cạnh BC.

Bài 1

A

B CP

Q

R

M

N

O

G

Lời giải. Gọi giao điểm của hai đường thẳng BR và CQ là N . Biến đổi góc, tađược RNQ = 180− 2BAC ⇒ N thuộc đường tròn ngoại tiếp 4AQR.

52 HƯỚNG DẪN GIẢI

Gọi giao điểm thứ hai của hai đường tròn ngoại tiếp các tam giác NBC và NQRlà G. Dễ dàng chứng minh 4GBR ∼ 4GCQ (g.g), dẫn tới

GB

GC=RB

QC=BP

CP.

Điều này cho ta GP là phân giác trong góc BGC.

Gọi O là tâm ngoại tiếp 4ABC. Để ý rằng BGC = BNC = 180o − QAR =

180o − BOC ⇒ O thuộc đường tròn ngoại tiếp 4BGC ⇒ GO là phân giáctrong góc BGC ⇒ G,P,O thẳng hàng.

Gọi T là giao điểm của hai đường thẳng AG và QR. Bằng biến đổi góc, ta đượcAGR = RTG. Từ đây dẫn tới 4GQR ∼ 4GCB, mà GT,GP lần lượt là cácphân giác của góc RAQ, BGC ⇒ RTG = BPG⇒ AMG = BPG. Mà giả thiếtcho ta AM ‖ BC ⇒ G,P,M thẳng hàng, tức là G,P,O,M thẳng hàng. Vậy BMđi qua điểm O cố định.

Cho 4ABC có đường cao AH. Giả sử đường tròn đường kính BC tiếp xúcđường tròn nội tiếp 4ABC. Chứng minh rằng AH +BC = AB + AC.

Bài 2

Lời giải. Đặt ID = r(D ∈ BC) là bán kihs đường tròn nội tiếp 4ABC. Vìđường tròn nội tiếp 4ABC tiếp xúc trong đường tròn đường kính BC (bạnđọc hãy thử giải thích tại sao hai đường tròn đó không thể tiếp xúc ngoài) nên

IM =BC

2− r.

2.2 Hình học 53

B CH

A

I

D M

Lời giải. Đặt ID = r, BC = a, CA = b, AB = c, p =a+ b+ c

2. Từ đó, ta có

được ID =BC

2− r, MD =

|DB −DC|2

.

Ta có

IM2 = ID2 +DM2 ⇒(BC

2− r)2

= r2 +

(DB −DC

2

)2

⇔ BC2

4−BC · r =

(DB −DC)2

4

⇔ (DB +DC)2

4−BC · r =

(DB −DC)2

4

⇔ r =DB ·DCBC

=(p− b)(p− c)

a.

Lại có

AH2 =4S2

BC2=

4p(p− a)(p− b)(p− c)a2

=4.S

r(p− a)r

a= AH(p− a)⇔ AH = p− a.

54 HƯỚNG DẪN GIẢI

Vậy, ta có điều phải chứng minh.

Nhận xét. Các bài hình học có nội dung tính toán đôi lúc sẽ gây khó khăn cho cácbạn học sinh, vì nó đòi hỏi sự kết hợp nhuần nhuyễn giữa khả năng nhìn hình vàkĩ năng tính toán. Các cách tính toán thường dùng để xử lí các bài toán hình họcgồm có:

• Đặt hệ trục toạ độ.

• Tính toán dựa trên độ dài các cạnh.

• Dùng định lí sin, hệ thức cos, định lí Menelaus và định lí Ceva dạng lượnggiác.

Sau đây là một số bài tập làm thêm cho các bạn.

1. Xét một họ các tam giác cân có tính chất sau; Chúng có đáy nằm trên mộtđường thẳng d cố định, có đỉnh A thuôc đáy là một điểm cố định và có bánkính đường tòn nội tiếp bằng r không đổi. Chứng minh rằng cạnh bên khôngđi qua A của các tam giác này luôn tiếp xúc một đường tròn cố định.

2. Cho tam giác ABC có BD,CE là hai đường phân giác D ∈ AC,E ∈ AB>Giả sử ID = IE, chứng minh 4ABC cân tại A hoặc BAC = 60o.

3. Cho4ABC có 2BC = AB+AC. Gọi O, I lần lượt là tâm đường tròn ngoạitiếp và nội tiếp 4ABC. Chứng minh rằng 4AIO vuông tại I.

4. (VMO 2009) Cho 2 điểm cố định A,B và điểm C di động trên mặt phẳngsao cho ˆACB = a (0 < a < 180) không đổi cho trước. Hình chiếu của tâmđường tròn nội tiếp I của tam giác ABC xuống ba cạnh AB, BC, CA lầnlượt là D, E, F . AI và BI cắt EF lần lượt tại M,N .

(a) Chứng minh độ dài MN không đổi.

(b) Chứng minh đường tròn (DMN) luôn đi qua một điểm cố định.

5. (Trung Quốc MO 2008) Cho tam giác nhọn không cân ABC có I là tâmđường tròn nội tiếp. Gọi E là tiếp điểm của I trên cạnh BC, đoạn thẳng AEcắt (I) tại điểm thứ hai là D khác E. Trên đường thẳng AE lấy điểm F saocho CE = CF . Đường thẳng BD cắt đường thẳng CF tại K. Chứng minhrằng KF = CE.

2.2 Hình học 55

6. (IMO 2001) Cho tam giác ABC có A = 60o và AP,BQ lần lượt là các phângiác trong góc A, B. Biết rằng AB + BP = AQ + QB, tính các góc còn lạicủa tam giác ABC.

Cho 4ABC có hai đường cao BE và CF . Đường tròn bàng tiếp góc A là(Ia). Hai tiếp tuyến chung trong của (AEF ) và (Ia) cắt BC tại P và Q.Chứng minh rằng BP = CQ.

Bài 3

A

B

C

E

F

J

G

H

M N

P

K

D

Lời giải. Bài toán này đã xuất hiện trong đề thi Olympiad hình học toàn Liênbang Nga mở rộng (Sharygin Olympiad 2017). Gọi J là tâm đường tròn ngoại

56 HƯỚNG DẪN GIẢI

tiếp4AEF , S là giao của hai tiếp tuyến chung trong, (I), (K) lần lượt là đườngtròn nội tiếp các tam giác ABC, SPQ; (I), (K), (Ia) lần lượt tiếp xúc với BC tạiD.D′, T . Gọi L là điểm đối xứng với T qua Ia. Ta có JA ‖ IaL và S là tâm vị tựtrong của (J) và (Ia) nên A, S, L thẳng hàng.

Lại có A là tâm vị tự ngoài của (I) và (Ia), đồng thời ID ‖ IaL nên A,D,L thẳnghàng. Do đó A,D, S, L thẳng hàng. Mặt khác, S là tâm vị tự ngoài của (K) và(Ia), đồng thời KD′ ‖ IaL nên S,D′, L thẳng hàng. Suy ra D ≡ D′. Từ đó ta cóDP = TQ. Mà DB = TC nên BP = CQ.

Nhận xét. Ngoài lời giải trên, tác giả Nguyễn Văn Linh còn có phần mở rộng rấtđáng chú ý cho bài toán này, xin giới thiệu cùng bạn đọc.Trong lời giải bài toán trên chỉ dùng tới dữ kiện JA ⊥ BC, do đó ta mở rộng bàitoán như sau:

Bài toán. Cho tam giác ABC với (Ia) là đường tròn bàng tiếp góc A. Một đườngtròn bất kì qua B,C cắt AC,AB lần lượt tại E,F . Tiếp tuyến chung trong của(AEF ) và (Ia) cắt BC tại P,Q. Chứng minh rằng BP = CQ.

Nếu thay đường tròn bàng tiếp góc A bằng đường tròn nội tiếp (I), ta được bàitoán tương tự.

Bài toán. Cho tam giác ABC với (I) là đường tròn nội tiếp. Một đường tròn bấtkì qua B,C cắt AC,AB lần lượt tại E,F . Tiếp tuyến chung trong của (AEF ) và(I) cắt BC tại P,Q. Chứng minh rằng BP = CQ.

Để ý rằng ở bài toán ban đầu, đường tròn đường kính PQ tiếp xúc với đường tròn(AEF ) và (O). Vậy ta có bài toán sau:

Bài toán. Cho tam giác ABC nội tiếp đường tròn (O), ngoại tiếp đường tròn (I).Một đường tròn tâm J bắt kì qua B,C cắt AC,AB tại E,F . Tiếp tuyến chungngoài của (AEF ) và (I) cắt BC tại P,Q. Chứng minh rằng (J, JP ) tiếp xúc với(O) và (AEF ).

Tới đây ta thay đường tròn nội tiếp (I) bằng đường tròn bàng tiếp (Ia) của gócA với P ′, Q′ là giao của tiếp tuyến chung trong của (AEF ) và (Ia) với BC thì(J, JP ′) vẫn tiếp xúc với (O), (AEF ) lần lượt tại M,K. Do đó P ′ ≡ P,Q′ ≡ Q. Tathu được bài toán sau:

Bài toán. Cho tam giác ABC ngoại tiếp đường tròn (I), (Ia) là tâm đường trònbàng tiếp góc A. Một đường tròn bất kì qua B,C cắt AC,AB lần lượt tại E,F .Một tiếp tuyến chung ngoài của (AEF ) và (I) cắt BC tại P . Chứng minh rằng Pnằm trên tiếp tuyến chung trong của (AEF ) và (Ia).

2.2 Hình học 57

Cho đường tròn (O1) và (O2) ngoài nhau có AB là một tiếp tuyến cungngoài và CD là một tiếp tuyến chung trong. Gọi P = AB ∩ CD,Q =AD ∩BC. Chứng minh rằng PQ ⊥ O1O2.

Bài 4

Lời giải. Ta sẽ chứng minh AC,BD,O1O2 đồng quy. Gọi Y = PQ ∩ BD ⇒(BD, Y X) = −1. Mặt khác, ta có 4O1AC ∼ 4PBD ⇒

Y B

Y D=XB

XD=

XB

MP·

MP

XD=

AX

AM· CMCX

=AX

CX⇒ 4O1AX ∼ 4PBY ⇒ O2PQ = X1OM ⇒

O1PO2 = 90◦. Từ đây, ta có điều phải chứng minh.

O1

O2

A

B

C

D

P

Q

X

Y

Cho tứ giác ABCD nội tiếp đường tròn tâm O và một điểm P thuộc cungCD không chứa A và B. Gọi E = PA ∩ BD,G = BP ∩ AC,H = AP ∩CD,F = BP ∩CD,Q = EF ∩HG. Chứng minh rằng PQ luôn đi qua mộtđiểm cố định khi P di chuyển.

Bài 5

58 HƯỚNG DẪN GIẢI

O

A

B C

D

P

E

FG

HQR

X

Lời giải. Gọi PQ cắt (O) tại điểm thứ hai là R và tiếp tuyến tại P cắt CD tại X.

Áp dụng định lí Pascal cho bộ 6 điểm(P C BD P A

), ta có ba điểm E, G,

X thẳng hàng. Suy ra, A(XQ,GE) = −1 ⇒ P (PR,BA) = −1 ⇒ APBR điềuhoà. Từ đó, ta kết luận PQ đi qua giao điểm của hai tiếp tuyến tại A và B. Bàitoán chứng minh hoàn tất.

Nhận xét.

1. Định lí Pascal có thể mở rộng cho một đường conic bất kì (không nhất thiếtphải là đường tròn).

2. Một số bài tập ứng dụng định lí Pascal:

(a) Tứ giác ABCD nội tiếp có tiếp tuyến tại A và B cắt nhau ở N1, tại Bvà C cắt nhau ở N2, tại C và D cắt nhau ở N ′1, tại D và A cắt nhauở N ′2; các đường thẳng AD và BC cắt nhau ở M1, các đường thẳngAB và CD cắt nhau ở M2; AC và BD cắt nhau ở P . Chứng minh bốnđiểm M1, N1, P,N

′1 thẳng hàng và bốn điểm M2, N2, P,N

′2 thẳng hàng.

(b) Cho một đường tròn với hai dây AB và CD không song song. Đườngvuông góc với AB kẻ từ A cắt đường vuông góc với CD kẻ từ C và từ Dlần lượt tại M,P . Đường vuông góc với AB kẻ từ B cắt đường vuônggóc với CD kẻ từ C và D lần lượt tại Q và N . Chứng minh rằng cácđường thẳng AD,BC,MN đồng quy và các đường thẳng AC,BD,PQcũng đồng quy.

2.2 Hình học 59

(c) (IMO Shortlist 2011) Cho ABC là một tam giác với đường tròn nộitiếp tâm I và đường tròn ngoại tiếp (C). D và E là giao điểm thứ haicủa (C) với các tia AI và BI tương ứng. DE cắt AC tại điểm F , và cắtBC tại điểm G. P là giao điểm của đường thẳng đi qua F song songvới AD và đường thẳng qua G song song với BE. Giả sử rằng K làgiao điểm của các tiếp tuyến của (C) tại A và B. Chứng minh rằng bađường thẳng AE,BD và KP song song hoặc đồng 0quy.

Cho tam giác ABC nội tiếp đường tròn tâm O cố định có hai điểm BC cốđịnh, A di chuyển trên (O). Gọi AD,BE,CF là ba đường cao của tam giáccắt nhau tại H. DE,DF lần lượt cắt HB,HC tại Q, R. Gọi M là trung điểmQR. Chứng minh rằng HM đi qua điểm cố định.

Bài 6

Lời giải. Trước hết xin phát biểu và chứng minh bổ đề sau.

Bổ đề 2. Cho 4ABC nội tiếp đường tròn (O) có hai tiếp tuyến tại B và C cắtnhau tại T . Phân giác BE, CF cắt nhau tại I. Khi đó IT chia đôi EF .

O

A

B C

E

F

I

T

M

60 HƯỚNG DẪN GIẢI

Thật vậy, chứng minh được BFC = FCT , CEB = EBT . Gọi M là giao điểmcủa EF và IT . Ta có:

SIME

SIMF

=SIME

SIBT· SIBTSIEC

· SIECSIFB

· SIFBSICT

· SICTSIMF

=IM · IEIB · IT

· IB ·BTIE · EC

· IF · FBIC · CT

· IC · ITIM · IF

=FB

EC· IEIF· ICIB

= 1

Từ đó, ta có điều phải chứng minh.

O

A

B

C

H

D

E

F

Q

RM

T

F ′

E′

S

X

Y

D′

Q′

R′

M ′

2.2 Hình học 61

Quay lại bài toán. Gọi D′, E ′, F ′ lần lượt là giao điểm thứ hai của ADBE,CFvới đường tròn (O). Tiếp tuyến tại B và F ′ cắt nhau tại X, tiếp tuyến tại F ′ vàE ′ cắt nhau tại S, tiếp tuyến tại E ′ và C cắt nhau tại Y .

Ta có H là tâm đường tròn nội tiếp 4DEF , từ đó suy ra H cũng chính là trựctâm 4D′E ′F ′. Sử dụng Bổ đề 2 cho 4D′E ′F ′, suy ra H,M,S thẳng hàng. Kếthợp với tứ giác XSY T ngoại tiếp đường tròn (O), ta có ST,CF ′, BE ′ đồng quytại H, suy ra H, M , T thằng hàng, kết thúc chứng minh.

Cho 4ABC nội tiếp đường tròn (O) cố định có B, C cố định và A di độngtrên (O). Gọi E, F lần lượt là điểm đồi xứng của B, C qua CA, AB. GọiM = CE∩AB,N = BF ∩AC. Chứng minh rằng đường thẳng qua A vuônggóc với MN đi qua một điểm cố định.

Bài 7

Hướng dẫn.

62 HƯỚNG DẪN GIẢI

O

A

B C

F

E

NM

P

J

K

X

Gọi P = BN ∩CM . Ta có A là tâm đường tròn nội tiếp4PBC. Do đó, BAC =

90◦ + BPC

2⇒ BPC = const. Suy ra tâm đường tròn ngoại tiếp ∆PBC cố định

(gọi tâm đường tròn này là K).

Gọi J là tâm đường tròn bàng tiếp góc P của ∆PBC. Ta chứng minh đượcKJ ⊥ MN ‖ d (với d là đường thẳng qua A vuông góc với MN). Gọi T làtrung điểm cung BC không chứa P của đường tròn (PBC). Suy ra được T là tađường tròn (ABC) nên T ≡ O. Từ đó, ta có O là trung điểm AJ . Sử dụng phépđối xứng tâm O, ta suy ra điều phải chứng minh.

2.2 Hình học 63

Cho tam giác ABC nhọn, BE,CF là các đường cao. M là trung điểm củaBC. N là giao điểm của AM và EF . Gọi X là hình chiếu của N lên BC.Y, Ztheo thứ tự là hình chiếu của X trên AB,AC .Chứng minh rằng N làtrực tâm tam giác AY Z.

Bài 8

A

B CM

E

F

N

X

Y

ZK

Lời giải. Dễ chứng minh khẳng định bài toán đúng trong trường hợp tam giácABC cân tại A. Xét trường hợp tam giác ABC không cân, không mất tổng quátgiả sử AB > AC.

Gọi K là hình chiếu của M trên EF . Vì BEC = BFC = 90o nên tứ giác BEFCnội tiếp đường tròn đường kính BC. Vì M là trung điểm của BC và MK vuônggóc với EF nên K là trung điểm của EF .

Cũng vì tứ giác BEFC nội tiếp nên AEF = ABC, dẫn tới hai tam giác AEFvà ABC đồng dạng. Lưu ý rằng AM,AK tương ứng là trung tuyến của các tam

64 HƯỚNG DẪN GIẢI

giác ABC,AEF nên các tam giác AKE,AMB đồng dạng. Điều này kéo theo

AKE = AMB.

Mặt khác MKN = MXN = 90o nên tứ giác MNKX nội tiếp, kéo theo

XKE = AMC.

Từ hai điều trên suy ra

AKE + XKE = AMB + AMC = 180o,

hay A,K,X thẳng hàng. Từ đó, chú ý hai tam giác AEF,ABC đồng dạng, tathu được XAC = KAE = NAF . Điều này dẫn tới

NE

NF=XB

XC.

Mặt khác, do CF,XY cùng vuông góc với AB nên CF ‖ XY . Theo định lýThales, ta suy ra

XB

XC=Y B

Y F.

VậyNE

NF=Y B

Y F, từ đó theo định lý Thales đảo, Y N ‖ BE. Mà BE vuông góc

với AC nên Y N cũng vuông góc với AC. Chứng minh tương tự ta cũng có ZNvuông góc với AB. Vì thế, N là trực tâm của tam giác AY Z.

Nhận xét.

1. Với trường hợp AB = AC, tứ giác MNKX sẽ suy biến thành đoạn thẳng.

2. Các tính chất của đường đối trung là ý tưởng xuyên suốt bài toán này.

• Nếu AN là đường đối trung của tam giác ABC với N ∈ [BC] thìNB

NC=

AB2

AC2.

• Giao điểm của hai tiếp tuyến tại B và tại C của đường tròn ngoại tiếptam giác ABC (nếu có) thuộc đường đối trung xuất phát từ A của tamgiác ABC

2.2 Hình học 65

Cho tam giác ABC có đường trung tuyến AM . Đường cao BE cắt đườngtrung tuyến AM tại P . Lấy Q sao cho QE ⊥ AM, CQ ⊥ AB. Chứng minhrằng AQ ⊥ CP .

Bài 9

A

BCM

P

E

Q

H

KL

Lời giải. Gọi H = CQ ∩ BE và K thuộc BC thoả HK ⊥ CP . Ta có P là trựctâm ∆HKC. Suy ra, PK ⊥ HC||AB. Gọi L thuộc AB sao cho PL||AC. Ta cóLB

LA=PB

PE=AK

KE. Lại có,

PB

PE=AC

AE, do đó

LB

LA=AK

KE=AC

AE=KC

KA. Mặt

khác, ∆PAB ∼ ∆EQC ⇒ LB

LA=HQ

HC. Do đó,

KC

KA=HQ

HC⇒ AQ ‖ HK ⊥

CP .

Cho tam giác ABC nội tiếp đường tròn (O). Tiếp tuyến tại B,C của đườngtròn (O) cắt nhau tại T . Gọi M,N lần lượt là các điểm thuộc tia BT,CTsao cho BM = BC = CN . Đường thẳng MN cắt CA,AB theo thứ tự tạiE,F ;BE giao CT tại P,CF giao BT tại Q. Chứng minh rằng AP = AQ.

Bài 10

Lời giải. Gọi AD là phân giác của tam giác ABC. Do B,C đối xứng nhau quaOT và BM = CN nên M,N đối xứng qua OT , suy ra BC ⊥MN .

66 HƯỚNG DẪN GIẢI

A

BC

T

D

O

M NE

F

P

Q

K

Ta có FBM = 180o − ABC − CBM = 180o − ABC − CAB = ACB, chú ý gócđồng vị ABC = BFM do đó 4ABC ∼ 4MFB. Từ đó ta chú ý FM ⊥ BC nêntheo định lý Thales

QC

QF=BC

FM=BM

FM=AC

AB=DC

DB⇒ QD ‖ BF.

Tương tự PD ‖ CE. Từ đó theo định lý Thales và tính chất đường phân giác tacó

DQ

DP=DQ

BF.BF

CE.CE

DP=CD

BC.AB

AC.BC

BD=CD

BD.AB

AC= 1⇒ DP = DQ.

Nhận xét. Bài toán trên được lấy của tác giả Nguyễn Minh Hà, và cấu hình bàitoán là rất cũ. Tuy nhiên, rất bất ngờ khi chỉ có rất ít các bạn làm hoàn chỉnh bàitoán này. Có lẽ ở trong phòng thi, các bạn đã không tìm ra điểm mấu chốt là phângiác AD, dẫn tới việc sa vào dùng định lý sin để tính toán và mắc kẹt ở đó. Nếu ápdụng thành thục định lý sin cũng sẽ dẫn tới kết quả, nhưng nó đòi hỏi tính toánthật nhanh và chính xác.Tổng quát của bài toán như sau:

2.2 Hình học 67

Bài toán (Trần Quang Hùng). 4ABC nội tiếp (O).M,N thuộc cung BC sao choMN ‖ BC và tia AM nằm giữa hai tia AB,AN . Trên tia BM,CN lấy các điểmP,Q sao cho BP = BN = CM = CQ. PQ cắt AM,AN tại S, T . CS cắt BP tạiK và BT cắt CQ tại L. Chứng minh rằng AK = AL.

Lời giải của bài toán tổng quát trên có thể được tìm thấy trên fanpage Trườngđông Toán học miền Nam 2017.

Cho hai đường tròn (O1) và (O2) tiếp xúc ngoài tại M . Một đường thẳngcắt (O1) tại A,B và tiếp xúc với (O2) tại E (B nằm giữa A và E). Đườngthẳng EM cắt (O1) tại điểm J khác M . C là một điểm thuộc cung MJkhông chứa A,B của (O1) (C khác M và J). Kẻ tiếp tuyến CF với đườngtròn (O2) (F là tiếp điểm) sao cho các đoạn thẳng CF và MJ không cắtnhau. Gọi I là giao điểm của các đường thẳng CJ và EF , K là giao điểmkhác A của đường thẳng AI và đường tròn (O1). Chứng minh rằng:

1. Tứ giác MCFI là tứ giác nội tiếp và JA2 = JI2 = JM.JE.

2. CI là phân giác ngoài tại C của tam giác ABC.

3. K là tâm đường tròn ngoại tiếp của tam giác BCI.

Bài 11

O1M

O2

E

AB

J

C

F

IK

D

Lời giải.

68 HƯỚNG DẪN GIẢI

1. Đường thẳng JO1 cắt (O1) tại D, ta có JD là phân giác trong góc AJB,CD là phân giác trong góc ACB. Dễ thấy JO1 ‖ EO2 (bạn đọc tự chứngminh) ⇒ JO1M = EO2M ⇒ MBJ = MCI = MFI ⇒ Tứ giác MCIF

nội tiếp ⇒ MEF = MFJ = MIJ ⇒ IJ2 = JM.JE. Ngoài ra, ABJ =

BAJ = BME ⇒ JBE = BMJ ⇒ KA2 = JB2 = JM, JE.

2. Dễ thấy CD ⊥ IJ , mà CD là phân giác trong góc ACB nên CI là phângiác ngoài góc ACB.

3. Câu 1 cho ta JA = JB = JI ⇒ AIJ = IAJ = JBK. Mà JBI = JIB ⇒KBI = KIB ⇒ KB = KI. Cuối cùng, KCI = KAJ = KIC ⇒ KI =KC. Vậy KB = KC = KI, và ta có K là tâm ngoại tiếp 4KCI.

Nhận xét. Bài toán này không gây quá nhiều khó khăn cho các bạn khi làm bài.Bổ đề đơn giản sau đóng vai trò quyết định cho hai câu đầu, và gợi ý cho câu cuối:Cho hai đường tròn (O1) và (O2) tiếp xúc ngoài tại M . Một tiếp tuyến chung ngoàitại C ∈ (O2) của (O2) cắt (O1) tại hai điểm A,B phân biệt. Đường thẳng CM cắt(O1) tại điểm thứ hai D. Khi đó DA = DB.

O1 M O2

C

A

B

D

2.2 Hình học 69

Cho tam giác ABC. Đường tròn (K) bất kỳ tiếp xúc đoạn thẳng AC,AB lầnlượt tại E,F . (K) cắt đoạn thẳng BC tại M,N sao cho N nằm giữa B vàM . FM giao EN tại I. Đường tròn ngoại tiếp các tam giác IFN và IEMcắt nhau tại J khác I. Chứng minh rằng IJ đi qua A và KJ vuông góc IJ .

Bài 12

A

B C

K

F

E

N M

I

J

Lời giải. Gọi J0 là hình chiếu của K lên AI, dễ thấy A,F, J0, K,E thuộc đườngtròn đường kính AK. Từ đó ta có góc nội tiếp và góc tạo bởi tiếp tuyến và dâycung bằng nhau ∠′F = ∠AEF = ∠FNI ⇒ Tứ giác FJ ′IN nội tiếp. Tương tựtứ giác EJ0IM nội tiếp từ đó J0 là điểm chung khác I của đường tròn ngoạitiếp tam giác IFN và IEM , vậy j′ ≡ J ⇒ A, I, J thẳng hàng⇒ KJ⊥IJ .

70 HƯỚNG DẪN GIẢI

Cho (I) là đường tròn nội tiếp tam giác nhọn ABC, tiếp xúc BC,CA,AB ởD,E, F . (O) là đường tròn ngoại tiếp tam giác ABC. Lấy D′ đối xứng vớiD qua EF .

1. Chứng minh rằng AD′, BC,OI đồng quy.

2. Gọi H, J lần lượt là trực tâm các tam giác ABC và AEF . Gọi R làgiao HJ và ID, T là giao D′J và OI. Chứng minh rằng D′, R, T, Iđồng viên.

Bài 13

O

B

C

A

I

D

F

E

D′

S

H

J

R

T

Lời giải.

1. • Gọi A′, B′, C ′ lần lượt là điểm chính giữa các cung nhỏ BC,CA,ABthì 4A′B′C ′ có OI là đường thẳng Euler và có phép vị tự biến

2.2 Hình học 71

4A′B′C ′ thành 4DEF . Do đó OI song song hoặc trùng với đườngthẳng Euler của 4DEF . Mặt khác I là tâm (DEF ) nên OI là đườngthẳng Euler của 4DEF .

• Ta đưa hình vẽ về dạng như sau:

I

D

E F

D′

A

S

K

K′

V

Với S là giao BC và AD′, K là trực tâm của 4DEF , K ′ là giao DD′

với (I) và V là giao AI với SD.

Ta sẽ chứng minhD′K

DK=IA

IV. Khi đó theo Thales ta sẽ có K thuộc

IS.

Do tính đối xứng ta cóD′K

DK=DK ′

DK.

Đặt IDK = α và bán kính của (I) là R. Ta có IV =R

cosα.

Đồng thời IA =R

cos EDF. Suy ra

IA

IV=

cosα

cos EDF.

72 HƯỚNG DẪN GIẢI

Dễ thấy DK = 2R cos EDF và DK ′ = 2R cosα (Suy từ định lý sin

cho (I)). Từ đóDK ′

DK=

cosα

cos EDF=IA

IV. (đpcm)

2. Ta đưa hình vẽ về dạng như sau:

O

B

C

A

I

D

F

E

D′

S

H

J

R

T

M

K

L

N

K′

Ta thấy I và J đối xứng qua EF nên D′J và DI giao nhau ở N trên EF .Gọi L là giao DJ và OI thì L là tâm Euler của 4DEF và DKJI là hìnhbình hành.Gọi M là chân đường cao từ D của 4DEF . Ta có kết quả quen thuộcM,H, J thẳng hàng.

Ta có HAJ = IJK = JKM và AHJ = MJK.Từ đó 4AHJ ∼ 4KJM nên

AH

AJ=

KJ

KM=

ID

KM(2.19)

2.2 Hình học 73

Ta cóIL

IT=

1

2.IK

IT=

1

2.JD′

JT=

ID

2JT. (2.20)

Theo định lý Thales ta cóJN

JD′=KD

KD′=DK

DK ′nên IN =

DK

DK ′.JK

Do đó JK − IN =2ID.KM

DK ′.

Lại cóTJ

NT=JK

IN⇒ TJ

IN=

JK

JK − IN=

DK ′

2KM

⇒ 2JT =IN.DK

KM=DK.JK

KM=ID.DK

KM.

kết hợp với (2.19) và DK = IJ ta có

2JT =AH.IJ

AJ.

Lại dùng Thales cho 4AJH có IR ‖ AH thì thấy 2JT = IR.

Kết hợp với (2.40) ta cóIL

IT=ID

IRnên DJ ‖ RT .

Từ đó có

NJ

NT=NR

ND⇒ NI

NT=

NR

ND′⇒ NI.NR = NT.ND′.

Vì vậy D′, R, I, T đồng viên.

Cho tam giác ABC,P là điểm bất kỳ. A1 là hình chiếu của P lên BC.A2 làtrung điểm AA1.A2P cắt BC tại A3.A4 đối xứng A1 qua A3. Chứng minhrằng PA4 luôn đi qua một điểm cố định.

Bài 14

Lời giải. Chúng ta phát biểu và làm bài toán tổng quát hơn.

Bài toán. Cho tam giác ABC,P là điểm bất kỳ. A1 là hình chiếu song song củaP theo phương l cố định lên BC.A2 là trung điểm AA1.A2P cắt BC tại A3.A4 đốixứng A1 qua A3. Chứng minh rằng PA4 luôn đi qua một điểm cố định.

74 HƯỚNG DẪN GIẢI

A

B C

P

A1

A2

A3A4

A5

Gọi L là hình chiếu song song phương l của A lên BC. Gọi A5 là trung điểmAL, ta sẽ chứng minh rằng A4, P, A5 thẳng hàng thật vậy, từ liên hệ tỷ số đơnvà tỷ số kép dễ thấy

A1(A4A2A5P ) = (LAA5) = −1( do PA1 ‖ AL và A5 là trung điểm AL.

A2(A4A1A3A5) = (A4A1A3) = −1( do A2A5 ‖ A1A4 và A3 là trung điểm A1A4.

Từ đó A1(A4A2A5P ) = A2(A4A1A3A5) nên A4, A5, P thằng hàng. Ta có điềuphải chứng minh.

Nhận xét. Ngoài lời giải tỷ số kép, có một số cách tiếp cận khác chỉ sử dụng côngcụ hình học lớp 8, tuy vậy với cách làm bằng tỷ số kép, các bạn hoàn toàn có thểtổng quát bài toán.

Cho năm điểm A, B, C, D và E cùng nằm trên một đường tròn. Gọi M , N ,P và Q lần lượt là hình chiếu vuông góc của E xuống các đường thẳng AB,BC, CD và DA. Gọi hình chiếu vuông góc của E xuống các đường thẳngMN , NP , PQ, QM lần lượt là R, S, T và U cùng nằm trên một đườngthẳng.

Bài 15

Lời giải. Ta dễ dàng chứng minh được bổ đề sau:

2.2 Hình học 75

Bổ đề 3. Cho tam giác ABC nội tiếp đường tròn (O) và M là một điểm bất kìtrên đường tròn đó. Gọi chân các đường vuông góc kẻ từ M lên ba đường thẳngBC, CA, AB lần lượt là D, F , E. Khi đó D, F , E thẳng hàng (đường thẳng chứaD, F , E là đường thẳng Simson ứng với điểm M của tam giác ABC).

O

B

A

C

D

E

M

N

P

Q

R

S

T

U

I

J

Quay lại bài toán. Gọi I, J lần lượt là hình chiếu vuông góc của E xuống cácđường thẳng AC,BD. Theo bổ đề trên ta thấy:

• M,N, I thẳng hàng (đường thẳng Simson ứng với điểm E của tam giácABC). Do đó R thuộc đường thẳng MI.

• N,P, J thẳng hàng (đường thẳng Simson ứng với điểm E của tam giácBCD). Do đó S thuộc đường thẳng NP .

Mặt khác, ta thấy M, I,Q,E cùng nằm trên đường tròn đường kính AE, nêntừ kết quả trên ta có R,U, T thẳng hàng (đường thẳng Simson ứng với điểmE của tam giác MIQ). Lại có I,N, P,E cùng nằm trên đường tròn đường kínhCE, nên R, S, T thẳng hàng (đường thẳng Simson ứng với điểm E của tam giácNPI. Vậy R, S, T, U thẳng hàng.

76 HƯỚNG DẪN GIẢI

2.3 Phương trình hàm - Dãy số

Tìm các hàm f : R→ R thoả mãn:

f(x2) + f(xy) = f(x)f(y) + yf(x) + xf(x+ y), ∀ xy ∈ R (2.21)

Bài 1

Lời giải. Bài này là một bài khá là quen thuộc và đã xuất hiện khá nhiều cácgiải trên mạng. Bài viết sẽ trình bày 2 cách giải quen thuộc nhất. Nhưng trứchết giai đoạn đầu là như nhau:

Thay x = y = 0 vào đề ta có:

2f(0) = (f(0))2 ⇔[f(0) = 2f(0) = 0

Với f(0) = 2 ta thay x = 0 vào đề ta có:

4 = 2f(y) + 2y ⇔ f(y) = 2− y, ∀y ∈ R

Thử lại ta có:{VT = 2− x2 + 2− xy = 4− x2 − xy

VP = (2− x)(2− y) + y(2− x) + x(2− x− y) = 4− x2 − xy

Vậy ta có một nghiệm là f(x) = 2− x, ∀ x ∈ R.

Với f(0) = 0 thì có ba cách xử lí:

• Cách 1: Nghiệm lai.

Thay y = −x ta có:

f(x2) + f(−x2) = f(x)f(−x)− xf(x) (2.22)

Thay x = −y ta có:

f(y2) + f(−y2) = f(y)f(−y) + yf(−y) (2.23)

Từ (2.22) và (2.23) ta có:

f(−x) = −f(x), ∀x ∈ R

2.3 Phương trình hàm - Dãy số 77

Thay ngược lại (2.22) ta có:

0 = −(f(x))2 − xf(x)⇔ f(x)(f(x) + x) = 0, ∀x ∈ R

Điều trên dẫn đến

f(x) = 0, ∀x ∈ A (2.24)f(x) = −x, ∀x ∈ B (2.25)

với A,B ⊂ R và A ∪B = R.

Ta sẽ đi chứng minh A = B = R. Sử dụng phản chứng, tức tồn tại x0 6=y0 ∈ R, x0y0 6= 0 sao cho: {

f(x0) = 0f(y0) = −y0

Thay y = 0 vào đề ta có:

f(x2) = xf(x), ∀x ∈ R (2.26)

Lần lượt thay x = x0 và x = y0 vào (2.26) ta có được kết quả f(x20) = 0 vàf(y20) = −y20.

Thay x = x0, y = y0 và đề, sau đó đổi thứ tự và thu gọn ta có được:

y20 = x0y0 + (x0 − y0)f(x0 + y0)

Nếu f(x0 + y0) = 0 thì ta có y20 = x0y0 dẫn đến x0 = y0 hoặc y0 = 0, vô lý.

Nếu f(x0 + y0) = −x0 − y0 thì ta có: y20 = x0y0 + y20 − x20 dẫn đến x0 = y0hoặc x0 = 0, vô lý.

Vậy A = B = R hay ta có hai nghiệm nữa là f(x) = 0, ∀x ∈ R vàf(x) = −x, ∀x ∈ R.

• Cách 2: Ta vẫn có thể tính được f là hàm lẻ. Khi đó thay y = 0 vào (2.21)ta có:

f(x2) = xf(x), ∀x ∈ R (2.27)

Thay đổi vai trò của x và y trong (2.21) ta có:

f(y2) + f(xy) = f(x)f(y) + xf(y) + yf(x+ y), ∀x, y ∈ R (2.28)

78 HƯỚNG DẪN GIẢI

Lấy (2.21) trừ (2.28) vế theo vế ta có:

f(x2)− f(y2) = yf(x)− xf(y) + (x− y)f(x+ y), ∀x, y ∈ R

Thay (2.27) vào phương trình trên ta có:

(x− y)f(x) + (x− y)f(y) = (x− y)f(x+ y), ∀x, y ∈ R

hayf(x) + f(y) = f(x+ y), ∀x 6= y ∈ R (2.29)

Ngoài ra, thay y = −x vào (2.21) ta có:

(f(x))2 = −xf(x), ∀x ∈ R

và thay y = x vào (2.21), sử dụng đẳng thức trên ta có:

xf(x) + xf(x) = −xf(x) + xf(x) + xf(2x)⇔ f(2x) = 2f(x), ∀x 6= 0 ∈ R

Mà f(x) = 0 nên ta có

f(2x) = 2f(x), ∀x, y ∈ R (2.30)

Từ (2.29) và (2.30) ta có:

f(x) + f(y) = f(x+ y), ∀ x, y ∈ R (2.31)

Đến đây ta sẽ sử dụng phương pháp tính bằng hai cách f((x + 1)2) nhưsau:

f((x+1)2) = (x+1)f(x+1) = (x+1)(f(x)+f(1)) = xf(x)+f(x)+xf(1)+f(1)

f((x+ 1)2) = f(x2 + 2x+ 1) = f(x2) + 2f(x) +f(1) = xf(x) + 2f(x) +f(1)

Từ hai đẳng thức trên ta suy ra được f(x) = xf(1), ∀x ∈ R.

Từ đây dễ dàng tìm được f(1) = 0 hoặc f(1) = −1 hay ta có 2 nghiệm làf(x) = 0, ∀x inR hay f(x) = −x, ∀x ∈ R.

2.3 Phương trình hàm - Dãy số 79

Nhận xét. Đây là một bài toán khá dễ chịu, cả về ý tưởng lẫn hướng giải quyết.Với các giải thứ 2 thì sau khi đã nhận biết được sự công tính và tính chất f(x2) =xf(x) thì một các tự nhiên, ta sẽ suy nghĩ đến đại lượng f((x + 1)2) vì giải pháptính bằng 2 cách sẽ hữu hiệu với những gì đã có.Với cách đầu tiên, rõ ràng là khá nguy hiểm khi có nhiều bạn sẽ suy ra nghiệmngay từ phương trình f(x)(f(x) + x) = 0, ∀x ∈ R mà quên đi việc kiểm tranghiệm ngoại lai. Sau đây sẽ là một số bài toán tương tự cho cách 1:

1. (MOSP 2002) Tìm tất cả các hàm f : R→ R thoả mãn:

f(f(x) + y) = f(x2 − y) + 4f(x)y, ∀x, y ∈ R.

2. (Balkan 2000) Tìm tất cả các hàm f : R→ R thoả mãn:

f(xf(x) + f(y)) = (f(x))2 + y, ∀x, y ∈ R.

3. Tìm tất cả các hàm f : R→ R thoả mãn:

(f(x+ y))2 = f(x)f(x+ 2y) + yf(y), ∀x, y ∈ R.

4. Tìm tất cả các hàm f : R→ R thoả mãn:

f(x2 + f(y)) = xf(x) + y, ∀x, y ∈ R.

Cho hàm số :f : N→ N thoả

f(f(n)) + f(n) = 6n+ 4 (2.32)

1. Tính f(2017)

2. Tìm tất cả các hàm f thoả mãn.

Bài 2

Lời giải. Giả sử hàm f thõa mãn yêu cầu bài toán. Với n là số tự nhiên bất kì,taxét dãy {xn} như sau: x0 = n và xn+1 = f(xn).

Thay n bởi xn ta được:

xn+2 + xn+1 − 6xn − 4 = 0

80 HƯỚNG DẪN GIẢI

suy ra :xn = a.(−3)n + b.2n − 1.

Mà do

xn = 2n(b+ a

(−3

2

)n− 1

2n

).

Xét a < 0 thì khi chọn n chẵn và đủ lớn ta có xn tiến về âm vô cùng, vô lí. Tươngtự xét a > 0 và chọn n lẻ và đủ lớn cũng có được xn tiến về âm vô cùng ,vô lí.Vậy a = 0. Khi đó xn = b.2n − 1 .

Thay n = 0, 1 ta có được f(n) = 2n + 1. Thử lại đúng. Vậy f(n) = 2n + 1 vàf(2017) = 4035.

Nhận xét. Một hàm số có tập xác định trên N luôn được biểu diễn như một dãysố vì xn = f(n) với f : N→ R. Phần việc tìm số hạng tổng quát và tính giá trị rấtquen thuộc với nhưng học sinh chuyên toán.Bài này tất cả các đoàn đều làm đúng và được trọn vẹn điểm.

Cho dãy số

(xn) :

{x1 = x2 = 97

xn+2 = xn+1xn +√(

x2n+1 − 1)

(x2n − 1) ∀n ≥ 1.

1. Chứng minh 2 + 2xn là số chính phương.

2. Chứng minh 2 +√

2 + 2xn là số chính phương,

Bài 3

Hướng dẫn. Ta chứng minh bằng quy nạp:

•(2 +√

3)k

+1(

2 +√

3)k ∈ Z ∀k ∈ N

• xn =1

2

((2 +√

3)4Fn

+1(

2 +√

3)4Fn

)∀n ∈ N, trong đó Fn là dãy Fi-

bonacci. Khi đó ta được

2 + 2xn =

((2 +√

3)2(Fn)

+1(

2 +√

3)2(Fn)

)2

.

2.3 Phương trình hàm - Dãy số 81

2 +√

2 + 2xn =

((2 +√

3)(Fn)

+1(

2 +√

3)(Fn)

)2

.

Xét dãy số nguyên {an}∞n=1 thoả mãn:

a1 = 2; a2 = 7;−1

2< an+2 −

a2n+1

an≤ 1

2. (2.33)

Chứng minh rằng với mọi n > 1,ta có an là số lẻ.

Bài 4

Lời giải. Hệ thức đã cho ở đề bài là đặc trưng của dãy truy hồi cấp 2. Ta có giữa

hai sốa2n+2

an− 1

2và

a2n+2

an+

1

2có duy nhất 1 số nguyên nên dãy đã cho xác định

duy nhất.

Tính một số giá trị đầu:a1 = 2, a2 = 7, a3 = 25, a4 = 89. Nên ta nghĩ đến dãy sốxác định như sau:

a1 = 2, a2 = 7, và an+2 = 3an+1 + 2an. (2.34)

Ta sẽ chứng minh dãy số này thoả mãn đề bài. Bằng quy nạp ta sẽ chứng minh:a2n+1 − anan+2 = (−1)n.2n−1.

• Với n = 1,đẳng thức hiển nhiên đúng.

• Giả sử đẳng thức đúng đến k − 1.Hay a2k − ak−1ak+1 = (−1)k−1.2k−2. Taphải chứng minh:a2k+1 − akak+2 = (−1)k.2k−1 hay

a2k+1 − akak+2 = −2(a2k − ak−1ak+1)

⇔ a2k+1 − 2ak+1ak−1 = akak+2 − 2a2k⇔ ak+1(ak+1 − 2ak−1) = ak(ak+2 − 2ak)

⇔ ak+1 − 2ak−1ak

=ak+2 − 2ak

ak+1

Hiển nhiên đúng do các tỉ số trên đều bằng 3 theo công thức truy hồi.Đẳng thức được chứng minh.

82 HƯỚNG DẪN GIẢI

Trở lại bài toán. Từ đẳng thức ta có

an+2 −a2n+1

an=

(−1)n.2n−1

an(2.35)

Ta phải chứng minh∣∣∣∣∣(−1)n.2n−1

an

∣∣∣∣∣ ≤ 1

2, hay an ≥ 2n với n > 1 (2.36)

• Với n = 2,hiển nhiên.

• Giả sử an ≥ 2n,do an+1 = 3an + 2an−1, nên an+1 > 2n+1,đúng.

Vậy ta có (2.36),hay dãy số này thoả mãn bài toán, từ tính duy nhất ta có dãyvừa lập chính là dãy trong bài toán.

Giờ ta có:an+2 = 3an+1 + 2an ⇒ an+2 ≡ an+1 (mod 2) (2.37)

Mà a2 = 7 nên ta có an là số lẻ với mọi n > 1, điều phải chứng minh.

Tìm f : R→ R thoả mãn:

f(x3 + f(y)) = y + f 3(x) (2.38)

với mọi x, y thực.

Bài 5

Lời giải. Ta sẽ chứng minh f(0) = 0. Từ đề bài dễ thấy f là một song ánh. Dovậy sẽ tồn tại duy nhất số thực b sao cho f(b) = 0. Đặt f(0) = a.Trong (2.36) cho x = y = 0 :

f(a) = a3.

Trong (2.36) cho y = a, x = b :

f(b3 + a3) = a

Suy ra f(b3 + a3) = a = f(0), kéo theo b3 + a3 = 0.Trong (1) cho x = 0, y = b :

f(0) = b+ a3 ⇔ a3 + b = a

2.3 Phương trình hàm - Dãy số 83

Trong (2.36) cho x = 0, y = a :

f(f(a)) = a+ f 3(0)⇔ f(a3) = a+ a3

Trong (2.36) tiếp tục cho x = a, y = b :

f(a3) = b+ f 3(a) = b+ a9

Từ đó suy ra b+ a9 = a+ a3.Giải hệ gồm ba phương trình hai ẩn sau :

b+ a9 = a+ a3

a3 + b3 = 0a3 + b = a

⇔ a = b = 0

Vậy tóm lại là f(0) = 0. Từ đó dễ dàng suy ra được :

f(f(y)) = y,∀y ∈ R (2.39)

f(x3) = f 3(x),∀x ∈ R

Kết hợp hai điều trên với phương trình hàm đã cho ta suy ra được :

f(x+ y) = f(x) + f(y),∀x ∈ R

Khi đó sẽ có :f(n) = f(1).n,∀n ∈ Z

Thay vào (2.39) ta có f(1) = ±1.

Nếu f(1) = 1 ta có

f((x+ 1)3) = [f(x) + 1]3 = f 3(x) + 3f 2(x) + 3f(x) + 1, ∀x ∈ R

Cũng có :

f((x+ 1)3) = f(x3 + 3x2 + 3x+ 1) = f 3(x) + 3f(x2) + 3f(x) + 1,∀x ∈ R

So sánh hai kết quả trên thì được :

f(x2) = f 2(x),∀x ∈ R (2.40)

Điều này cho ta tính chất :

f(x) ≥ 0,∀x ≥ 0.

84 HƯỚNG DẪN GIẢI

Kết hợp với tính cộng tính của f ta có được kết quả f(x) = f(1)x = x.

Tương tự với f(1) = −1 ta có:

f(x2) = −f 2(x)

hay f(x) ≤ 0 nếu x ≥ 0, từ đó sẽ có f giảm. Khi đó f(x) = f(1)x = −x.

Vậy ta có 2 nghiệm hàm là f(x) = x và f(x) = −x với mọi x ∈ R.

Nhận xét. Đây là một bài toán không mới, nhưng các kỹ thuật của bài toán nàyrất hay được sử dụng. Bài toán có thể suy ra được rất nhiều giả thuyết cho f nhưf song ánh (f(f(x)) = x, ∀x ∈ R), f cộng tính và f(x3) = (f(x))3). Từ nhữngđiều đó, theo một các tự nhiên như cách 2 của A1 thì ta nên xét f((x + 13) bằnghai cách để tìm được những mối liên hệ còn lại (ở đây là tính đơn điệu của hàmsố).

Tìm tất cả các hàm f : R→ R thảo mãn:

f(xf(x+ y)) = f(yf(x)) + x2, ∀x, y ∈ R (2.41)

Bài 6

Lời giải. Đầu tiên ta sẽ chứng minh f (0) = 0 ⇔ x = 0. Thật vậy nếu f (0) 6= 0

thì thay x = 0 và y =x

f(0)thì ta có

f (0) = f (x) , ∀x ∈ R.

Tức là f (x) là hàm hằng nhưng dễ thấy hàm này không thoả, vậy f (0) = 0.

Ngoài ra nếu f (x) = 0 thì thay y = 0 ta có f(0) = f(0) + x2 ⇔ x = 0 ta đượcđiều phải chứng minh.

Thay y = 0 ta có:f (xf (x)) = x2.

Tiếp theo ta sẽ chứng minh f (x) đơn ánh. Thật vậy nếu với hai số a và b bất kìkhác 0 thoả mãn f(a) = f(b) thì thay x = a, y = b− a ta được:

f (af (b)) = f ((b− a) f (a)) + a2 = f ((b− a) f (a)) + f (af (a)) .

Nênf ((b− a) f (a)) = f (af (b))− f (af (a)) .

2.3 Phương trình hàm - Dãy số 85

Vì f (a) = f (b) nên

f ((b− a) f (a)) = 0⇔ (b− a) f (a) = 0⇔ a = b.

Vậy f đơn ánh. Ta có đánh giá:

f (xf (x)) = x2 = (−x)2 = f (−xf (−x))

Vì f là đơn ánh nên xf (x) = −xf (−x) hay f (x) = −f (−x).

Vì f (xf (x)) = x2 nên R+∪{0} ⊂ f (R) và vì f là hàm lẻ nên f (−xf (x)) = −x2nên R− ⊆ f (R). Điều này chứng tỏ f (R) = R (vì ta đã sẵn có f(R) ⊂ R do fđơn ánh).

Tiếp theo, vì f là một toàn ánh nên tồn tại u ∈ R sao cho f (u) = 1. Khi đó vìf (uf (u)) = u2 ⇒ f (u) = u2 ⇒ u2 = 1⇒ u = ±1 nên u chỉ có hai giá trị là ±1.

• Nếu u = 1, tức là f (1) = 1 thì khi đó thay x = 1 và y = x− 1 ta có:

f (f (x)) = f (x− 1) + 1

thay x = 1, y = −x− 1 ta có:

f (f (−x)) = f (−x− 1) + 1⇒ −f (f (x))− f (x+ 1) + 1

Từ đó ta có:

f (x+ 1) = f (x− 1) + 2⇒ f (x+ 2) = f (x) + 2⇒ f (x+ 4) = f (x) + 4

Mặt khác thay x = 2 ta có:

f (2f (y + 2)) = f (yf (2)) + 4 = f (2y + 4)

⇒ 2f (y + 2) = 2y + 4⇒ f (y) = y.

• Tương tự với u = −1 ta được f (x) = −x.

Thử lại thấy cả hai hàm đều thoả.

86 HƯỚNG DẪN GIẢI

Tìm tất cả hàm số f : R→ R thoả:

f(f(y) + x2 + 1) + 2x = y + (f(x+ 1))2, ∀x, y ∈ R (2.42)

Bài 7

Lời giải. Thay x = 0 vào ta có:

f(f(y) + 1) = y + f(1)2 (2.43)

Từ đây ta suy ra f song ánh. Do đó tồn tại k thoả mãn f(k) = 0.

Thay x = 0, y = k vào ta được

f(1) = k + f(1)2 ⇒ k = f(1)− f(1)2 = a− a2

với a = f(1).f(f(y) + 2) = 2 + y + f(0)2 (2.44)

Từ (2.43) và (2.44) dẫn đến

f(f(y) + 2)− f(f(y) + 1) = 2 + f(0)2 − f(1).

Vì tính song ánh của f nên từ phương trình này ta suy ra f(x + 1) − f(x) =2 + f(0)2 − f(1)2 với mọi x ∈ R. Khi đó

f(1)− f(0) = 2 + f(0)2 − f(1)2. (2.45)

Ta cũng có f(2)− f(1) = f(1)− f(0) hay f(2) = 2f(1)− f(0).

Thay x bởi −x ta có:

f(f(y) + x2 + 1)− 2x = y + f(1− x)2 (2.46)

Từ (2.46) và phương trình đề ta suy ra

f(1− x)2 + 2x = f(x+ 1)2 − 2x. (2.47)

Thay x = 1 vào phương trình (2.47) ta được f(0)2+4 = f(2)2 = [2f(1)− f(0)]2.Do đó f(0)f(1) = f(1)2 − 1. Nhân cả hai vế của (2.45) với f(1)2 = a2 ta suy ra

a3 − a(a2 − 1) = 2a2 + (a2 − 1)2 − a4 ⇔ a = f(1) = 1.

Ta suy ra k = f(1)− f(1)2 = 0 hay f(0) = 0. Khi đó f(x+ 1)− f(x) = 1.

2.3 Phương trình hàm - Dãy số 87

Thay x bởi f(x) ta được f(f(x) + 1) = f(f(x)) + 1. Mặt khác, từ (2.43) ta cóf(f(x) + 1) = x+ 1 nên ta suy ra f(f(x)) = x.

Thay x = 1 ta có: f(y2 + 2) = f(y+ 1)2 + 1− 2y. Vì tính toàn ánh của f nên tồntại h thoả mãn f(h) = −2x. Ta suy ra

x2 + 2 = f(f(x2 + 2)) = f(f(x+ 1)2 + 1 + f(h)

)= h− 2f(x+ 1) + f (f(x+ 1) + 1)2 = h− 2 (f(x) + 1) + (x+ 2)2.

Từ đây ta tìm được h = 2f(x)− 4x hay

f(2f(x)− 4x) = −2x (2.48)

P (2x+ 1, x)⇒ f(f(2x+ 1) + x2 + 1

)= f(x+ 1)2 + 1

Do đó

f(2x+ 1) + x2 + 1 = f(f(f(2x+ 1) + x2 + 1

))= f

(f(x+ 1)2 + 1 + f(0)

)= f (f(x+ 1) + 1)2 − 2f(x+ 1) = (x+ 2)2 − 2f(x)− 2

Ta tìm được f(2x) + 2f(x) = 4x hay

2f(x)− 4x = −f(2x). (2.49)

Từ (2.48) và (2.49) ta suy ra f (−f(2x)) = −2x = f (f(−2x)). Vì f là đơn ánhnên

f(−2x) = −f(2x)

hay f(−x) = −f(x)∀x ∈ R. Áp dụng các tính chất của hàm vừa được chứngminh ở trên, ta sẽ có

(2.47)⇔ f(x−1)2+4x = f(x+1)2 ⇔ [f(x)− 1]2+4x = [f(x) + 1]2 ⇔ f(x) = x.

Vậy f(x) = x, ∀x ∈ R.

88 HƯỚNG DẪN GIẢI

Tìm tất cả các hàm f(x) : [1; +∞)→ [1; +∞) thoả{x ≤ f(x) ≤ 2x+ 2

xf(x+ 1) = (f(x))2 − 1(2.50)

Bài 8

Lời giải. Ta có :

x+ 1 ≤ f(x+ 1) ≤ 2x+ 4 ∀x ∈ [1,+∞)

⇒ x2 + x ≤ xf(x+ 1) = f 2(x)− 1 ≤ 2x2 + 4x,∀x ∈ [1,+∞)

⇒(x+

1

4

)2

+3

4≤ f 2(x) ≤ 2(x+ 1)2 − 2,∀x ∈ [1,+∞)

⇒ x+1

4≤ f(x) ≤

√2(x+ 1),∀x ∈ [1,+∞)

⇒ x+ 1√2≤ f(x) ≤

√2(x+ 1),∀x ∈ [1,+∞)

Thực hiện liên tiếp quá trình này, ta được :

x+ 12n√

2≤ f(x) ≤ 2n

√2(x+ 1),∀x ∈ [1,+∞)

Với mỗi x thuộc nửa khoảng từ 1 tới dương vô cực, ta cho n→ +∞ ta được :

x+ 1 ≤ f(x) ≤ x+ 1,∀x ∈ [1,+∞)

Như vậy ta phải có :f(x) = x+ 1, ∀x ∈ [1,+∞)

Thử lại thấy thoả. Đó là đáp số duy nhất của bài toán.

Tìm tất cả các hàm f : R→ R thoả mãn:

f(yf(x)− x) = f(x)f(y) + 2x, ∀x, y ∈ R (2.51)

Bài 9

2.3 Phương trình hàm - Dãy số 89

Lời giải. Thay x = y = 0 vào (2.51) ta được f(0) = (f(0))2 nên ta f(0) = 0hoặc f(0) = 1.

• Trường hợp f(0) = 0. Thay y = 0 vào (2.51), ta được f(x) = 2x, suy raf(x) = 2x với mọi x. Thử lại, ta thấy thoả mãn.

• Trường hợp f(0) = 1. Thay y = 0 vào (2.51), ta được

f(−x) = f(x) + 2x, ∀x ∈ R (2.52)

Thay y bởi −y vào (2.51) và sử dụng (2.52), ta được:

f(yf(x) + x) + 2 [yf(x) + x] = f(−yf(x)− x)

= f(x)f(−y) + 2x = f(x) [f(y) + 2y] + 2x

từ đó suy raf(yf(x) + x) = f(x)f(y)

Do đó phương trình (2.51) có thể được viết lại thành:

f(yf(x)− x) = f(yf(x) + x) + 2x, ∀x, y ∈ R (2.53)

Với mọi x khác 0 mà f(x) 6= 0, thay y =x

f(x)vào (2.53), ta được

f(2x) = 1− 2x (2.54)

Với mọi x khác 0 mà f(x) = 0 ta có f(−x) = f(x) + 2x = 2x 6= 0 nên theotrên thì f(−2x) = 1 + 2x. Suy ra

f(2x) = f(−2x)− 4x = 1− 2x (2.55)

Từ (2.54) và (2.55), ta suy ra với mọi x 6= 0 thì f(x) = 1−2x. Mà f(0) = 1nên ta có f(x) = 1− x, ∀x ∈ R. Thử lại, ta thấy thoả mãn.

Tóm lại, có hai hàm số thoả mãn yêu cầu đề bài là f(x) = −2x và f(x) =1− x.

90 HƯỚNG DẪN GIẢI

Tìm f: R+ → R+ thoả mãn:

f(x+ y) + f(x).f(y) = f(xy) + f(x) + f(y) (2.56)

Bài 10

Lời giải. Ta thêm biến z ∈ R và thay y bởi y + z ta có:

f(x+ y + z) = f(x) + f(y + z) + f(xy + xz)− f(x)f(y + z)

= f(x) + f(y) + f(z) + f(xy) + f(yz) + f(zx) + f(x)f(y)f(z)

−f(x)f(y)− f(y)f(z)− f(z)f(x)

+f(x2yz)− f(xy)f(xz)− f(x)f(yz)

với mọi x, y, z ∈ R+.

Tương tự thay x = y còn y = x+ z ta có

f(x2yz)− f(xy)f(xz)− f(x)f(yz) = f(xy2z)− f(xy)f(yz)− f(y)f(xz) (2.57)

với mọi x, y, z ∈ R+.

Thay y = 1 vào (2.57) ta có:

f(x2z)− f(x)f(xz)− f(x)f(z) = f(xz)− f(x)f(z)− f(1)f(xz)

với mọi x, z ∈ R+. Điều này có nghĩa là f(x2z) = (1 − f(1))f(xz) + f(x)f(xz)với mọi x, z ∈ R+, vì thế

f(xy) = (1− f(1))f(y) + f(x)f(y)

với mọi x, y ∈ R+.

Từ f(xy) = (1 − f(1))f(y) + f(x)f(y) = (1 − f(1))f(x) + f(x)f(y) với mọix, y ∈ R+ ta được f(x) = c hoặc f(1) = 1.

Nếu f(x) = c, ta có c+ c2 = 3c, vì thế f(x) = 2 với mọi x ∈ R+.

Ngược lại, f(1) = 1, vì thế f(xy) = f(x)f(y) với mọi x, y ∈ R+, vì thế f(x+y) =f(x) + f(y) với mọi x, y ∈ R+,

Từ f : R+ → R+, vì thế ta có f(x) = cx với hằng số c. Thay lại ta có c = 1. Khiđó đáp án là f(x) = 2 với mọi x ∈ R+ và f(x) = x với mọi x ∈ R+.

2.3 Phương trình hàm - Dãy số 91

Tìm các hàm f : Q2 → Q thoả mãn

f(x, y) + f(y, z) + f(z, x) = f(0, x+ y + z) (2.58)

với mọi x, y, z ∈ Q.

Bài 11

Lời giải. (Đáp án tham khảo của Trần Quốc Nhật Hân từ Diễn đàn Toán họcVMF).

f : Q2 → Q : f (x, y) + f (y, z) + f (z, x) = f (0, x+ y + z) (2.59)

Thay x = y = z = 0 từ (2.58) ta có

f (0, 0) = 0

Thay y = z = 0 từ (2.58) ta có

f (x, 0) + f (0, x) = f (0, x)⇒ f (x, 0) = 0∀x (2.60)

Thay z = 0 từ (2.58) và (2.60) ta có:

f (x, y) + f (0, x) = f (0, x+ y) (2.61)

Theo (2.61) ta có

f (0, x+ (y + z)) = f (x, y + z) + f (0, x)

Tiếp tục từ (2.61) ta có:

f (0, (x+ y) + z) = f (x+ y, z) + f (0, x+ y)

= f (x+ y, z) + f (x, y) + f (0, x) (2.62)

Khi đó:f (x, y + z) = f (x+ y, z) + f (x, y) (2.63)

Thay z = y vào (2.63) ta được:

f (x, 2y) = f (x+ y, y) + f (x, y) (2.64)

92 HƯỚNG DẪN GIẢI

Thay z = 2y vào (2.63) ta được:

f (x, 3y) = f (x+ y, 2y) + f (x, y) (2.65)= f (x+ 2y, y) + f (x, y) + f (x, y) (2.66)= f (x+ 2y, y) + 2f (x, y) (2.67)

⇒ f (x, (n+ 1) y) = f (x+ ny, y) + nf (x, y)∀n ∈ N (2.68)

Sử dụng (2.63) ta có:

f (x, 4y) = f (x+ 3y, y) + f (x, 3y) (2.69)= f (x+ 3y, y) + f (x+ 2y, y) + 2f (x, y) (2.70)

Ngoài ra cũng từ (2.63) ta có:

f (x, 4y) = f (x+ 2y, 2y) + f (x, 2y) (2.71)= (f (x+ 2y + y, y) + f (x, y)) + (f (x+ y, y) + f (x, y)) (2.72)= f (x+ 3y, y) + f (x+ y, y) + 2f (x, y) (2.73)

Từ (2.70), (2.73) ta được

f (x+ 2y, y) = f (x+ y, y)⇒ f (x+ y, y) = f (x, y) , ∀x, y (2.74)

Khi đó từ (2.74) ta có:

f (x+ ny, y) = f (x, y)∀n ∈ N

Thay x bởi x− y vào (2.74) ta được:

f (x, y) = f (x− y, y)⇒ f (x− ny, y) = f (x, y)∀n ∈ N

⇒ f (x+ ny, y) = f (x, y)∀n ∈ Z. (2.75)

Sử dụng (2.68),(2.74) ta được:

f (x, ny) = nf (x, y)∀n ∈ N

Thay z = y và y bởi − vào (2.63) ta được:

f (x, 0) = f (x− y, y) + f (x,−y)⇒ f (x,−y) = −f (x− y, y)(2.75)

= −f (x, y)

⇒ f (x, ny) = nf (x, y)∀n ∈ Z

2.3 Phương trình hàm - Dãy số 93

Thay y bởiy

nta được:

f (x, y) = nf(x,y

n

)∀n ∈ Z⇒ f

(x,p

qy

)= pf

(x,y

q

)=p

qf (x, y)

⇒ f (x, zy) = zf (x, y)∀z (2.76)

⇒ f (x, y) = yf (x, 1)∀y (2.77)

Từ (2.63),(2.74) ta lại có:

f (x, y + z) = f (x, z) + f (x, y) , ∀x, y, z (2.78)

Thay y bởiy

mta vào (2.75) ta có:

f(x+

n

my,y

m

)= f

(x,y

m

)(2.76)⇒ 1

mf(x+

n

my, y)

=1

mf (x, y)

⇒ f (x+ zy, y) = f (x, y) , ∀zThay x = 0 và chọn z ∈ Q ta được:

f (zy, y) = f (0, y)(2.77)

= yf (0, 1)∀y, z ⇒ f (x, y) = ay∀x, y.

Vậy ta có f(x, y) = ay, ∀(x, y) ∈ Q2.

Nhận xét. Đây là bài khó nhất của đêm tiêu thụ bài giảng thứ hai phần Phươngtrình hàm và không có đoàn nào làm (!) Cách làm như những bài Phương trìnhhàm rời rạc một biến, tuy nhiên khối lượng công việc nhiều hơn hai lần như vậyvì phải xử lý hai biến và mối liên hệ.Một số bài toán tương tự:

1. (IMO Longlist 1967) Cho hàm số ϕ(x, y, z) được xác định bởi bộ ba các sốthực x, y, z, thoả mãn có hai hàm số f, g đều được xác định bởi các cặp sốthực thoả mãn:

ϕ(x, y, z) = f(x+ y, z) + g(x, y + z)

với mọi x, y, z ∈ R.

Chứng minh rằng tồn tại một hàm số đơn biến thực h sao cho

ϕ(x, y, z) = h(x+ y + z)

với mọi số thực x, y, z.

94 HƯỚNG DẪN GIẢI

2. (Olympiad các nước vùng Baltic 1998) Tìm tất cả các hàm số f : Z+×Z+ →Z+ thoả mãn các điều kiện sau với mọi số nguyên dương x, y:

(a) f(x, x) = x.

(b) f(x, y) = f(y, x).

(c) (x+ y)f(x, y) = yf(x, x+ y).

Tìm f : R⇒ R thoả:

f(x)f(yf(x)− 1) = x2f(y)− f(x), ∀x, y ∈ R

Bài 12

Lời giải. Thay x = 0 vào đề ta có:

f(0)f(yf(0)− 1) = −f(0)

Nếu f(0) 6= 0 thì ta có f(x) ≡ −1, thay vào đề thì vô lý. Vậy f(0) = 0. Khi đóthay y = 0 vào đề thì có:

f(x)f(−1) = −f(x), ∀x ∈ R

Suy ra f(−1) = −1.

Bây giờ ta sẽ chứng minhf(x) = 0⇔ x = 0 (2.79)

Chiều ngược đã đúng, ta sẽ chứng minh thêm: Nếu f(x) = 0 thì suy ra x = 0.

Với x thoả f(x) = 0 thì ta thay y = −1 vào đề ta có:

0 = −x2 ⇔ x = 0

Vậy ta có (2.79).

Bây giờ, thay x = y = 1 ta có;

f(1)f(f(1)− 1) = 0.

Do (2.79) nên ta có f(1) = 1. Thay tiếp x = 1 ta có:

f(y − 1) = f(y)− 1.

2.3 Phương trình hàm - Dãy số 95

Khi đó bài toán thànhf(x)f(yf(x)) = x2f(y) (2.80)

Thay x = y trong (2.80) ta có

f(x)f(f(x)) = x2, ∀x ∈ R (2.81)

Thay x bởi x− 1 trong (2.81) ta có:

[f(x)− 1][f(f(x))− 1] = (x− 1)2 ⇔ f(x) + f(f(x)) = 2x.

Thay vào (2.81) ta có f(x) = x, ∀x ∈ R. Thử lại thoả.

Vậy f(x) = x, ∀x ∈ R.

Nhận xét. Mấu chốt của bài toán là chứng minh (2.79). Mọi luaapj luận về sauđề sử dụng đến (2.79) để được giải quyết đơn giản hơn. Việc tính được f(x− 1) =f(x)− 1 cũng cho ta thấy hướng đi sẽ thay x bởi x− 1.

Cho f : Q2 → R+ với Q2 = {(x, y)|x, y ∈ Q}. Giả sử f thoả mãn các điềukiện sau:

1. f(a, b, c) = f(a, c)f(b, c), ∀a, b, c ∈ Q.

2. f(c, ab) = f(c, a)f(c, b), ∀a, b, c ∈ Q.

3. f(a, 1− a) = 1, ∀a ∈ Q.

Chứng minh rằng f(a, a) = f(a,−a) = 1 và f(a, b)f(b, a) = 1.

Bài 13

Lời giải. Lần lượt thay b = 1 vào điều kiện (a) và (b) ta có:

f(a, c) = f(a, c)f(1, c) ⇔ f(1, c) = 1 (2.82)

f(c, a) = f(c, a)f(c, 1) ⇔ f(c, 1) = 1 (2.83)

(do tập xác định của hàm f là R+).

Thay a = b = −1 lần lượt vào điều kiện (1) và (2) ta có:

f(−1, c)2 = f(1, c) = 1 ⇔ f(−1, c) = 1 (2.84)

96 HƯỚNG DẪN GIẢI

f(c,−1)2 = f(c, 1) = 1 ⇔ f(c,−1) = 1 (2.85)

Thay a bởi 1− 1

avào điều kiện (3) ta có:

f

(1− 1

a,

1

a

)= 1 (2.86)

Thay c = 1− 1

avào (2.83) ta có:

f

(1− 1

a, 1

)= 1

Khi đó

f

(1− 1

a, a

)f

(1− 1

a,

1

a

)= f

(1− 1

a, 1

)= 1

Do điều kiện (c) nên ta có:

f

(1− 1

a

)= 1 (2.87)

Từ đó

f

(1− 1

a

)= f(1− a, a)f

(−1

a, a

)

⇒ f

(−1

a, a

)= 1

Khi đó ta có:1 = f(1, a) = f

(−1

a, a

)f (−a, a)⇔ f(−a, a) = 1, theo (2.82)

1 = f(−1, a) = f

(−1

a, a

)f (a, a)⇔ f(a, a) = 1, theo (2.84)

Tiếp theo, do f(x, x) = 1, ∀x ∈ Q nên ta có:

f(a, b)f(b, a) = f(a, b)f(a, a)f(b, a)f(b, b) = f(a, ab)f(b, ab)

= f(ab, ab) = 1

Vậy ta có 3 đẳng thức cần chứng minh.

2.3 Phương trình hàm - Dãy số 97

Nhận xét. Nếu nhìn nhận theo phương trình hàm một biến thì ta có thể thấyrằng, điều kiện (1) cho ta tính nhân tính trái và điều kiện số (2) cho ta tính nhântính phải và điều đầu tiên cần chứng minh chính là chứng minh hàm số chẵn tráinhưng lẻ phải.Có một các tiếp cận để chứng minh đẳng thức cuối cùng, có phần tự nhiên hơnnhưng khi làm thì phức tạp hơn như sau:

Để chứng minh f(a, b)f(b, a) = 1 thì ta cần chứng minh f(b, a) = f

(1

a, b

)hoặc

f(b, a) = f

(a,

1

b

). Ta sẽ chứng minh điều đầu tiên. Ta dễ dàng có được

f

(a,−1

a

)= 1

f

(a,±1

a

)= 1 (2.88)

Sử dụng điều kiện (1) và (2) ta lần lượt có:

f(b, a)f

(1

b, a

)= 1⇒ f

(1

b, a

)=

1

f(b, a)

f

(1

b, a

)f

(1

b,

1

a

)= 1⇒ f

(1

b, a

)=

1

f

(1

b,

1

a

)hay

f(b, a) = f

(1

b,

1

a

).

Từ đó và (2.88) ta có

f(b, a) = f

(1

b,

1

a

)= f

(a

b,

1

a

)= f

(a

b,

1

a

)f(ab,a

b

)= f

(a

b,1

b

)= f

(a

b,1

b

)f

(b,

1

b

)= f

(a,

1

b

)Vậy ta có f(b, a) = f

(a,

1

b

)hay đã có điều cần chứng minh.

98 HƯỚNG DẪN GIẢI

Cho hàm số f : N? × N? → N? (với N? = N ∪ {0}) thoả mãn các điều kiệnsau:

1. f(0, x) = x+ 1

2. f(x+ 1, 0) = f(x, 1)

3. f(x+ 1, y + 1) = f(x, f(x+ 1, y))

Tính f(1, 2017), f(2, 2017), f(3, 2017), f(4, 2017).

Bài 14

Lời giải. Ta có:

f(1, n) = f(0, f(1, n− 1)) = f(1, n− 1) + 1.

Bằng quy nạp ta có được:

f(1, n) = f(1, 0) + n = n+ f(0, 1) = n+ 2

Hay f(1, 2017) = 2019.

Tiếp theo:f(2, n) = f(1, f(2, n− 1)) = f(2, n− 1) + 2.

Bằng quy nạp ta có

f(2, n) = 2n+ f(2, 0) = 2n+ f(1, n) = 2n+ 3.

Hay f(2, 2017) = 4037.

Tiếp theo như quy nạp cho x = 3 như sau:

f(3, n) = f(2, f(3, n− 1)) = 2f(3, n− 1) + 3.

với n ∈ N?

Nếu đặt an = f(3, n) thì ta tính được

a0 = f(3, 0) = f(2, 1) = 5

và dãy truy hồian = 2an−1 + 3

2.3 Phương trình hàm - Dãy số 99

Dễ dàng tìm được số hạng tổng quát là an = 2n+3 − 3 với a0 = 5. Vậy ta cóf(3, n) = an = 2n+3 − 3 hay

f(3, 2017) = 22020 − 3

Tiếp tục sử dụng kết quả trường hợp x = 3 cho x = 4 như sau:

f(4, n) = f(3, f(4, n− 1)) = 2f(4,n−1)+3 − 3.

Đặt an = f(4, n) thì ta có

a0 = f(4, 0) = f(3, 1) = 24 − 3 = 13

và dãy truy hồian + 3 = 2an−1+3

Sử dụng quy nạo ta có

an + 3 = 22...2a0+3︸ ︷︷ ︸

n lần mũ

= 22...216︸ ︷︷ ︸

nlnm

= 22...22︸ ︷︷ ︸

n+3 lần mũ

Vậy

f(4, n) = an = 22...22︸ ︷︷ ︸

n+3 lần mũ

−3.

Thay n = 2017 ta được kết quả là:

f(4, 2017) = 22...22︸ ︷︷ ︸

2020 lần mũ

−3.

Nhận xét. Đây là một bài toán rất nỗi tiếng về hàm nhiều biến, có tên gọi làPhương trình hàm Ackermann, thuộc lớp hàm có thể tính toán được. Trường hợpx = 4 cũng chính là câu 6 đề IMO 1981. Phương trình hàm Ackermann có thể mởrộng cho dạng ba biến nhưng vì khối lượng tính toán quá lớn nên phương trìnhhàm này rất ít được sử dụng cả trong lý thuyết và ứng dụng.

100 HƯỚNG DẪN GIẢI

Cho dãy số (xn) được xác định bởi:{x1 = 4

xn+1 = x2n − 2,∀ ∈ N∗

Tínhlim

n→+∞

xn+1

x1x2x3...xn

Bài 15

Lời giải. Bằng quy nạp ta chứng minh được xn ≥ 4,∀n ∈ N∗,Mặt khác:

x2n+1−4 = (xn−2)2−4 = x2n(x2n−4) = . . . = (x1x2. . . xn)2(x21−4) = 12(x1x2. . . xn)

Suy ra: (xn+1

x1x2..xn

)2

= 12 +4

(x1x2. . . xn)2

Lại có:xn+1 − xn = x2n − xn − 2 = (xn + 1)(xn − 2) > 0,∀n ∈ N∗

Do đó: (xn) là dãy tăng⇒ xn ≥ 4,∀n ∈ N∗.Khi đó: 0 ≤ 4

x1x2..xn≤ 1

42n−1 .

Mà limn→+∞1

42n−1 = 0 nên theo định lí kẹp, ta có:

limn→+∞

4

x1x2..xn= 0

Vậy nên:

limn→+∞

xn+1

x1x2..xn= lim

n→+∞

√12 +

4

(x1.x2...xn)2= 2√

3

Nhận xét. Ý tưởng bài toán trên hoàn toàn giống với một bài toán dãy số trongkì thi Olympic toán sinh viên toàn quốc năm 2005, chỉ có điều sự khác biệt ở đâylà giá trị x1 = 5.Ý tưởng của bài toán ấy còn được xuất hiện một lần nữa trong

2.3 Phương trình hàm - Dãy số 101

kì thi Olympic toán sinh viên quốc tế năm 2010,với nội dung như sau: Cho dãy số(xn) được xác định bởi hệ thức truy hồi :{

x1 =√

5xn+1 = x2n − 2,∀n ∈ N∗

Tìm limn→+∞x1x2...xnxn+1

.

Bài tập bổ sung: Cho dãy số

(xn) :

{x0 = a

xn+1 = 2x2n − 1, ∀n ≥ 0

Tìm các giá trị của a để xn < 0,∀n ≥ 0.

Cho dãy số (un) được xác định bởi công thức truy hồi : u1 = α ∈ R

un+1 =u3n + 9un − 6

3u2n − 6un + 7,∀α ∈ N∗

Tìm α để dãy số (un) có giới hạn hữu hạn khi và tìm giới hạn của dãy sốtrong các trường hợp đó.

Bài 16

Lời giải. Nếu α = −1 thì un = −1∀n ∈ N∗.Xét α 6= −1. Khi đó, ta có:

un + 1 =u3n−1 + 9un−1 − 6

3u2n−1 − 6un−1 + 7+ 1

=u3n−1 + 3u2n−1 + 3un−1 + 1

3u2n−1 − 6un−1 + 7=

(un−1 + 1)3

3u2n−1 − 6un−1 + 7

Lại có:

un − 3 =u3n−1 + 9un−1 − 6

3u2n−1 − 6un−1 + 7− 3

=u3n − 9u2n−1 + 27un−1 − 27

3u2n−1 − 6un−1 + 7=

(un−1˘3)3

3u2n−1 − 6un−1 + 7

102 HƯỚNG DẪN GIẢI

Xét hàm số f(x) =x− 3

x+ 1. Ta có:

f(xn) =un − 3

un + 1= +

(un−1 − 3

un−1 + 1

)3

= [f(xn−1)]3 = [f(xn−2)]

32 = ... = [f(u1)]3n−1

= [f(α)]3n−1

Đặtβ = [f(α)]3

n−1

Từ (*) ta có:

un − 3

un + 1= βn ⇔ un − 3 = βnun + βn ⇔ un =

3 + βn1− βn

Vậy:

un =

3 +

(α− 3

α + 1

)3n−1

1−(α− 3

α + 1

)3n−1

Biện luận:α− 3

α + 1= 1⇔ α− 3 = α + 1⇔ 0α = 4

(vô nghiệm)∣∣∣∣α− 3

α + 1

∣∣∣∣ < 1⇔ (α− 3)2 < (α + 1)2 ⇔ −6α + 9 < 2α + 1⇔ α > 1

∣∣∣∣α− 3

α + 1

∣∣∣∣ > 1⇔ −1 6= α < 1

Vậy:

• Nếu α = −1 thì limn→+∞ un = −1.

• Nếu α = 1 thì un = 1∀n ∈ N∗ suy ra limn→+∞ un = 1.

• Nếu α > 1 thì

limn→+∞

(α− 3

α + 1

)3n−1

⇒ limn→+∞

= 3.

2.3 Phương trình hàm - Dãy số 103

• Nếu −1 6= α < 1 thì

limn→+∞

(α− 3

α + 1

)−3n−1

= 0⇒ limn→+∞

un = −1.

Cho hai dãy số (an), (bn) được xác định bởi hệ thức truy hồi:

a1 = 3, b1 = 2, an+1 = a2n + 2b2n, bn+1 = 2anbn,∀n ∈ N∗

Tìmlim

n→+∞2n√bn; lim

n→+∞2n√a1a2...an

Bài 17

Lời giải. Ta có:

an + bn√

2 = (an−1 +√

2bn−1)2 = (an−2 +

√2bn−2)

22 = . . . = (a1 +√

2b1)2n−1

= (3 + 2√

2)2n−1 = (

√2 + 1)2

n

,∀n ∈ N∗

Tương tự, ta có:an − bn

√2 = (

√2− 1)2

n

,∀n ∈ N∗

Từ hai điều trên suy ra:an =

1

2[(√

2 + 1)2n

+ (√

2− 1)2n]

bn =1

2√

2[(√

2 + 1)2n − (

√2− 1)2

n]

Mặt khác:

2n

√(√

2 + 1)2n

4√

2< 2n√bn < 2n

√an <

√2 + 1

limn→+∞

2n

√(√

2 + 1)2n

4√

2

104 HƯỚNG DẪN GIẢI

= limx→+∞

√2 + 1

(4√

2)

1

2n

=√

2 + 1

Theo định lí kẹp ta suy ra:

limx→+∞

2n√bn = lim

n→+∞2n√an =

√2 + 1

Mặt khác:

a1a2..a3 =b22b1

b32b2

...bn+1

2n+1=anbn2n

Do đó:

limx→+∞

2n√a1a2...an = lim

x→+∞

2n

√anbn2n

= limn→+∞

2n√an.

2n√bn.

2n

√1

2n= (√

2 + 1)2

= 3 + 2√

2

limn→+∞

2n

√1

2n= lim

n→+∞

(1

2n

) 1

2n = 1

Nhận xét. Không khó để nhận ra được việc tìm số hạng tổng quá là cần thiết, đặcbiệt sự xuất hiện của a2n + 2b2n và 2anbn có thể tạo ra được hằng đẳng thức. Saukhi tìm được số hạng tổng quát thì việc tính limn→+∞ 2n

√an và limn→+∞

2n√bn có

nhiều hướng đi, trong đó sử dụng định lý kẹp là hữu hiệu nhất.

Cho dãy số (xn) được xác định bởi hệ thức truy hồi: x1 = a ≥ 1

xn+1 =x2n − 2xn

2

[xn]2,∀n ∈ N∗

Chứng minh dãy số (xn) đã cho có giới hạn hữu hạn khi n→ +∞.

Bài 18

2.3 Phương trình hàm - Dãy số 105

Lời giải. Xét các trường hợp:

TH1: a là số nguyên. Khi đó: [a] = a; a = 0. Do đó:

x1 = a, x2 =x21 − 2x1

2

[x1]2=a2

a2= 1

Bằng quy nạp ta chứng minh được: xn = 1,∀n ∈ N∗.Do đó: limx→+∞ xn = 1.

TH2: a không phải là số nguyên. Khi đó:

x2 =a2 − 2x1

2

[a]2=

([a] + a)2 − 2a2

[a]2(2.89)

=2[a]2 − ([a]− a)2

[a]2= 2−

([a]− a

[a]

)2

= 2−(

1− a

[a]

)2

(2.90)

Vì a không phải số nguyên nên 0 < a < 1. Mặt khác a ≥ 1 nên [a] ≥ 1. Từđó ta có: 0 <

a

[a]< 1.

Kết hợp với (2.90) ta có: 1 < x2 < 2.

Bằng quy nạp ta chứng minh được:1 < xn < 2,∀n ∈ N∗.Do 1 < xn < 2 nên [xn] = 1. Suy ra:

xn = xn − [xn] = xn − 1,∀n ∈ N∗

Vì thế:xn+1 = x2n − 2(xn − 1)2 = −x2n + 4xn − 2,∀n ∈ N∗

Từ đó ta có:

2− xn+1 = (2− xn)2 = (2− xn−1)22

= ... = (2− x2)2n−1

Do đó:lim

n→+∞(2− xn+1) = lim

n→+∞(2− x2)2

n−1

= 0

(Vì 0 < 2− x2 < 1)

Vậy:

• Nếu a ∈ Z+ thì limn→+∞ xn = 1.

106 HƯỚNG DẪN GIẢI

• Nếu a /∈ Z, a ≥ 1 thì limn→+∞ xn = 2.

Nhận xét. Một số bài toán dãy số liên quan đến phần nguyên sau sẽ giúp các bạntích lũy nhiều phương pháp để giải quyết dạng này tốt hơn:

1. Cho dãy {xn} xác định như sau: x1 = 1

xn+1 =

[3

2xn

], ∀n ≥ 1.

Chứng minh rằng dãy {xn} có vô hạn các số chẵn, có vô hạn các số lẻ.

2. Tìm công thức của xn biết x0 = 1 ,

xn+1 = xn +[xn√

5].

Tìm tất cả các hàm số f : R→ R thoả mãn:

f(yf(x+ y) + f(x)) = 4x+ 2yf(x+ y), ∀x, y ∈ R

Bài 19

Lời giải. Thay y = 0 ta có f(f(x)) = 4x, suy ra f song ánh. Khi đó tồn tại a đểf(a) = 0. Khi đó

f(f(0)) = 0 = f(a)⇒ a = f(0)

do f song ánh.

Mà lại có 4a = f(f(a)) = f(0) = a vậy suy ra a = 0 hay f(0) = 0.

Thay x = 0 và y =1

2ta có:

f

(1

2f

(1

2

))= f

(1

2

)

Do f song ánh nên f(

1

2

)= 1. Mà ta lại có f(1) = f

(f

(1

2

))= 4.

1

2= 2 nên

f(2) = f(f(1)) = 4

2.3 Phương trình hàm - Dãy số 107

Thay y = 1− x vào đề ta có:

f((1− x)f(1) + f(x)) = 4x+ 2(1− x)f(1) = 4x+ 4− 4x = 4 = f(2)

Do f đơn ánh và thu gọi vế trái phương trình trên ta có:

2− 2x+ f(x) = 2⇔ f(x) = 2x

Thử lại thoả. Vậy f(x) = 2x, ∀x ∈ R.

Đặt I = [0, 1] và G = {(x, y)|x, y ∈ I}. Tìm tất cả các hàm số f : G → Ithoả mãn với mọi x, y, z ∈ I ta có:

1. f(f(x, y), z) = f(x, f(y, z)),

2. f(x, 1) = x, f(1, y) = y,

3. f(zx, zy) = zkf(x, y).

với k là một số thực dương không liên quan đến x, y, z.

Bài 20

Lời giải. Ta sẽ xét hai trường hợp:

• Nếu x ≤ y thì ta có

f(x, y) = f

(x

y, 1

)yk = xyk−1

• Nếu y ≤ x thì ta có

f(x, y) = f(

1,y

x

)xk = xk−1y

Cả hai trường hợp trên đều sử dụng tính chất (3) và (2).

Từ tính chất (1) và 2 kết quả trên, chọn z đủ bé nhưng khác 0 và x ≥ y ta có:

f(f(x, y), z) = f(xk−1y, z) = (xk−1y)k−1z = x(k−1)2

yk−1z

108 HƯỚNG DẪN GIẢI

f(f(x, y), z) = f(x, f(y, z)) = f(x, yk−1z = xk−1yk−1z

So sánh hay phương trình trên ta có:

(k − 1)2 = k − 1⇔[k = 1k = 2

• Nếu k = 1 thì ta có f(x, y) = min{x, y}.

• Nếu k = 2 thì ta có f(x, y) = xy.

Nhận xét. Đây thực chất là đề Trung Quốc 1991. Do tính chất mới lạ nên mìnhgiới thiệu vào cho những ai quan tâm có cơ hội tìm hiểu. Về bài toán, có một sốđiều sau:

• Nếu xem cách tác động f vào như phép nhân thì ta có tính chất (1) của hàmf chính là phép liên hợp ((x.y).z = x.(y.z)). Tính chất số 2 có thể xem nhưlà điều kiện biên của bài toán. Tính chất 3 chính là tính thuần nhất (hay tacó thể gọi f là Hàm số thuần nhất, Homogeneous function).

• Bài toán này lại được sử dụng lại vào năm 2004 trong đề chọn đội tuyểnBulgari, câu 1, ngày 4 như sau: Tìm tất cả các hàm số f : [0, 1] × [0, 1] →[0, 1] thoả mãn các điều kiện sau với mọi k > 0:

1. f(f(x, y), z) = f(x, f(y, z)),

2. f(x, y) = f(y, x),

3. f(x, 1) = x,

4. f(zx, zy) = zkf(x, y).

với mọi x, y, z ∈ [0, 1].

Bài toán có kết quả như bài Trung Quốc 1991, phần lời giải xin được phépmời bạn đọc suy nghĩ.

2.4 Số học 109

2.4 Số học

Biết rằng với dãy nguyên dương 1 < k1 < k2 < ... < kn và dãy nguyên tươngứng s1, s2, ..., sn, với mọi số nguyên dương N đều tồn tại i ∈ {1, 2, ..., n} saocho N ≡ si (mod ki).

1. Tìm dãy {kn} và {sn} thoả mãn khi k1 = 2 và khi k1 = 3.

2. Chứng minh rằng∑n

i=1

1

ki> 1.

3. Tìm n nhỏ nhất để có các dãy TMĐK đề bài.

Bài 1

Lời giải.

1. • Ví dụ 1:

i 1 2 3 4 5k 2 3 4 6 12s 1 0 0 4 2

• Ví dụ 2:

i 1 2 3 4 5 6 7 8 9 10 11 12k 3 4 6 8 9 12 16 18 24 36 48 72s 1 0 5 6 0 2 7 3 18 33 15 51

2. ChọnN là bội chung của k1, k2, . . . , kn. Ta thấy rằng trong tập {1; 2; . . . ;N}có

N

kisố đồng dư với si theo (mod k)i. Do đó ta có:

n0∑i=1

N

ki≥ N ⇔

n0∑i=1

1

ki≥ 1.

3. Ta sẽ chứng minh n ≥ 5. Giả sử tồn tại n0 ≤ 4 là số nhỏ nhất để tồn tại 2dãy số nguyên thoả mãn đề bài.

Nếu k1 ≥ 3 thì∑n0

i=1

1

ki≤ 1

3+

1

4+

1

5+

1

6< 1. Vì thế k1 = 2.

Ta cũng có nhận xét là n0 ≥ 3. Ta có 2 trường hợp sau:

110 HƯỚNG DẪN GIẢI

TH1: s1 = 1. Còn lại các số chẵn. Khi đó xét số nguyên dương N chẵn bấtkì:

• Nếu ki chẵn thìN

2≡ si

2(mod

ki2

).

• Nếu ki lẻ thìN

2≡ si.2

−1 (mod ki) với 2−1 là phần tử nghịch đảo

của 2 theo (mod ki). Khi đó với i = 2, n0 tồn tại bộ

k′i−1 =

ki2

nếu ki chẵn

ki nếu ki lẻ

Nếu bộ số trên là phân biệt thì với mọi số nguyên N tồn tại bộn0 − 1 số k′i và s′i tương ứng thoả mãn đề bài. Mâu thuẫn với giảthiết n0 là số nhỏ nhất.Vì vậy k′1, k

′2, k′n0−1 có đúng hai số bằng nhau.

Suy ra (k2, . . . , kn0) = (2m+ 1, 4m+ 2, k) (k nếu có).

– Với k lẻ ta có (k′1, k′2, k′n0−1) = (2m+ 1, 2m+ 1, k) từ đó có

2

2m+ 1+

1

k≥ 1.

Do đó m = 1 và k = 3 = 2m+ 1 vô lý.– Với k = 2t với t ≥ 2 ta có (k′1, k

′2, k′n0−1) = (2m + 1, 2m + 1, t)

từ đó có2

2m+ 1+

1

t≥ 1.

Do đó m = 1 và t = 2 hoặc t = 3. Khi t = 3 ta có k = 6 =4m+ 1 vô lý, nên t = 2 và k = 4.Ta có (k′1, k

′2, k′3) = (2, 3, 3). Rõ ràng khi phân hoạch tập số

nguyên theo mod3 thì mỗi tập đều có số lẻ và số chẵn, nênbộ nêu trên không thoả mãn.

TH2: s1 = 0. Còn các số lẻ. Ta xét tập các số lẻ trừ đi 1 và bộ (si − 1)tương ứng. Ta lại có toàn các số chẵn như trường hợp 1.

Vậy n nhỏ nhất là 5.

Nhận xét. Bài toán trên tuy nhìn đề khá khó nhưng chỉ sử dụng một vài kĩ thuậtvà nhận xét cơ bản. Câu a. là ví dụ cũng như tiền đề để có ý tưởng của các câu b.

2.4 Số học 111

và c.. Để có các ví dụ này, chúng ta không thể chỉ nâng mod theo lũy thừa của 2vì khi đó sẽ luôn dư ra ít nhất một số dư. Chú ý: 2, 3, 4, 6, 12 đều là ước của 12.Quan sát này có gợi cho bạn điều gì không? Khi nhận thấy được quy luật của câua. thì kĩ thuật xuống thang của câu c là khá rõ ràng.

Tìm n ∈ N và n > 1 sao cho 3n − 1... n3.

Bài 2

Lời giải.

• Gọi p là ước nguyên tố nhỏ nhất của n thì 3n ≡ 1 (mod p). Mà 3p−1 ≡ 1

(mod p) nên (n; p− 1)... ordp(3). Do đó ordp(3) = 1 do cách chọn p. Vì thế

3 ≡ 1 (mod p)⇒ p = 2.

• Do n chẵn nên v2(3n−1) = v2(3−1) + v2(3 + 1) + v2(n)−1 = v2(n) + 2.Màv2(3

n − 1) ≥ 3v2(n) nên v2(n) + 2 ≥ 3v2(n)⇒ v2(n) = 1.

• Đặt n = 2m với m lẻ. Nếu m > 1 thì gọi q là ước nguyên tố nhỏ nhất của

m. Khi đó (2m; q−1)... ordq(3). Từ đó ordq(3) = 1 hoặc ordq(3) = 2. Nhưng

cả 2 trường hợp đều suy ra q = 2 vô lý.Khi đó m = 1 và n = 2. Thử lại đúng.

Vậy n = 2.

Nhận xét. Đây là một bài toán cơ bản của việc áp dụng bổ đề nâng lũy thừaLTE. Công thức được áp dụng hoàn toàn có thể được chứng minh bằng cách đặtn = 2mk với k lẻ và phân tích 3n − 1 thành nhân tử. Thay 3 bằng một số khác,trong một số trường hợp ta có bài toán tương tự.

Chứng minh rằng tồn tại vô số số nguyên tố dạng 2nk + 1 với k nguyêndương và n ≥ 2017.

Bài 3

Hướng dẫn. Ta chứng minh 2 ý sau:

112 HƯỚNG DẪN GIẢI

• Tập hợp ước nguyên tố của các số có dạng 22k + 1 là vô hạn. Để có điềuđó ta sẽ chứng minh (22n + 1; 22m + 1) = 1 với mọi m 6= n.Thật vậy cho m > n và p là ước nguyên tố của 22n +1 thì 22m ≡ (22n)2

m−n ≡1 (mod p) suy ra 22m + 1 6 ... p.

• Nếu p là ước nguyên tố của 22n + 1 thì p... 2n+1. Ta dễ dàng có điều này

bằng cách sử dụng cấp.

Nhận xét. Đây là một bài toán áp dụng tính chất của cấp khá cơ bản. Tuy nhiênviệc chỉ ra tập ước nguyên tố của 22k + 1 là vô hạn là rất quan trọng.

Tìm số nguyên dương n nhỏ nhất sao cho 1n + 2n + . . .+ 2016n 6 ... 2017.

Bài 4

Lời giải. Ta chứng minh bài toán tổng quát sau:

Bài toán. Cho p là số nguyên tố lẻ. Chứng minh rằng 1k + 2k + ... + (p − 1)k... p

với mọi k 6 ... p− 1.

Thật vậy, mỗi số nguyên tố p có ít nhất một căn nguyên thuỷ m. Khi đó,{m;m2; . . . ;mp−1} ≡ {1; 2; . . . ; p− 1}. Vì thế

1k + 2k + ...+ (p− 1)k = mk + (m2)k + . . .+ (mp−1)k = mk.(mk)p−1

mk − 1

Do m là căn nguyên thuỷ của p và k 6 ... p−1 nên mk−1 6 ... p, lại có (mk)p−1−1... p.

Suy ra điều phải chứng minh.Từ bài toán trên suy ra số n nhỏ nhất cần tìm là 2016.

Nhận xét. Bài toán trên được giải khá gọn gàng bằng cách sử dụng căn nguyênthủy, tuy nhiên bạn đọc hãy tự tìm hiểu vấn đề ở mở đầu bài giải: "Mỗi số nguyêntố đều có ít nhất một căn nguyên thủy". Đồng thời, bạn hãy thử tìm thêm cáchchứng minh khác bằng quy nạp.

2.4 Số học 113

Cho 9 số nguyên dương phân biệt d1, d2, . . . , d9 và đa thức P (x) = (x +d1)(x + d2) . . . (x + d9). Chứng minh rằng có số N nguyên dương sao cho∀x ≥ N thì P (x) có ước nguyên tố lớn hơn 20.

Bài 5

Lời giải. Gọi d = max{d1, d2, . . . , d8}. Chọn N = d8.Giả sử mệnh đề ở đề bài là sai.Có 8 số nguyên tố nhỏ hơn 20. Khi x > N phân tích P (x) theo 8 số nguyên tốđầu tiên:

P (x) = 2a1 .3a2 .5a3 .7a4 .11a5 .13a6 .17a7 .19a8

Do đó x+ di = Π8j=1p

αi,j

j .Mà x+ di > d8 ⇒ x+ di có ước pαi,1

1 > d.Có 9 số x+ di nên theo định lý Dirichlet tồn tại i 6= j:

x+ di... pαi,1

1 > d

x+ dj... pαj,1

1 > d

Chọn α = min{αi,1;αj,1} thì x+di... pα1 và x+dj

... pα1 . Từ đó suy ra |di−dj|... pα1 ⇒

|di − dj| > d vô lý.Vậy ta có đpcm.

Nhận xét. Nhớ rằng số các số nguyên tố nhỏ hơn 20 là 8 số. Trong đề bài lại có 9nhân tử của P (x), điều này có gợi cho bạn nhớ tới định lý Dirichlet? Sau khi có ýtưởng thì cuối cùng chỉ là việc chọn sao cho phù hợp.

Cho P (x) ∈ Z[x]. Biết a1, a2, . . . , an là các số nguyên thoả mãn: ∀x ∈ Z, ∃i ∈{1; 2; . . . ;n} sao cho P (x)

... ai. Chứng minh rằng ∃j : P (x)... aj ∀x ∈ Z.

Bài 6

Lời giải. Ta chứng minh bằng phản chứng.

Giả sử ∀i ∈ {1; 2; . . . ;n},∃x ∈ Z : P (x) 6 ... ai.Ta có P (x+ tpi) ≡ P (x)(mod p)∀x ∈ Z, t ∈ Z, p nguyên tố. Với mỗi số ai ta lấy 1ước nguyên tố, lấy lần lượt sao cho được tập {p1, p2, . . . , pk} gồm các số nguyên

114 HƯỚNG DẪN GIẢI

tố phân biệt.Theo giả sử phản chứng, ta đặt:

P (xi) 6... pi, i = 1, k

Và chọn được số x sao cho x ≡ xi (mod p)i theo định lí thặng sư Trung Hoa.

Khi đó P (x) 6 ... pi ∀i = 1, k nên P (x) 6 ... ai ∀i = 1, k, trái giả thiết.Vậy ta có đpcm.

Nhận xét. Điều phải chứng minh dễ dàng dẫn chúng ta tới ý tưởng phản chứng.Nhớ rằng chúng ta có công cụ chọn số khá mạnh trong tay là định lý thặng dưTrung Hoa, sau đó chọn thế nào cho đơn giản và phù hợp.

Tìm các số nguyên dương n sao cho với mọi số nguyên dương k, tồn tại sốtự nhiên a sao cho a3 + a− k chia hết cho n.

Bài 7

Lời giải. Bài toán tương đương với việc tìm n sao cho {a3 + a | a = 1, 2, . . . , n}là hệ thặng dư đầy đủ modulo n.Gọi p là một ước nguyên tố của n, ta sẽ chứng minh p = 3.Do {a3 + a | i = 1, 2, . . . , n} là hệ thặng dư đầy đủ modulo n nên {a3 + a | i =1, 2, . . . , p− 1} là hệ thặng dư thu gọn modulo p. Từ đó:

(13 + 1)(23 + 2) . . . ((p− 1)3 + (p− 1)) ≡ −1 (mod p).

Suy ra(12 + 1)(22 + 1) . . . ((p− 1)2 + 1) ≡ 1 (mod p)

Với p nguyên tố ta có:

• (12 + 1)(22 + 1) . . . ((p− 1)2 + 1) ≡ 0 (mod p) với p ≡ 1 (mod 4).

• (12 + 1)(22 + 1) . . . ((p− 1)2 + 1) ≡ 4 (mod p) với p ≡ 3 (mod 4).

(Chứng minh điều trên bằng hệ thặng dư hoặc bằng định lý Lagrange cho sốhọc)Do đó p chỉ có thể bằng 3.Ta chứng minh các số n có dạng 3k đều thoả mãn bằng quy nạp.

• k = 1 thì mệnh đề đúng.

2.4 Số học 115

• Giả sử mệnh đề đúng đến k, ta chứng minh nó đúng với k + 1. Thật vậyta có {i3 + i | i = 1, 2, . . . , 3k} là hệ thặng dư đầy đủ modulo3k, cần chứngminh {i3 + i | i = 1, 2, . . . , 3k+1} là hệ thặng dư đầy đủ modulo3k+1.Ta viết i3 + i = 3k.qi + ri với mỗi i = 1, 2, . . . , 3k (ri < 3k) và i = 3k + ti vớimỗi 3k < i ≤ 2.3k, i = 2.3k + ti với mỗi 2.3k < i ≤ 3k+1.

– Với 1 ≤ i ≤ 3k ta có:Nếu qi

... 3 thì i3 + i ≡ ri (mod 3k+1).Nếu qi ≡ 1 (mod 3) thì i3 + i ≡ 3k + ri (mod 3k+1).Nếu qi ≡ 2 (mod 3) thì i3 + i ≡ 2.3k + ri (mod 3k+1).

– Với 3k < i ≤ 2.3k ta có:

i3 + i ≡ t3i + ti + 3k (mod 3k+1)

với ti = 1, 2, . . . , 3k.

– Với 2.3k < i ≤ 3k+1 ta có:

i3 + i ≡ t3i + ti + 2.3k (mod 3k+1)

với ti = 1, 2, . . . , 3k.

Từ các điều trên, kết hợp với giả thiết quy nạp ta có được đpcm.

Nhận xét. Bài toán có sử dụng định lý Lagrange trong số học: Với đa thức P (x)bậc n, phương trình P (x) ≡ a (mod p) có không quá n nghiệm trên modp.

Tìm đa thức P (x) hệ số nguyên biết với mọi số nguyên tố p, a, b nguyêndương thì ab ≡ 1 (mod p)⇒ P (a).P (b) ≡ 1 (mod p).

Bài 8

Lời giải.

• Đặt degP = n và Q(x) = xnP (1

x) thì Q(x) ∈ Z[x]. Kí hiệu a−1 là số nguyên

để a.a−1 ≡ 1 (mod p) thì P (a)P (a−1) ≡ 1 (mod p). Từ đó anP (a)P (a−1) ≡an (mod p).

116 HƯỚNG DẪN GIẢI

• an.P (a−1) ≡ an.P (1

a) ≡ Q(a) (mod p). Do đó P (a)Q(a) ≡ an (mod p) với

mọi a sao cho (a, p) = 1.

• Từ đó theo định lý Lagrange thì P (x).Q(x)−xn chia hết cho mọi số nguyêntố p nên P (x)Q(x) = xn. Mà deqP = n nên P (x) = xn, Q(x) = 1 hoặcP (x) = −xn, Q(x) = −1.

Nhận xét. Bài toán có sử dụng định lý Lagrange trong số học và phần tử nghịchđảo: Với mỗi số nguyên tố p và số nguyên a không chia hết cho p, tồn tại duy nhấtsố a−1 sao cho a.a−1 ≡ 1 (mod p).

An và Bảo cùng nhau chơi một trò chơi: họ lần lượt viết các số tuỳ thích lênbảng thành một dòng, mỗi người 3 số, An viết trước. Sau đó Bảo "nhường"An điền dấu + hoặc − tuỳ ý vào giữa các số đã viết. An thắng nếu kết quảtrên bảng không chia hết cho bất cứ số tự nhiên nào từ 11 đến 18. Bảothắng nếu xảy ra trường hợp ngược lại. An nói rằng mình kiểm soát nhiềuhơn, nên chắc chắn chiến thắng. Bạn có đồng ý không? Tại sao?

Bài 9

Lời giải. Bảo mới là người có chiến thuật thắng.Thật vậy, ở 2 lượt đầu Bảo điền số 11.12.13 . . . 18. Ta sẽ chứng minh Bảo có cáchđiền ở lượt cuối cùng để đảm bảo thắng.Gọi số Bảo cần điền ở lượt cuối là x.Sau lượt đi thứ ba của An, bỏ qua các số Bảo đã điền (do chúng chia hết cho11, 12, 13, . . . , 18), ta có 8 kết quả có thể thu được (do có 23 = 8 cách đặt dấu)là a1, a2, . . . , a8.Do các số này cùng tính chẵn, lẻ nên chỉ có 2 loại số dư khi chia 8 số này cho 4.Hơn thế, có đúng 4 số đồng dư với nhau theo mod 4.Có 3 số đồng dư với nhau theo (mod 3) theo nguyên lý Dirichlet, giả sử làa1 ≡ a2 ≡ a3 (mod 3). Không giảm tổng quát giả sử a4 ≡ a3 (mod 4).Đến đây ta chỉ cần chứng minh tồn tại x sao cho a1 + x, a2 + x, . . . , a8 + x chiahết cho ít nhất một trong các số 11, 12, 13, . . . , 18. Khi đó bài toán được chứngminh do với mỗi i luôn tồn tại j sao cho ai − x = −(aj + x).

Theo định lý Thặng dư Trung Hoa, ta chọn được: a1 + x... 9; a2 + x

... 5; a4 +

x... 16; a5 + x

... 11; a6 + x... 13; a7 + x

... 17; a8+,... 7.

2.4 Số học 117

Khi đó a1+x... 18; a2+x

... 15; a3+x... 12; a4+x

... 16; a5+x... 11; a6+x

... 13; a7+

x... 17; a8 + x

... 14.

Nhận xét. Bài toán trên là một sự kết hợp thú vị giữa số học và tổ hợp. Lưu ýrằng các số từ 11 đến 18 có tổng cộng 7 ước nguyên tố, nên mục đích của việc chọnsố sẽ là làm cho các ước số nguyên tố đó xuất hiện.

Giả sử N∗ phân hoạch thành 3 dãy tăng {an}, {bn}, {cn} thoả mãn:

i. can = bn + 1

ii. an+1 > bn

iii. cn+1cn − (n+ 1)cn+1 − ncn chẵn.

Chứng minh rằng an = n2.

Bài 10

Lời giải. Ta chứng minh được những điều sau:

• cn ≥ n.

• an < bn < can < an+1 < bn+1

• Trong đoạn [1; 2; 3; . . . ; c(an)] có:

n số hạng của dãy {an},n số hạng của dãy {bn},an số hạng của dãy {cn}.

Từ đó có can = an + 2n.Với 1 vài biến đổi từ iii. ta có can−1, can , can+1 cùng tính chẵn, lẻ. Khi đóthấy được can−1, bn, can , an+1, can+1 là các số liên tiếp. Vì thế can = an+1−1.Từ i, ii và iii, ta xét c1 = 1 hay b1 = 1 đều không được, suy ra a1 = 1.Như vậy theo quy nạp ta có an = n2.

Nhận xét. Sau khi đã nhận ra được một vài tính chất của dãy số thì điều phảichứng minh là khá rõ ràng.

118 HƯỚNG DẪN GIẢI

Tìm các đa thức thoả điều kiện: a2 − b2 ∈ Q thì P (a)− P (b) ∈ Q.

Bài 11

Lời giải. Không giảm tổng quát, giả sử P (0) = 0.

• Với mọi số nguyên dương n thì (√n)2 ∈ Z+ ⇒ P (

√n) ∈ Q. Chứng minh

tương tự ta có P (−√n) ∈ Q.

• Đặt P (x) = Q(x2) + xH(x2). Do có vô số x mà P (x) hữu tỉ nên tất cả hệsố của P (x) đều hữu tỉ, suy ra tất cả hệ số của Q(x) và H(x) đều hữu tỉ.Khi đó cho x =

√n với n nguyên và không là số chính phương ta suy ra

H(n) = 0 với vô số n. Vậy thì H(x) = 0 ∀x ∈ R.

• Từ điều trên ta có P (x) = Q(x2) ∀x ∈ R.Lấy x > 0 bất kì có (

√x+ 1)2 − (

√x)2 ∈ Z nên P (

√x+ 1) − P (

√x) =

Q(x+ 1)−Q(x) ∈ Q.Nếu deg(Q(x + 1) − Q(x)) ≥ 1 thì tồn tại số nguyên tố p đủ lớn để có xthoả |Q(x+ 1)−Q(x)| = √p (do Q(x+ 1)−Q(x) liên tục trên R) là số vôtỉ, vô lý. Vì thế Q(x + 1) − Q(x) là hằng số hữu tỉ. Hơn thế, Q(0) = 0 nênQ(x) = ax với a ∈ Q.Vì vậy P (x) = ax2 + b với a hữu tỉ.

Nhận xét. Bằng một số tính chất của đa thức, đặc biệt là tính chất liên tục, ta cóđược lời giải hoàn chỉnh.

Tìm tất cả đa thức P (x) hệ số nguyên thoả mãn

2n | P (3n) ∀n ∈ N∗

Bài 12

Lời giải. Nếu degP > 0. Ta viết P (x) = (x− 1)k.Q(x) với Q(1) 6= 0 và k ∈ N.Ta có 2n | (3n − 1)k.Q(3n). Chọn n > k và n lẻ thì

n ≤ kv2(3n − 1) + v2(Q(3n)) = k + v2(Q(3n))

⇒ v2(Q(3n)) ≥ n− k

2.4 Số học 119

Do đó Q(3n)... 2n−k với n > k và n lẻ.

Với n chẵn ta cón ≤ k(v2(n) + 2) + v2(Q(3n)).

Chọn n đủ lớn và v2(n) phù hợp, ta có v2(Q(3n))... 2m với m ≤ n− k(v2(n) + 2).

Từ đó Q(1)... 2m với m lớn đủ (vì Q(3n)−Q(1)

... 3n − 1).Khi đó suy ra Q(1) = 0 vô lý.Vì vậy degP = 0 nên P (x) là hằng số, suy ra P (x) = 0 ∀x ∈ R.

Nhận xét. Bài toán khá khó kết hợp giữa bổ đề nâng lũy thừa LTE và cách chọnsố cho phù hợp. Kĩ thuật chọn số là một trong những kĩ thuật quan trọng nhấtkhi tiếp cận các bài toán số học.

Tìm các đa thức P (x) hệ số nguyên, a, b nguyên dương và a > b sao cho:

P (n) | an − bn ∀n ∈ N∗

Bài 13

Lời giải. Trước hết xin phát biểu không chứng minh bổ đề dưới đây.

Bổ đề 4. Với đa thức P (x) hệ số nguyên và degP > 0, tập {p ∈ P : ∃n ∈ Z+ : p |P (n)} là tập vô hạn.Chứng minh bổ đề trên dễ dàng xin giành cho bạn đọc.

Trở lại bài toán.

• Nếu degP = 0 thì P (x) = d với d | a− b.

• Nếu degP > 0, lấy p nguyên tố sao cho p | P (n0) với n0 nguyên dương vàp > a > b thì có p | an0 − bn0 và p | an0+p − bn0+p do P (n0) ≡ P (n0 + p)(mod p).Lại có an0 ≡ bn0 (mod p) nên p | an0(ap − bp). Mặt khác p > a > a− b nện

p 6 ... a− b. Nhưng ap − bp ≡ a− b (mod p), suy ra điều vô lý.

Vậy P (x) = d ∀x ∈ R và d | a− b.

120 HƯỚNG DẪN GIẢI

Tìm tất cả các đa thức P (x) hệ số nguyên sao cho

(P (n);P (2017n)) = 1 ∀n ∈ Z+

Bài 14

Lời giải. Nếu degP > 0 thì tồn tại p nguyên tố là ước của P (2017m) với mnguyên dương nào đó (áp dụng bổ đề Schur).Theo định lý thặng dư Trung Hoa, ta tìm được số n sao cho{

n ≡ 2017m (mod p)

n ≡ m (mod p− 1))

Lúc đó P (n) và P (2017n) cùng chia hết cho p, vô lý.Do đó degP = 0. Vì vậy P (x) = 1∀x ∈ R hoặc P (x) = −1∀x ∈ R.

Nhận xét. Ta cần một giá trị đa thức cụ thể làm "mốc". Sau đó bằng cách áp dụngđịnh lý thặng dư Trung Hoa ta tìm được các số và đa thức tương ứng thoả mãn.

Cho số nguyên dương d. Gọi f(d) là số nguyên dương nhỏ nhất có đúng d

ước nguyên dương. Chứng minh rằng f(2k+1)... f(2k) ∀k ∈ N.

Bài 15

Lời giải. Với k nguyên dương bất kì, số ước của số nguyên dương f(2k) =Πni=1p

αii là Πn

i=1(αi + 1) = 2k. Khi đó αi = 2βi − 1 = 1 + 2 + . . . + 2βi−1 với mọii = 1, n.Lúc đó ta viết

f(2k) = Πmi=1Π

βi−1j=1 p

2j

i .

Ta xét tập T = {p2m|p nguyên tố ,m ∈ N}. Khi đó f(2k) bằng tích của k phần tửnhỏ nhất của tập T . Vì vậy dễ thấy đpcm.

Nhận xét. Bằng một cách viết khác, ta tìm được quy luật của số f(d).

2.5 Tổ hợp 121

2.5 Tổ hợp

23 người bạn muốn cùng nhau chơi bóng đá. Họ sẽ phải chọn ra một ngườilàm trọng tài và 22 người còn lại chia làm hai đội đá với nhau. Họ muốnchia sao cho tổng cân nặng của mỗi đội là bằng nhau. Giả sử cân nặng củatừng người trong số 23 người là các số nguyên dương và với bất kì cáchchọn trọng tài nào thì họ cũng có thể chia thành hai đội mà tổng cân nặngcủa mỗi đội bằng nhau. Chứng minh rằng 23 người này có cân nặng bằngnhau.

Bài 1

Lời giải. Đặt a1, a2, .., a23 là cân nặng của những người này và S = a1 +a2 + ..+a23. Từ giả thiết, ta có nếu bỏ mỗi ai ra khỏi S thì ta đều chia được hai đội cótổng khối lượng bằng nhau. Do đó, S − ai = 2P . Điều này chứng tỏ rằng S vàcác số ai có cùng tính chẵn lẻ. Ta thực hiện tạo ra một bộ b1, b2, .., b23 mới cũngthoả mãn yêu cầu của đề bài bằng cách:

• Nếu tất cả ai đều chẵn thì đặt bi = ai2

• Nếu tất cả các ai đều lẻ thì đặt bi = ai − 1

Nếu bi không đồng thời bằng 0 thì ta vẫn tiếp tục thực hiện quá trình trên, khiđó, tổng S sẽ giảm sau mỗi quá trình và giảm tới 0. Từ đó, suy ngược lại thì tađược các ai bằng nhau, do đó cân nặng của mỗi người là bằng nhau.

Nhận xét. Chúng ta vừa sử dụng mô hình đơn biến (một số tác giả gọi là phươngpháp xuống thang) để chứng minh bài toán trên, và đó cũng chính là mấu chốtquan trọng nhất của bài toán, đưa số cân nặng giảm dần. Một số bài toán sauđây cũng sử dụng mô hình này để giải nhưng mức độ phức tạp hơn, và cần thêmnhững lập luận để đi tới kết quả chứ không đơn giản như bài tập này.

Ví dụ 1. Cho 2017 số nguyên dương đặt trên một vòng tròn. Biết rằng với 6 sốliên tiếp tuỳ ý, ta luôn có thể chia chứng thành hai nhóm ba có tổng bằng nhau.Chứng minh rằng tất cả các số đã cho bằng nhau.

Hướng dẫn. Ta có, cứ 6 số liên tiếp thì chia thành hai nhóm 3 bằng nhau nên

a1 + a2 + a3 + a4 + a5 + a6... 2.

122 HƯỚNG DẪN GIẢI

a2 + a3 + a4 + a5 + a6 + a7... 2.

⇒ a1 ≡ a7 (mod 2)

Tương tự, ta suy ra đượcai ≡ ai+6 (mod 2)

Do 6 và 2017 có cùng tính chẵn lẻ nên tất cả các số trên vòng tròn có cùng tínhchẵn lẻ. Ta thực hiện thao tác sau liên tiếp:

• Nếu các số ai trên vòng cùng chẵn thì thay ai bởiai2

.

• Nếu các số ai trên vòng cùng lẻ thì thay ai bởiai + 1

2.

Khi đó, bộ các số ai mới vẫn thoả yêu cầu đề bài. Tuy nhiên tổng S = |a1|+ . . .+|a2017| sẽ giảm ngặt nếu có một số trong bộ bằng 1. Vì ta thu được một bộ các sốtoàn các số 1 nên dãy ban đầu có các số ai bằng nhau.

Nhận xét. Bài toán trên có thể thay việc chia 2 nhóm thành 3, 4, 5, ... nhóm màvẫn giải được bằng cách tương tự. Ta xét các bài tương tự sau.

Ví dụ 2. Bộ 5 số nguyên là tốt nếu có thể đặt chúng là a, b, c, d, e để a−b+c−d+e =29. Tìm tất cả các bộ 2017 số sao cho 5 số liên tiếp bất kì trong chúng đều tốt.

Hướng dẫn. Ở bài toán này, điểm khó là không biết các số đã cho có dương haykhông, vì thế, đại lượng tổng ở trên không xét tiếp tục được. Tuy nhiên, cách ápdụng vẫn tương tự như sau:

• Từ tất các các số của bộ cho 29, ta thu được điều kiện tốt trở thành a− b +c− d+ e = 0.

• Tất các các số đã cho cùng là số chẵn.

• Xét đại lượng S = Σ2017i=1 |ai2 | thì thông qua phép chia 2, tổng này phải giảm

ngặt. Từ đó suy ra tất cả các số này phải là 0 và tất cả các số ban đầu là 29.

Ví dụ 3 (VMO 2014). Tìm tất cả các bộ 2014 số hữu tỉ không âm sao cho nếu bỏđi bất kì số nào trong chúng thì các số còn lại có thể chia được thành 3 nhóm rờinhau, mỗi nhóm có 671 số sao cho tích các số trong mỗi nhóm là bằng nhau.

2.5 Tổ hợp 123

Hướng dẫn. Bài toán này khó hơn các bài trước vì: các số cho trong đề bài là sốhữu tỉ mà không phải là số nguyên như các bài trước và phép toán đề cập trongbài này là phép toán nhân mà không phải là cộng. Ta lần lượt giải quyết hai vấnđề này như sau:

• Qui đồng mẫu để đưa về số nguyên.

• Xét số mũ của 1 ước nguyên tố để đưa về dạng tổng.

• Chú ý thêm trường hợp số 0 (nếu có 1 số thì phải có ít nhất 4 số).

Bạn An chơi trò chơi xếp hình với luật chơi như sau. Cho một hình vuông4 × 4 chia thành 16 ô, có 15 mảnh ghép và một ô trống. Trong mỗi bướcchơi, An sẽ được phép trượt các mảnh ghép vào ô trống để thu được hìnhmới. Bạn An sẽ thắng nếu sau hữu hạn bước trượt, An thu được hình nhưsau:

Hỏi An có thể chiến thắng nếu hình ban đầu là hình sau hay không?

Bài 2

Lời giải. Ta đánh số các ô trong bảng ban đầu từ 1 đến 15 theo chiều từ tráisang phải, từ trên xuống dưới. Ta cũng thực hiện đánh số các miếng ghép trong

124 HƯỚNG DẪN GIẢI

bảng ban đầu với thứ tự như vậy. Đối với một hình bất kì, gọi các miếng ghéptrong bảng tại vị trí i là ai. Khi đó, bảng ban đầu sẽ có ai = i,i = 1, .., 15. Ta sẽđếm số các cặp nghịch đối (i, j) sao cho i < j, ai > aj. Ban đầu thì số cặp (i, j)

này là bằng 0. Một phép trượt theo hàng sẽ không làm thay đổi số lượng cặpnày. Một phép trượt theo cột sẽ làm thay đổi lượng này một số lẻ. Mà vì ở hìnhban đầu và hình cuối cùng thì ô trống đều nằm ở góc phải dưới nên số phéptrượt theo cột là chẵn. Điều đó chứng tỏ rằng tính chẵn lẻ của số cặp nghịch đốiđược bảo toàn. Do đó, ta không thể chuyển hình ban đầu về thành hình cuốisau hữu hạn bước.

Nhận xét. Bài toán trên có đề cập tới khái niệm cặp ngịch đối, được cải biên từbài toán sau đây.

Ví dụ 4 (Pablo Soberon Bravo). Cho 2017 số nguyên dương xếp theo thứ tự. Ởmỗi bước, ta thực hiện đổi chỗ 2 số bất kì, hỏi sau 2017 bước nào đó ta có thể thuđược dãy ban đầu hay không?

Ta có thể dễ dàng khẳng định được là không thông qua việc đếm số cặp nghịch đốitrong dãy.

Cho một dãy vô hạn a1, a2, .., an, ... với a1 = 1 và mỗi n > 1 thì:

• Nếu ước lẻ lớn nhất của n đồng dư với 1 module 4 thì an = an−1 + 1.

• Nếu ước lẻ lớn nhất của n đồng dư với 3 module 4 thì an = an−1 − 1.

1. Chứng minh rằng trong dãy số đó mỗi số nguyên dương xuất hiện vôsố lần.

2. Đặtbn = min

i∈N(ai = n)

Tìm một công thức tính bn theo n.

Bài 3

Lời giải. Đề bài khiến ta tìm ước lẻ lớn nhất của một số tự nhiên bất kì, do đó,mỗi số tự nhiên an, ta sẽ thực hiện phân tích an = 2m.pn với pn lẻ thì pn chínhlà ước lẻ lớn nhất của a. Để có cái nhìn rõ ràng về sự tăng giảm của dãy số an,

2.5 Tổ hợp 125

ta cần xét dãy số qn sau đây

qn =

{1, nếu pn ≡ 1 (mod 4)−1, nếu pn ≡ 3 (mod 4)

Ta dựa vào bảng sau để đoán qui luật của an.

q a1 1 12 1 23 1 −14 1 25 1 36 −1 27 −1 18 1 29 1 310 1 411 −1 312 −1 213 1 314 −1 215 −1 116 1 217 1 318 1 419 −1 320 1 4

Ta có thể dễ dàng đoán được quy luật của dãy số này là tăng từ 1 lên 2, sau đógiảm về 1, xong lại tăng lên 3, rồi lại giảm về 1,... Việc này được xảy ra vô hạnlần nên mỗi số tự nhiên xuất hiện vô hạn lần trong dãy, và đây chính là điềumà người giải hướng đến. Bài toán này có một điểm thú vị là dãy số đề cho córất nhiều tính chất. Do đó, mấu chốt là phải lựa chọn một số tính chất có lợicho việc chứng minh. Ở đây, mình xin phép đưa ra một lời giải cho bài toán này.Một số tính chất mà ta sẽ sử dụng đến đó là

• q2m = 1, ∀m

• i, j đối xứng nhau qua 2k thì qi = −qj

126 HƯỚNG DẪN GIẢI

• a2m−1 = 1,∀m

• Tồn tại một số hạng an0 với 2m ≤ n0 < 2m+1 sao cho un0 = m+ 1

Bằng việc chứng minh 4 tính chất này và sử dụng các lập luận rằng trong mỗiđoạn 2m − 1 đến 2m+1 − 1 thì dãy số an tăng từ 1 đến m + 1 rồi giảm xuống 1theo bước nhảy là 1, ta sẽ thu được kết quả là dãy số an lặp lại vô hạn lần.

1. Xét dãy (qn) với

qn =

{1, nếu pn ≡ 1 (mod 4)−1, nếu pn ≡ 3 (mod 4)

thì ta có an = an−1 + qn. Ta chứng minh 4 tính chất sau

(a) q2m = 1,∀m. Tính chất này có được hiển nhiên do 2m ≡ 1 (mod 4)

(b) Nếu i+j2

= 2m thì qi = −qj. Thật vậy, không mất tính tổng quát ta giảsử i < j và i = 2k.p ≡ 1 (mod 4) thì j = 2m+1−2k.p = 2k(2m+1−k−p)nên ước lẻ lớn nhất của j là 2m+1−k−p ≡ −1 (mod 4) . Do đó qi = −qj

(c) a2m−1 = 1. Ta có a1 = 1. Ta xét các bộ đối xứng qua 2m là:

(1, 2m − 1), (2, 2m − 2), .., (2m−1 − 1, 2m−1 + 1)

nên theo tính chất 3, qn tại các bộ này là đối nhau. Hơn nữa, a1 =a2m−1 = 1 nên từ 2 đến 2m − 1 có số lần xuất hiện 1 và −1 trong dãyqn là bằng nhau, do đó u2m−1 = u1 = 1

(d) ∀m,∃n0, 2m ≤ n0 < 2m+1 : un0 = m + 1 (*). Ta chứng minh điều này

bằng qui nạp(*) đúng với m = 1 do từ u2 đến u22 = u4 có u2 = 2.Giả sử (*) đúng với m. Ta chứng minh (*) đúng với m+ 1. Ta có:un0 = m + 1 và u2m+1−1 = 1 nên từ vị trí n0 + 1 đến 2m+1 − 1 thìdãy qn có lượng số 1 nhiều hơn tổng số số −1 là m. Xét đối xứngqua vị trí 2m+1 thì dãy số an tăng m đơn vị từ vị trí 2m+1 + 1 đến2m+1 +2m+1−1−n0 = 2m+2−1−n0. Hơn nữa, q2m+1 = 1 nên từ vị trí2m+1 đến vị trí 2m+2− 1−n0 thì an tăng m+ 1 đơn vị. Do a2m+1−1 = 1nên a2m+2−1−n0

= m+ 2.Vậy, (*) đúng với n+ 1. Do đó, tính chất 4 được chứng minh.

2.5 Tổ hợp 127

Theo tính chất 3 và 4, ta có trong mỗi đoạn từ 2k − 1 đến 2k+1 − 1 thì antăng từ 1 đến n + 1 và giảm lại tới 1, do đó, các số tự nhiên được lặp lạivô hạn lần trong an

2. Từ lập luận trong tính chất 4 thì ta có được rằng, vị trí đầu tiên của aibằng m sẽ có được từ vị trí aj bằng m − 1 bằng cách lấy đối xứng của iqua 2m rồi cộng 1. Do đó, ta có được công thức truy hồi của bn là:

bn = 2n − 1− bn−1

. Từ đó, ta có thể chứng minh công thức của bn bằng qui nạp như sau:{b2n = 22n−1 + 22n−3 + ..+ 21 = 22n+1−2

3

b2n+1 = 22n + 22n−2 + ..+ 20 = 22n+2−13

Vậy, công thức tổng quát của bn là:{b2n = 22n−1 + 22n−3 + ..+ 21 = 22n+1−2

3

b2n+1 = 22n + 22n−2 + ..+ 20 = 22n+2−13

128 HƯỚNG DẪN GIẢI

Cho lục giác đều ABCDEF cạnh 1 được điền các số như hình vẽ

0

1

0

1

1

2

A

BF

C

D

E

Có một con ếch ở vị trí A nhảy xung quanh các đỉnh của đa giác với độ dàicác bước nhảy nguyên. Gọi m là số cách nhảy của ếch sao cho tổng các sốnó nhảy qua là 2017. Chứng minh rằng m không là số chính phương.

Bài 4

Lời giải. Ta thấy 4ACE và 4BDF là hai tam giác đều có cạnh là√

3 nên mỗilần nhảy, ếch sẽ nhảy từ một đỉnh của tam giác này sang đỉnh của tam giác kia.

Chia nhóm I = (A,C,E) tương ứng với các số (0, 0, 1) và II = (B,D, F ) tươngứng với các số (1, 1, 2). Ta thấy {x+ y|x ∈ I, y ∈ II} = {1, 1, 1, 1, 2, 2, 2, 2, 3}chứng tỏ tổng các số trên hai bước nhảy liên tiếp của ếch có thể nhận được là 4trường hợp bằng 1, 4 trường hợp bằng 2 và 1 trường hợp bằng 3. Nếu gọi sn làsố hành trình của ếch có tổng là n thông qua chẵn bước thì

sn = 4sn−1 + 4sn−2 + sn−3 (∗)

Một cách tương tự, gọi tn là số hành trình của ếch có tổng là n thông qua lẻbước thì công thức truy hồi vẫn là (*). Vì vậy, nếu gọi un = sn + tn là số hànhtrình của ếch sao cho tổng các số nhảy qua là n thì

un = 4un−1 + 4un−2 + un−3, với n ≥ 3

2.5 Tổ hợp 129

Ta có u0 = 1, u1 = 6, u2 = 28 và từ công thức truy hồi thì m = u2017 ≡ 2 (mod4)nên m không thể là số chính phương

Nhận xét. Bài toán có thể giải bằng cách đặt 6 dãy truy hồi an, bn, cn, dn, en, fnchỉ số hành trình của ếch có tổng bằng n và kết thúc tại A,B,C,D,E, F . Tuynhiên, cách tiếp cận đó khá phức tạp, đòi hỏi phải khác thác nhiều mối liên hệgiữa các đường đi. Ta cũng có một bài toán tương tự

Ví dụ 5 (Saudi Arabia TST 2017). Người ta đặt các số 1, 2, 3, 4 trên vòng tròntheo thứ tự đó. Một con kiến xuất phát từ số 1 và ở mỗi bước, nó s4 bò qua số bêncạnh. Hỏi con kiến có bao nhiêu cách bò sao cho tổng tất cả các số mà nó bò qua(kể cả số ban đầu) là 21.

Cho hai dung dịch A và B thoả mãn:

i. Số đo khối lượng của 1 lít A bằng số đo thể tích của 1kg B.

ii. p lít dung dịch A nặng bằng q lít dung dịch B với p, q là số nguyên tốphân biệt.

Người ta chia các dung dịch A và B vào các bình giống nhau chứa 1 lítdung dịch và vỏ bình nặng 1kg. Chứng minh rằng có duy nhất 1 cách đểghép các bình cùng loại A hoặc B với nhau sao cho tổng khối lượng thuộc(2017, 2018).

Bài 5

Lời giải. Trước hết, ta xét định lí Beatty với nội udng như sau: Cho hai số vô tỉdương α, β. Xét hai dãy số

A = ([nα])

B = ([nβ])

1

α+

1

β= 1 khi và chỉ khi A,B là phân hoạch của Z+.

Định lí này có thể chứng minh bằng cách sử dụng các bất đẳng thức về phầnnguyên. Ta sẽ áp dụng định lí này để giải bài toán trên. Gọi x, y lần lượt là khốilượng riêng của các dung dịch thì d 1

x= 1 ·y, px = qy nên x =

√qp, y =

√pq. Khối

lượng mỗi bình là α = 1 +√

qp, β = 1 +

√pq.

130 HƯỚNG DẪN GIẢI

Dễ thấy 1α

+ 1β

= 1, thoả mãn định lí Beatty. Suy ra hai dãy [mα], [nβ] là phân

hoạch của số nguyên dương nên ta có đpcm

Nhận xét. Ta có một số bài toán tương tự như bài toán này

Ví dụ 6 (APMO 2006). Với mỗi số nguyên dương n, gọi an, bn lần lượt là số cáchviết 10n trong hệ nhị phân, ngũ phân. Chứng minh rằng (an), (bn) là phân hoạchcủa Z+\{1}.

Để giải bài này, chú ý rằng: số chữ số của M trong hệ p phân là [logpM ]+1. Ngoàira, α = log210, β = log510 thoả mãn điều kiện của định lý Beatty. Từ đó, ta cũngcó một nhận xét thú vị rằng: tổng số chữ số của 2n và 5n trong hệ thập phân làn+ 1.

Ví dụ 7 (VN TST 2000). Cho số nguyên dương k . Dãy số (un) xác định bởi:u1 = 1 và un+1 là số nguyên dương nhỏ nhất không thuộc tập hợp

{u1, u2, . . . , un, u1 + k, u2 + 2k, . . . , un + nk}

Chứng minh rằng tồn tại α vô tỷ dương sao cho un = [nα] với mọi n.

Đây là một kết quả có từ 1959. Ta có thể phân tích cách tiếp cận như sau: Xuấtphát từ việc α =

√2, β =

√2 + 2 thoả mãn điều kiện Beatty. Ta có hai dãy với

công thức an =[n√

2], bn = an + 2n là phân hoạch của Z+ . Từ đó, để giấu

dãy bn đi, ta chỉ cần xét an + 2n . Để ý a1 = 1, a2 = 2, a3 = 4, b1 = 3, b2 =6, b3 = 10 nên a4 có thể định nghĩa là số nguyên dương nhỏ nhất không thuộc{a1, a2, a3, a1 + 2, a2 + 4, a3 + 6} . Đó chính là cơ sở để có bài toán trên. Việc chứngminh hai dãy trùng nhau chỉ cần dùng quy nạp, chú ý rằng 1

α+ 1

α+k= 1 chỉ có

một nghiệm vô tỷ dương duy nhất.(Dãy Wythoff) Cho chuỗi S1 = 1 . Chuỗi Sn được tạo thành từ chuỗi Sn−1 bằngcách thay 1 → 01 và 0 → 1. Các chuỗi S1, S2, S3, . . . được ghép liên tiếp lại vớinhau thành một chuỗi vô hạn L . Gọi an là vị trí của số 1 thứ n trong chuỗi L.Chứng minh rằng tồn tại α vô tỷ dương sao cho

an = [nα] , ∀n.

Ở đây ta có nhận xét rằng số 0 thứ n được sinh ra từ số 1 thứ n nên nếu gọi kn làsố các số 0 đứng trước số 1 thứ n và bn là vị trí của số 0 thứ n, ta sẽ có an = n+ kn

và bn = 2n+ kn nên bn = an + n. Do đó1

α+

1

α + 1= 1 hay α là tỉ số vàng.

2.5 Tổ hợp 131

Có 2020 người đến một buổi tiệc được chia vào các phòng khác nhau saocho

i. Không người nào trong một phòng quen biết tất cả các người trongphòng đó

ii. Trong nhóm 3 người bất kì thuộc cùng một phòng, luôn tồn tại ít nhất2 người không quen biết nhau.

iii. Với bất kì một nhóm 2 người nào trong một phòng mà không quenbiết lẫn nhau, tồn tại đúng một người trong cùng phòng đó quen biếtcả hai người này.

1. Chứng minh rằng trong mỗi phòng, mỗi người có số người quen bằngnhau.

2. Tìm số phòng lớn nhất.

Bài 6

Lời giải. Đề bài khiến ta nghĩ đến cách giải bằng đồ thị. Ta xét người ở trongmột phòng bất kì và biểu diễn mỗi người là một đỉnh, hai người quen biết nhausẽ tạo thành một cạnh. Do vai trò các phòng là như nhau nên để giải ý 2 ta cầntìm số n là số người trong một phòng sao cho nó nhỏ nhất. Ta phát biểu lại bàitoán dưới ngôn ngữ của cạnh, đỉnh và các đa giác như sau:

Bài toán. Cho đồ thị gồm n đỉnh thoả mãn các điều kiện sau

1. Không đỉnh nào có thể được nối với các đỉnh còn lại

2. Không tồn tại tam giác trong đồ thị

3. Với hai đỉnh bất kì A,B mà không có cạnh AB, tồn tại duy nhất C sao chocó cạnh CA,CB

Chứng minh rằng mọi đỉnh đều có cùng số bậc. Tìm số n bé nhất thoả mãn yêucầu.

1. Xét một đỉnhA bất kì, giả sử degA = m tức ồn tại đúngm đỉnhB1, . . . , Bm

được nối với A. Do không có tam giác nên theo điều kiện 2, với mọi i 6= jthì không tồn tại cạnh BiBj. Theo tính duy nhất của điều kiện 3, tất cả

132 HƯỚNG DẪN GIẢI

các đỉnh khác A đều không thể cùng nối tới được Bi và Bj. Do đó, số đỉnhkhác A được nối tới Bi là degBi − 1, giả sử các đỉnh đó là Cij. Lại theođiều kiện 2 thì những đỉnh nối được tới đỉnh Cij thì không thể nối tới Bi,giả sử các đỉnh đó là các đỉnh Chk. Theo điều kiện 2 thì Cij không thể nốitới Cik và theo điều kiện 3 thì với k 6= i bất kì, Cij không thể nối tới cùngCkh và Ckh′ (do Bk 6= Cij nối tới Ckh và Ckh′). Do đó, với k 6= i bất kì thìCij nối được tới nhiều nhất là một Ckh nào đó. Nhưng theo 3, tồn tại duynhất đỉnh X nối được tới Bk và Cij, mà các đỉnh nối tới Bk là A và Ckh .Mà A không thể nối tới Cij nên với mọi k 6= i, tồn tại duy nhất Ckh nối tớiCij. Vậy, Cij được nối tới Bi và thêm m− 1 điểm nữa nên degCij = m.

Ta tiếp tục quá trình với đỉnh Bi thay cho đỉnh A rồi lặp lại toàn bộ quátrình, ta có được các đỉnh có cùng bậc.

Vậy, trong một phòng thì số người quen của mỗi người là bằng nhau

2. Gọi bậc của mỗi đỉnh là m. Khi đó, ta có được m đỉnh Bi và m(m−1) đỉnhCjk nên n = m2 + 1. Với m = 1 thì không thoả 1. Với m = 2 thì n = 5 . Tathấy trường hợp sau thoả yêu cầu bài toán

A

B

C

DE

Vậy n nhỏ nhất là 5 nên số phòng nhiều nhất có thể chia là 2020 : 5 = 404phòng.

2.5 Tổ hợp 133

Cho tập hợp A = {1, 2, . . . , 2n} với n nguyên dương. Một hoán vị các phầntử của A được gọi là đẹp nếu như có ít nhất hai phần tử hơn kém nhaun đơn vị đứng cạnh nhau. Chứng minh rằng số hoán vị đẹp nhiều hơn sốhoán vị không đẹp

Bài 7

Lời giải. Cách giải đơn giản nhất mà ta có thể nghĩ đến là dùng nguyên lí bùtrừ và đếm thẳng số hoán vị đẹp ra và so sánh nó với số hoán vị bất kì. Gọi Ailà tập tất cả những hoán vị sao cho trong các hoán vị đó, hai phần tử k và k+nđứng kề nhau. Khi đó, theo nguyên lí bù trừ thì số hoán vị đẹp là

n∑i=1

|Ai| −n∑

i 6=j=1

|Ai ∩ Aj|+n∑

i 6=j 6=k=1

|Ai ∩ Aj ∩ Ak| − . . .− (−1)n

∣∣∣∣∣n⋂i=1

Ai

∣∣∣∣∣= 2C1

nC12n−1(2n− 2)!− 22C2

nCn2n−2(2n− 4)! + . . .

=2n!(2n− 1)!

(n− 1)!(2n− 2)!.(2n− 2)!− 4n!(2n− 2)!

(2!)2(n− 2)!(2n− 4)!.(2n− 4)!

≥ 2n(2n− 1)!− 4n(n− 1)(2n− 2)

4

= 2n!− (2n− 2)!

42n(2n− 1) +

2n(2n− 2)!

16

>(2n)!

2

Vậy, số hoán vị đẹp nhiều hơn số hoán vị không đẹp.

Nhận xét. Ngoài cách sử dụng nguyên lí bù trừ (mang nặng tính toán) thì ta vẫncòn một vài hướng tiếp cận khác, ở đây chúng tôi sẽ trình bày hướng thứ 2, đó làsử dụng ánh xạ. Việc phải chứng minh số hoán vị đẹp nhiều hơn số hoán vị khôngđẹp khiến ta nghĩ đến việc liệu có tồn tại một ánh xạ là đơn ánh nhưng không toànánh được xây dựng từ tập các hoán vị không đẹp vào tập hoán vị đẹp hay không?Xét một hoán vị không đẹp {x1, x2, . . . , x2n} thì tồn tại duy nhất k ≤ 2n − 2 saocho |xk − x2n| = n. Ta xét phép đặt tương ứng hoán vị không đẹp trên với hoán vịđẹp {x1, x2, . . . , xk, x2n, xk+1, . . . , x2n−1} thì phép này là ánh xạ. Ta kiểm tra đượcánh xạ này là đơn ánh và không là toàn ánh (bạn đọc tự kiểm tra), từ đó suy rasố hoán vị đẹp nhiều hơn số hoán vị không đẹp.

134 HƯỚNG DẪN GIẢI

Cho n là một số nguyên dương và S là tập hợp các điểm (x, y) trên mặtphẳng với x, y không âm và x + y < n. Các điểm trong S được tô màu đỏhoặc xanh theo qui luật, nếu (x, y) là màu đỏ thì (x′, y′) được tô màu đỏ vớix ≤ x′ và y ≤ y′. Đặt A là số cách chọn n điểm xanh mà hoành độ x của nókhác nhau và đặt B là số cách chọn n điểm xanh mà tung độ y của chúngkhác nhau. Chứng minh rằng A = B

Bài 8

Lời giải. Ta có hình minh hoạ với n = 6 như sau:

1. 2. 3. 4. 5.

1.

2.

3.

4.

5.

0

A

B

C

D

E

F

GHIJK

L

M

N

O

P

Q R

S T U

1. 2. 3. 4. 5.

1.

2.

3.

4.

5.

0

A

B

C

D

E

F

GHIJK

L

M

N

O

P

Q R

S T U

1. 2. 3. 4. 5.

1.

2.

3.

4.

5.

0

A

B

C

D

E

F

GHIJK

L

M

N

O

P

Q R

S T U

Gọi a0, .., an−1 là số điểm xanh có hoành độ bằng 0, 1, .., n − 1. Gọi b0, .., bn−1 làsố điểm xanh có tung độ bằng 0, 1, .., n − 1. Khi đó, ta có A = a0...an−1, B =b0...bn−1. Ta cần chứng minh A = B hay nói cách khác a0..an−1 = b0..bn−1.

2.5 Tổ hợp 135

Ta nhận thấy rằng nếu trên đường chéo x + y = n− 1 có một điểm màu đỏ thìA = B = 0. Do đó ta chỉ nhìn trên trường hợp mà tất cả các điểm trên đườngchéo đều màu xanh. Tuy nhiên, khi trong trường hợp này, ta lại đưa bài toánvề việc đếm bài toán ban đầu với n − 1, đáp án sẽ có được bằng cách cộngthêm vào mỗi a′i, b

′i với 1 (Với a′i là số điểm xanh có hoành độ bằng i trong hình

(x, y), x + y < n − 1 va bi cũng như vậy). Phân tích này dẫn đến việc chúng taphải giải bài toán này bằng qui nạp. Ta sẽ tìm cách tính A,B với n từ việc tínhA,B với n − 1. Quan sát thêm một chút, việc tô màu như đề cho sẽ cho chúngta các hình tam giác vuông cân, từ đó nảy ra ý tưởng rằng a0, .., an là các hoánvị của b0, .., bn (*). Ta sẽ chứng minh kết quả này bằng qui nạp.

• Với n = 1 thì hình chỉ còn một điểm, khi đó a0 = b0 = 0 hoặc a0 = b0 = 1nên (*) đúng với n = 1.

• Giả sử (*) đúng với n = k, tức a0, .., ak−1 là hoán vị của b0, .., bk−1. Tachứng minh (*) đúng với n = k+ 1. Gọi ck là số điểm xanh có hoành độ ktrong trường hợp k + 1, dk là số điểm xanh có tung độ bằng k. Ta xét haitrường hợp

– Nếu tất cả các điểm trên đường x + y = k đều là màu xanh thì ta cóci = ai + 1, di = bi + 1, i = 0..k; ck = dk = 1 nên ta có đpcm

– Nếu tồn tại một điểm (m, k − m) màu đỏ thì phần (x, y) : x ≤m, y ≤ m đều màu đỏ, đặc biệt cm = dn−m. Xét hai phần P ={(x, y) : x < m, y > k −m} và Q = {(x, y) : x > m, y < k −m} thì tacó ai, ai+1, .., am là hoán vị của b0, b1, .., bk−m tương tự ta suy ra đượcđpcm.

136 HƯỚNG DẪN GIẢI

Cho một dãy n tấm bìa đặt sấp ở trên bàn được đánh số từ 1 tới n. Mỗi lầncho phép thay đổi trạng thái của k tấm bìa liên tiếp từ sấp thành ngửa vàngược lại

1. Chứng minh rằng có thể chuyển hết tấm bìa từ sấp sang ngửa khi và

chỉ khi m... k

2. Nếu n không chia hết cho k, tìm số bìa tối đa có thể chuyển sangngửa.

Bài 9

Lời giải.

1. Chiều thuận là chiều hiển nhiên vì nếu n... k thì n = kp, ta thực hiện đổi p

lần, mỗi lần k tấm theo thứ tự thì các tấm bìa từ sấp được chuyển thànhngửa. Ở chiều đảo, giả sử n không chia hết cho k, tức n = pk+r, 0 < r < k,khi đó, ta thực hiện tô màu các tấm bìa thứ 1, k, k + 1, 2k, . . . , pk, pk + 1thì số tấm được tô màu là một số lẻ. Tuy nhiên, một lần chọn k tấm liêntiếp và đổi trạng thái sẽ luôn làm thay đổi trạng thái của 2 tấm, do đó takhông thể chuyển các tấm trên về ngửa nếu n không chia hết cho k.

Hình 2.1: Trường hợp n = 13, k = 5

Vậy 1 được chứng minh

2. Nếu n không chia hết cho k, tức n = kp+ r, 0 < r < k. Ta để ý đến trườnghợp n = 13, k = 5 ở trên và tìm cách để đổi được nhiều tấm ngửa nhất. Tathực hiện như sau:

• Đổi các tấm từ 1 đến 5 thành ngửa

• Đổi các tấm từ 6 đến 10 thành ngửa

• Đổi trạng thái các tấm từ 9 đến 13, tức 9, 10 thành sấp, 11, 12, 13thành ngửa.

2.5 Tổ hợp 137

Cách đổi này giúp ta thu được 11 tấm ngửa. Tuy nhiên, nếu n = 11 thìta sẽ không thực hiện bước cuối cùng, vì nếu thực hiện như vậy, ta sẽ chỉlàm thêm 1 tấm ngửa nhưng lại mất đi 4 tấm, tức là đã lỗ 3 tấm. Trongtrường hợp này, ta chỉ thực hiện hai bước đầu và thu được 10 tấm ngửa.

Với những phân tích đó, ta sẽ đưa ra lời giải cho hai trường hợp, r ≤ k

2và

r >k

2

(a) Nếu r ≤ n

2, ta chứng minh rằng số tấm bìa được đổi sang ngửa tối

đa là kp. Ta thực hiện các bước sau:

i. Tô màu 1 các tấm bìa ở vị trí 1, k, k + 1 . . . , kp, kp+ 1 thì ta có sốlẻ các tấm bìa, mà mỗi lần đổi sẽ làm thay đổi trạng thái của 2tấm bìa nên tồn tại 1 tấm bìa không thể đổi sang ngửa.

ii. Tô màu 2 các tấm bìa ở vị trí 2, k − 1, k + 2, . . . , kp− 1, kp+ 2 thìta có số lẻ các tấm bìa mà mỗi lần đổi sẽ làm thay đổi trạng tháicủa 2 tấm bìa nên tồn tại 1 tấm bìa không thể đổi sang ngửa.Hơn nữa, các tấm bìa này khác các tấm bìa ở lần trên nên ta suyra có 2 tấm bìa không thể đổi sang ngửa

Hình 2.2: Trường hợp n = 16, k = 7

Tương tự như vậy đến r <k

2thì ta được có ít nhất r tấm bìa không

thể đổi sang ngửa, tức có tối đa kp tấm bìa đổi sang ngửa, và cáchxây dựng để kp tấm sang ngửa thì ta đã biết ở trên. Cách tô màu như

trên sẽ không thể áp dụng trong trường hợp r >k

2vì sẽ bị trùng các

màu với nhau. Điều đó dẫn đến kết quả khác cho trường hợp r >k

2

(b) Nếu r >k

2, ta thực hiện tô màu như trường hợp trên nhưng chỉ đến

bước k − r, ta thu được có ít nhất k − r tấm bìa không thể đổi sangngửa, tức có tối đa kp+ 2r− k tấm bìa được đổi sang ngửa, cách đổithoả mãn có kp+2r−k tấm bìa thành ngửa như sau: Đổi lần lượt cáctấm từ 1 đến kp thành ngửa theo lượt, mỗi lượt k tấm. Sau đó, đổi k

138 HƯỚNG DẪN GIẢI

tấm cuối cùng, ta thu được số tấm ngửa là n− (k − r) = n+ r − k =kp+ 2r − k

Vậy, nếu r ≤ k

2thì số tấm được đổi sang ngửa tối đa là kp, trường hợp

ngược lại thì số tấm được đổi sang ngửa là tấm kp+ 2r − k tấm.

Trong một bảng vuông n × n, ta đặt những chiếc đèn lên các ô của bảng,mỗi ô một đèn. Ở mỗi lần thay đổi, ta được phép chọn một đèn làm gốc vàthay đổi trạng thái của đèn đó và tất cả các đèn khác cùng hàng cùng cộtvới nó từ tắt sang bật và ngược lại. Với trạng thái ban đầu là bất kì, ta cóthể đưa tất cả đèn về trạng thái bật được hay không với

1. n = 6.

2. n = 2017.

Bài 10

Lời giải. Để tiện cho việc mô tả, ta qui ước một ô có trạng thái bật nếu ô đócó chứa dấu + và trạng thái tắt là dấu -. Ta có nhận xét rằng, việc bất kì trạngthái nào ta cũng có thể đưa về trạng thái tất cả các đèn đều bất tương đươngvới việc ta có thể thay đổi trạng thái của một đèn mà không làm thay đổi trạngthái của các đèn còn lại. Ta xét tới các trường hợp nhỏ trong bài

1. n = 6. Ta sẽ xem xét rằng có thể đổi trạng thái của một ô bất kì mà khônglàm thay đổi trạng thái của các ô còn lại hay không?

Hình 2.3: Trường hợp n = 6

2.5 Tổ hợp 139

Giả sử ô cần đổi trạng thái là ô được mang dấu - ở vị trí như hình trên. Tacần chọn những ô sao cho sau những lần lấy các ô đó làm gốc, thực hiệnthay đổi trạng thái, ta được ô dấu - trở thành ô dấu +, các ô còn lại trongbảng giữ nguyên trạng thái, hay nói cách khác, ô dấu - được tác động lẻlần và các ô còn lại được tác động chẵn lần. Ta nhận thấy rằng, những ôcó thể làm thay đổi trạng thái của ô dấu - là những ô được tô màu. Nhữngô không được tô màu chỉ có thể làm thay đổi trạng thái với những ô cùnghàng hoặc cùng cột với ô dấu - . Do đó, trường hợp đơn giản nhất mà tanghĩ tới sẽ là thay đổi trạng thái các ô tô màu, sau đó mới tính tiếp tớichuyện thay đổi trạng thái của các ô không tô màu để cho phù hợp. Ta xétviệc chọn lần lượt các ô tô màu làm gốc, ta thu được

• Ô dấu − được tác động 11 lần (11 ô tô màu)

• Các ô được tô màu mà khác dấu - được tác động 6 lần (bởi các ô cùnghàng hoặc cùng cột)

• Các ô không được tô màu được tác động 2 lần (một lần bởi ô cùnghàng, một lần bởi ô cùng cột)

Như vậy, ô dấu - đổi thành + và các ô còn lại giữ nguyên trạng thái, dođó, đối với bảng 6× 6 thì ta có thể chuyển hệ bóng đèn ở trạng thái bất kìvề trạng thái bật.

2. Ta để ý rằng, với n bất kì việc tác động như trên sẽ luôn làm ô dấu cộngđược tác động lẻ lần 2n − 1, các ô không tô màu được tác động 2 lần vàcác ô tô màu được tác động n lần. Như thế, việc có thể đổi được theo cáchnày hay không phụ thuộc vào tính chẵn lẻ của n. Với trường hợp n = 6này thì câu trả lời là có. Tuy nhiên với, n = 2017 thì chưa chắc. Ta sẽ mongrằng việc đổi được hay không chỉ phụ thuộc vào tính chẵn lẻ của n nênta sẽ xét trường hợp nhỏ hơn để có cái nhìn trực quan cho bài toán. Xéttrường hợp n = 3 và dấu ta cần đổi vị trí như sau:

Hình 2.4: Trường hợp n = 3

140 HƯỚNG DẪN GIẢI

Dấu − ở góc phải dưới và cần đổi dấu ˘ này thành + mà không làm thayđổi trạng thái của các ô còn lại. Xét bộ ô được tô màu như hình trên, khichọn một trong các ô trong bảng làm gốc và thay đổi trạng thái thì có 4hoặc 2 ô tô màu bị thay đổi trạng thái, tức tổng số các dấu – trên các ô tômàu không bị thay đổi tính chẵn lẻ. Mà việc chỉ đổi trạng thái của ô (3; 3)lại làm thay đổi tính chẵn lẻ của tổng số ô - trong các ô tô màu, tức khôngthực hiện được. Vậy, ta không thể đưa bảng 3 × 3 bất kì về bảng chỉ códấu + được. Từ đây, ta thấy rằng việc chứng minh đối với n = 2017 cũngtương tự như trường hợp n = 3 vì chứng minh của ta chỉ phụ thuộc vàotính chẵn lẻ của n. Do đó, với n = 2017 thì không thể kết luận rằng có thểđổi một trường hợp ban đầu bất kì về trạng thái tất cả các bóng đều bật

Cho một bảng 5×5 được tô trắng đen xen kẽ, các ô ở góc được tô đen. Trênmỗi ô đen có các đồng xu đen và trên các ô trắng có các đồng xu trắng.Các đồng xu có thể di chuyển đến các ô bên cạnh. A và B cùng chơi mộttrò chơi như sau: Đầu tiên, A khởi động trò chơi bằng cách lấy một đồngxu đen ra khỏi bảng rồi di chuyển một đồng xu trắng vào ô trống. Sau đó,B di chuyển một đồng xu đen vào ô trống. Các lượt sau đó, A sẽ di chuyểnmột đồng xu trắng vào ô trống và B di chuyển đồng xu đen vào ô trống.Trò chơi kết thúc khi một trong hai người không thể di chuyển được theoluật trên, và người còn lại là người chiến thắng. Hỏi có chiến thuật thắnghay không, nếu có hãy chỉ ra ai là người thắng?

Bài 11

Lời giải. Để ý rằng cứ đến lượt chơi của A thì ô trống là ô đen và A sẽ luônphải bỏ đồng xu trắng vào ô đen, đến lượt chơi của B thì ô trống là ô trắng vàB sẽ phải bỏ đồng xu đen vào ô trắng. Do đó, trò chơi sẽ kết thúc sau 24 lượtchơi. Ta có thêm nhận xét rằng sau khi khởi động trò chơi, A đã lấy một đồngxu đen ra khỏi bảng, và do đó các ô còn lại được phủ bởi các hình domino 2×1.Lượt đầu A di chuyển, tức là đã lấy một đồng xu ra khỏi một miếng dominonào đó. Nếu B tiếp tục thực hiện như A nhưng đối với phần còn lại của miếngdomino đó thì ta thấy rằng B luôn luôn thực hiện được thao tác của mình saukhi A thực hiện thao tác. Vậy, B là người chiến thắng

2.5 Tổ hợp 141

Cho lưới tam giác đều như hình vẽ, trong đó mỗi cạnh có chứa n điểm(không tính hai đầu mút của cạnh), các đoạn thẳng song song với cạnhtam giác lớn được nối với nhau. Đếm số tam giác đều có đỉnh là các điểmtrong lưới đã cho

B C

A

Bài 12

Lời giải. Ý tưởng đơn giản nhất được đưa ra ở đây là đếm truy hồi, . Gọi bậccủa tam giác đều này là số điểm có trên mỗi cạnh, không chứa đầu mút. Ta sẽtìm công thức truy hồi của dãy Sn với Sn là số tam giác đều trong tam giác lớnbậc n

1

2

3

4

5

2345

1

2

3

4

5

1

B C

A

142 HƯỚNG DẪN GIẢI

Ta thấy rằng, các tam giác trong tam giác bậc n sẽ có thể nằm trong một trongcác trường hợp sau:

• Nằm trong 1 trong 3 tam giác bậc n− 1 lần lượt chứa các đỉnh A,B,C

• Là một trong n tam giác chéo được đánh số như trên hình.

Tuy nhiên, các tam giác nhỏ trong 3 tam giác bậc n − 1 này có thể trùng nhaunên để không đếm trùng lặp, ta phải sử dụng nguyên lí bù trừ. Gọi X,Y ,Z lầnlượt là tập hợp các tam giác nằm trong tam giác bậc n − 1 chứa đỉnh A,B,C.Khi đó, ta có:

Sn = |X|+ |Y |+ |Z| − |X ∩ Y | − |Y ∩ Z| − |Z ∩X|+ |X ∩ Y ∩ Z|+ 3

= 3Sn−1 − 3Sn−2 + Sn−3 + 2

Như vậy, ta có được công thức truy hồi của dãy là

{S0 = 1, S1 = 5, S2 = 15

Sn = 3Sn−1 − 3Sn−2 + Sn−3 + n

Tìm công thức tổng quát từ công thức truy hồi này, ta được

Sn = C4n+4

Nhận xét. Chúng ta cũng có thể thử sức với một bài toán khác, cũng tương tựkiểu đếm này như sau:

Ví dụ 8 (Olympic Sinh viên 2016). Xét lưới 16 × 16 tạo thành từ dãy các điểmnhư hình sau (khoảng ách giữa các hàng và các cột là 1).

2.5 Tổ hợp 143

1. Đếm số hình vuông với đỉnh nằm trên lưới và có diện tích bằng 4. Đáp án:196

2. Đếm số hình vuông với đỉnh nằm trên lưới và có diện tích bằng 25. Đáp án:283

3. Đếm tất cả hình vuông có đỉnh nằm trên lưới. Đáp án: 5440

Cho 2018 bóng đèn đang ở trạng thái sáng được xếp liên tiếp nhau trênmột đường thẳng. Hai người cùng chơi trò chơi như sau: Ở mỗi lượt chơicủa mình, mỗi người sẽ chọn một bóng đèn sáng, sau đó đổi trạng thái củabóng đèn đó cùng với 4 bóng đèn phía sau nó.

1. Chứng minh rằng trò chơi sẽ dừng lại sau hữu hạn bước.

2. Ai có thể luôn là người chiến thắng, hãy đưa ra chiến thuật thắng ấy.

Bài 13

Lời giải.

1. Ta chứng minh bằng quy nạp.

Bài toán đã cho đúng với n = 5 thật vậy: Người thứ nhất có hai khả năngchọn bóng đèn: Một là chọn 4 bóng sau đổi trạng thái thì chỉ có bóng đầu

144 HƯỚNG DẪN GIẢI

sáng người thứ hai phải chọn 4 bóng đầu đổi trạng thái sau đó bóng đầutiên tắt, lúc này người thứ nhất phải chọn bốn bóng sau đổi trạng thái vàtrò chơi kết thúc. Hai là người thứ nhất đổi trạng thái bốn bóng đầu, khiđó người hai chọn bốn bóng sau và trò chơi kết thúc.

Giả sử bài toán đúng tới n = k. Khi n = k + 1 ta thấy rằng nếu trong lúcchơi không ai chọn bóng đầu tiên thì chỉ có k bóng phía sau được chọntrò chơi sẽ dừng theo giả thuyết quy nạp sau đó người chơi tiếp theo phảichọn bóng đầu tiên nếu muốn tiếp tục chơi, sau khi chọn bốn bóng đầutiên thì bóng đầu tiên tắt, ta không chọn nó được nữa. Trò chơi quy vềtrường hợp n=k cũng kết thúc.

2. Ta đánh số các bóng sáng đèn là 1 không sáng đèn là 0. Tổng các số ở vị tríchi hết cho 4 lúc ban đầu là 504 là chẵn. Mỗi lần chơi chọn bốn bóng liêntiếp nên tổng này thay đỗi tính chẵn lẽ một lần. Khi trò chơi dừng tổng là0 vẫn là số chẳn. Nên tổng số lượt chơi là chẳn. Vậy người sau thắng.

Cho 33 điểm khác nhau nằm bên trong một hình vuông có cạnh là 4. Vẽ 33đường tròn nhận các điểm này làm tâm, có cùng bán kính

√2. Chứng minh

rằng tồn tại một đường tròn trong số chúng chứa ít nhất 3 điểm trong số33 điểm nói trên.

Bài 14

Lời giải. Bài toán này là một bài toán Dirichlet khá đơn giản, công việc củachúng ta là chia hình vuông này thành 16 hình sao cho đường kính của mỗihình này nhỏ hơn hoặc bằng

√2. Và tất nhiên, cách chia đơn giản nhất là chia

hình này thành 16 hình vuông đơn vị. Khi đó, theo nguyên lí Dirichler, phái cómột hình vuông đơn vị chứa ít nhất 3 điểm A,B,C nên khoảng cách lớn nhấtcủa A đến B và C là

√2, do đó đường tròn (A,

√2) sẽ chứa B và C.

Nhận xét. Đây là một bài tập về Dirichlet khá đơn giản bởi vì cách chia hìnhkhông đem lại nhiều khó khăn cho người giải, tuy nhiên, đối với những bài toánsau đây thì việc phân chia không phải là dễ dàng

Ví dụ 9. Chứng minh các bài toán sau (Ngô Minh Phương - Các chuyên đề tổ hợpbồi dưỡng HSG)

2.5 Tổ hợp 145

1. Bên trong tam giác đều ABC có cạnh bằng 6, cho 13 điểm phân biệt. Chứngminh rằng tồn tại hai điểm trong số 13 điểm đã cho mà khoảng cách giữachúng không vượt quá

√3

2. Bên trong hình chữ nhật 3× 4, cho 6 điểm phân biệt, chứng minh rằng tồntại hai điểm trong số 6 điểm đã cho mà khoảng cách giữa chúng không vượtquá√

5

Đối với các bài toán ở ví dụ này, ta nhận xét rằng việc đơn giản hơn là ta chỉcần chứng minh tồn tại 2 điểm có khoảng cách nhỏ hơn một số nào đó, ít ra thìcũng dễ hơn 3 điểm. Tuy nhiên, về số phần cần chia là thực sự khác biệt và khôngdễ nhìn như bài toán trên. Ở mỗi bài toán, ta phải chia hình ban đầu thành cácphần sao cho mỗi phần đều có đường kính nhỏ hơn một số cho trước. Và sau đâylà từng cách chia cho mỗi bài toán

1. Ta cần chia tam giác này thành 12 phần, mỗi phần có đường kính nhỏ hơn√3:

2. Ta cần chia hình vuông 3×4 thành 5 hình có đường kính nhỏ hơn√

5. Hìnhđược chia như sau sẽ thoả mãn đề bài

146 HƯỚNG DẪN GIẢI

Tuy nhiên, việc nghĩ ra cách phân chia cho những hình trên là không đơn giản, đasố là "mò". Ở đây, chúng ta sẽ đưa ra một cách chia tương đối tổng quát. Mục tiêuchia hình của chúng ta ở mỗi bài toán là chia thành các đa giác (để dễ trình bàyđáp án) sao cho diện tích mỗi phần là lớn nhất, và đảm bảo đường kính mỗi phầnluôn nhỏ hơn một con số cố định. Chẳng hạn trong câu a, ta cần chia 12 phần cóđường kính nhỏ hơn

√3. Nếu bỏ đi mục tiêu đa giác, ta có thể sẽ nghĩ đến tìm

các hình có diện tích lớn nhất và có đường kính nhỏ hơn√

3 để phủ lên tam giácđều có cạnh là 6 này. Và hình mà chúng ta sẽ dùng ở đây chính là hình tròn. Tasẽ phác thảo sơ bằng cách vẽ các hình tròn đường kính

√3 và phủ chúng lên tam

giác cạnh bằng 6.

Và chỉ cần "cắt tỉa" gọn gàng, ta đã thu được kết quả như trên. Thậm chí, nếu sắpxếp gọn gàng, ta chỉ cần sử dụng 10 hình tròn, và do đó, đề bài có thể sửa lại chỉcần 11 điểm thì ta vẫn có thể giải đượcCũng một công đoạn tương tự như câu 1, ta có thể phác thảo cho câu b và thựchiện "cắt tỉa" để thu được đáp án của bài toán

2.5 Tổ hợp 147

Có một vấn đề nhỏ ở đây là phần còn lại không thể phủ bởi một hình tròn, nhưngmay thay vì đường kính của hình này vẫn nhỏ hơn

√5 và ta có được đáp án.

Có 2010 que diêm trên bàn. A và B cùng chơi trò chơi theo lượt như sau:Đến lượt của mình, họ sẽ lấy đi 1, 3, 4, 5 hoặc 7 que diêm. Người lấy quediêm cuối cùng sẽ chiến thắng. Nếu A chơi trước, hỏi người nào sẽ có chiếnthuật thắng?

Bài 15

Lời giải. Ta có nhận xét rằng không được phép lấy đi 2, 6 que diêm. Nên nếuA lấy a que thì B có thể lấy 8 − a que để được tổng số que lấy đi là 8. Do đó,nếu người nào ép người còn lại vào thế chia hết cho 8 thì người đó sẽ thắng.Do 2010 chia 8 dư 2 nên A với lượt chơi đầu tiên không thể đưa B về trạng tháichia hết cho 8. Nếu A bốc 4 que diêm, thì B lại rơi vào tình trạng không thể đưaA vào trạng thái chia hết cho 8, tuy nhiên, nếu B tiếp tục bốc 4 que và liên tiếpnhư thế thì cuối cùng chỉ còn lại 2 que diêm tại lượt của A và do đó B là ngườichiến thắng. Nếu A bốc số que diêm khác 4 thì B sẽ đưa được A về trạng tháichia hết cho 8 và do đó B chiến thắng.

148 HƯỚNG DẪN GIẢI

Tài liệu tham khảo

[1] Võ Quốc Bá Cẩn, Phạm Thị Hằng, Chuyên đề Bất đẳng thức hiện đại.http://www.mediafire.com/file/334rrf4ib3f8msj/

[2] Võ Quốc Bá Cẩn, Trần Quốc Anh, Sử dụng AM-GM để chứng minh bất đẳngthức.

[3] Nguyễn Văn Huyện, Về một bài toán trong "Hello IMO 2007".http://www.mediafire.com/file/stharkh32qqpcyb/

[4] Trần Nam Dũng, Về một dạng phương trình hàm đa thức.https://diendantoanhoc.net/index.php?app=core&module=attach&

section=attach&attach_id=20496.

[5] Nguyễn Chí Trung, Phương trình hàm đa thức một biến.

[6] Lê Phúc Lữ, Nguyễn Tuấn Anh, Một số vấn đề về đa thức bất khả quy.

[7] Lê Phúc Lữ, Các bài toán về nghiệm của đa thức.

[8] Lê Xuân Đại, Công thức nội suy Lagrange.

[9] Nguyễn Đình Toàn, Nguyễn Đức Anh, Tính chất số học của đa thức.

[10] Diễn đàn Art of Problem Solving https://artofproblemsolving.

com/community/ và Diễn đàn Toán học Việt Nam - VMF http://

diendantoanhoc.net/home/.

[11] Lê Phúc Lữ, Sự kết hợp giữa hình học và đại số trong các bài toán về phângiác, Kỷ yếu Gặp gỡ Toán học 2011.

[12] Nguyễn Văn Linh, Khai thác một bài toán hay từ đề thi Olympic Hình họcSharygin năm 2017.

[13] Tạp chí Mathley.